Está en la página 1de 92

Uno de los primeros libros de fantasía científica (1940), titulado Mr.

Tompkins en el país de las maravillas y escrito por


el físico George Gamow, describía un mundo donde la rapidez de la luz sólo era de 10 m/s (20 mi/h). El señor Tompkins
estudió relatividad y, cuando comenzó a “acelerar” en su bicicleta, “esperaba que su figura se acortara de inmediato, y es-
taba muy feliz acerca de ello ya que su creciente figura últimamente le causaba cierta ansiedad. Sin embargo, para su
gran sorpresa, nada le ocurrió ni a él ni a su bicicleta. Por otra parte, el cuadro alrededor suyo cambió por completo.
Las calles se hicieron más cortas, las ventanas de las tiendas comenzaron a parecer estrechas rendijas, y el policía en la
esquina se convirtió en el hom-
bre más delgado que había visto
jamás. ‘¡Por Júpiter!’, exclamó el
señor Tompkins emocionadísi-
mo, ‘Ahora veo el truco. De aquí
es de donde proviene la palabra
relatividad.’”
De hecho, la relatividad pre-
dice que los objetos que se mue-
ven en relación con nosotros a
gran rapidez, cerca de la rapidez
de la luz c, se acortan en longitud.
Uno no lo nota, como le ocurrió
al señor Tompkins, porque c 5 3
3 108 m/s es una rapidez increí-
ble. En este capítulo se estudia-
rá la contracción de la longitud,
la dilatación del tiempo, la falta
de acuerdo en cuanto a la simul-
taneidad, y cómo energía y masa
son equivalentes (E 5 mc2).

U L
Í T
P

O
La teoría especial
de la relatividad
PREGUNTA DE INICIO DE CAPÍTULO: ¡Adivine ahora!
Un cohete se aleja de la Tierra con una rapidez de 0.80c. El cohete dispara un misil con
una rapidez de 0.70c (el misil se apunta lejos de la Tierra y abandona el cohete a 0.70c en
36
C A

CONTENIDO
36–1 Relatividad galileana-
newtoniana
relación con este último). ¿Con qué rapidez se desplaza el misil en relación con la Tierra?
a) 1.50c; *36–2 El experimento de Michelson y
Morley
b) un poco menos que 1.50c; 36–3 Postulados de la teoría especial
c) un poco más que c; de la relatividad
d) un poco abajo de c; 36–4 Simultaneidad
e) 0.75c. 36–5 Dilatación del tiempo y la
paradoja de los gemelos

L
a física de finales del siglo XIX volvió la vista a un periodo de grandes progre-
36–6 Contracción de la longitud
sos. Las teorías desarrolladas durante los tres siglos precedentes tuvieron mu-
36–7 Espacio-tiempo
cho éxito para explicar una amplia gama de fenómenos naturales. La mecánica tetradimensional
newtoniana explicaba hermosamente el movimiento de los objetos sobre la Tie-
36–8 Transformaciones galileanas y
rra y en los cielos. Más aún, formó la base para tener éxito en los tratamientos de flui- de Lorentz
dos, el movimiento ondulatorio y el sonido. La teoría cinética explicaba el comporta- 36–9 Cantidad de movimiento
miento de los gases y otros materiales. La teoría de Maxwell del electromagnetismo no relativista
sólo conjuntó y explicó los fenómenos eléctricos y magnéticos, sino que también predi- 36–10 La rapidez última
jo la existencia de ondas electromagnéticas que se comportarían como la luz, de mane- 36–11 E 5 mc2; masa y energía
ra que la luz llegó a considerarse como una onda electromagnética. De hecho, parecía 36–12 Corrimiento Doppler para la luz
que el mundo natural, visto a través de los ojos de los físicos, estaba muy bien explica- 36–13 El impacto de la relatividad
do. Todavía quedaban algunos enigmas, pero se creía que éstos se explicarían pronto especial
mediante los principios ya conocidos.

951
Pero esto no resultó tan sencillo. En vez de ello, estos enigmas sólo se resolverían
con la introducción, en la primera parte del siglo XX, de dos nuevas teorías revolucio-
narias que cambiarían toda la concepción de la naturaleza: la teoría de la relatividad y
la teoría cuántica.
A la física, tal como se conocía a finales del siglo XIX (lo que se ha estudiado hasta
ahora en este libro), se le llama física clásica. La nueva física que se desarrolló a partir
de la gran revolución a la vuelta del siglo XX ahora se conoce como física moderna. En
este capítulo se presenta la teoría especial de la relatividad, que propuso Albert Eins-
tein (1879-1955; figura 36-1) en 1905. En el capítulo 37 se presenta la igualmente tras-
cendental teoría cuántica.

36–1 Relatividad galileana-newtoniana


La teoría especial de la relatividad de Einstein trata de cómo se observan los acontecimien-
tos, en particular sobre la forma como los objetos y eventos se observan desde diferentes
marcos de referencia. Desde luego, este tema ya lo habían explorado Galileo y Newton.
La teoría especial de la relatividad trata con acontecimientos que se observan y
FIGURA 36–1 Albert Einstein miden en los llamados marcos de referencia inerciales (secciones 4-2 y 11-8), que son
(1879-1955), una de las grandes mentes los marcos de referencia donde es válida la primera ley de Newton: si un objeto no ex-
del siglo XX, fue el creador de las perimenta fuerza neta, permanece en reposo o continúa en movimiento con rapidez
teorías especial y general de la constante en línea recta. Por lo general, es más fácil analizar los acontecimientos cuan-
relatividad. do los observan y miden observadores en reposo en un marco inercial. La Tierra, aun-
que no es un marco inercial (puesto que gira), está suficientemente cerca de serlo, de
manera que para la mayoría de los propósitos se le puede considerar un marco inercial.
Los marcos de referencia en rotación o que de alguna otra forma aceleran son marcos
no inerciales,† y no se tratarán en este capítulo (la teoría general de la relatividad de
Einstein se ocupa de ellos).
Un marco de referencia que se mueve con velocidad constante con respecto a un
marco inercial también es en sí mismo un marco inercial, pues en él también se sostienen
las leyes de Newton. Cuando se dice que observamos o realizamos mediciones desde
cierto marco de referencia, significa que estamos en reposo en ese marco de referencia.
Tanto Galileo como Newton estuvieron conscientes de lo que ahora se llama prin-
cipio de relatividad aplicado a la mecánica, es decir, eran conscientes de que las leyes
básicas de la física son las mismas en todos los marcos de referencia inerciales. Es posi-
FIGURA 36–2 Una persona deja ble que reconozca su validez en la vida cotidiana. Por ejemplo, los objetos se mueven
caer una moneda en un automóvil en en un tren o un avión que se desplaza suavemente (velocidad constante) de la misma
movimiento. Las imágenes superiores forma que lo hacen en la Tierra. (Esto supone que no hay vibraciones o sacudidas que
muestran el momento de liberación de podrían hacer al marco de referencia no inercial). Cuando usted camina, toma un tazón
la moneda, las inferiores ilustran un de sopa, juega billar o deja caer un lápiz en el suelo mientras viaja en un tren, avión o
momento posterior. a) En el marco de barco que se desplaza con velocidad constante, los objetos se mueven tal como lo ha-
referencia del automóvil, la moneda cen cuando usted está en reposo sobre la Tierra. Suponga que está en un automóvil
cae en línea recta hacia abajo (y el que viaja rápidamente con velocidad constante. Si deja caer una moneda desde arriba
árbol se mueve hacia la izquierda). de su cabeza adentro del automóvil, ¿cómo caerá? Caerá en línea recta hacia abajo con
b) En un marco de referencia fijo
respecto al automóvil, y golpeará el suelo directamente abajo del punto de liberación,
sobre la Tierra, la moneda tiene una
figura 36-2a. Esto es tal como los objetos caen en la Tierra —en línea recta hacia aba-
velocidad inicial (5 a la del auto) y
sigue una trayectoria curva †
En una plataforma giratoria (por ejemplo, un tiovivo), un objeto en reposo comienza a moverse
(parabólica). hacia fuera aun cuando ningún cuerpo ejerza fuerza sobre él. Por lo tanto, éste no es un marco
inercial. Véase la sección 11-8.

a) b)
Marco de referencia 5 automóvil Marco de referencia 5 Tierra

952 CAPÍTULO 36 La teoría especial de la relatividad


jo— y por lo tanto el experimento en el auto en movimiento está en concordancia con
el principio de relatividad. (Si deja caer la moneda por la ventanilla del automóvil, es-
to no ocurriría porque el aire en movimiento arrastraría la moneda hacia atrás en rela-
ción con el automóvil).
Sin embargo, note en este ejemplo que, para un observador en la Tierra, la mone- C U I D A D O
da sigue una trayectoria curva, figura 36-2b. La trayectoria real que sigue la moneda es
Las leyes son las mismas, pero las
diferente según se vea desde distintos marcos de referencia. Esto no viola el principio trayectorias pueden ser diferentes
de relatividad porque este principio afirma que las leyes de la física son las mismas en en distintos marcos de referencia.
todos los marcos inerciales. La misma ley de gravedad y las mismas leyes de movimien-
to se aplican en ambos marcos de referencia. La aceleración de la moneda es la misma
en ambos marcos de referencia. La diferencia en las figuras 36-2a y b es que, en el mar-
co de referencia de la Tierra, la moneda tiene una velocidad inicial (igual a la del auto-
móvil). Por lo tanto, las leyes de la física predicen que seguirá una trayectoria
parabólica como cualquier proyectil (capítulo 3). En el marco de referencia del auto-
móvil, no hay velocidad inicial, y las leyes de la física predicen que la moneda caerá en
línea recta hacia abajo. Las leyes son las mismas en ambos marcos de referencia, aun-
que las trayectorias específicas son diferentes.
La relatividad galileana-newtoniana implica ciertas suposiciones no verificables
que tienen sentido a partir de la experiencia cotidiana. Se supone que las longitudes de
los objetos son las mismas en un marco de referencia y en otro, y que el tiempo trans-
curre de igual forma en diferentes marcos de referencia. Entonces, en la mecánica clá-
sica, los intervalos de espacio y tiempo se consideran absolutos: su medición no cambia
de un marco de referencia a otro. La masa de un objeto, así como todas las fuerzas, se
suponen invariables por un cambio en un marco de referencia inercial.
Sin embargo, la posición de un objeto es diferente cuando se especifica en distintos
marcos de referencia, y lo mismo sucede con su velocidad. Por ejemplo, una persona C U I D A D O
puede caminar en el interior de un autobús hacia el frente con una rapidez de 2 m/s. La posición y la velocidad son
Pero si el autobús se desplaza a 10 m/s con respecto a la Tierra, entonces la persona se diferentes en distintos marcos de
mueve con una rapidez de 12 m/s con respecto a la Tierra. No obstante, la aceleración referencia, pero la longitud es la
de un objeto es la misma en cualquier marco de referencia inercial, de acuerdo con la misma (clásica)
mecánica clásica. Esto es así porque el cambio en velocidad y el intervalo de tiempo se-
rán los mismos. Por ejemplo, la persona en el autobús puede acelerar de 0 a 2 m/s en
1.0 segundo, de manera que a 5 2 m/s2 en el marco de referencia del autobús. Con
respecto a la Tierra, la aceleración es (12 m兾s - 10 m兾s)兾(1.0 s) = 2 m兾s2, que es la
misma.
Puesto que ni F ni m ni a cambian de un marco inercial a otro, entonces la segun-
da ley de Newton, F 5 ma, no cambia. En consecuencia, la segunda ley de Newton sa-
tisface el principio de relatividad. Se puede demostrar fácilmente que las otras leyes de
la mecánica también satisfacen el principio de relatividad.
El hecho de que las leyes de la mecánica sean las mismas en todos los marcos de
referencia inerciales implica que ningún marco inercial es especial en sentido alguno.
Esta importante conclusión se expresa al decir que todos los marcos de referencia iner-
ciales son equivalentes para la descripción de fenómenos mecánicos. Ningún marco de
referencia inercial es mejor que otro. Un marco de referencia fijo en un automóvil o un
avión que viajen con velocidad constante es tan bueno como uno fijo en Tierra. Cuan-
do alguien viaja suavemente con velocidad constante en un automóvil o avión, es tan
válido decir que la persona está en reposo y que la Tierra se mueve como decir lo con-
trario.† No hay un experimento que se pueda realizar para decir cuál marco “realmen-
te” está en reposo y cuál en movimiento. Por consiguiente, no hay forma de
caracterizar un marco de referencia particular como en reposo absoluto.
Sin embargo, en la segunda mitad del siglo XIX surgió una complicación. La exten-
sa teoría del electromagnetismo de Maxwell (capítulo 31) predijo con éxito que la luz
era una onda electromagnética. Las ecuaciones de Maxwell establecieron la velocidad
de la luz c como 3.00 3 108 mys; y esto es justo lo que se midió. Entonces surgió la pre-
gunta: ¿en qué marco de referencia la luz tiene precisamente el valor predicho por la
teoría de Maxwell? Se supuso que la luz tendría una rapidez diferente en distintos
marcos de referencia. Por ejemplo, si algunos observadores viajaran en una nave espa-
cial con una rapidez de 1.0 3 108 mys alejándose de una fuente de luz, uno esperaría
que ellos midieran la rapidez de la luz que los alcanza en (3.0 3 108 mys) 2 (1.0 3 108
mys) 5 2.0 3 108 mys. Pero las ecuaciones de Maxwell no prevén la velocidad relativa.
Las ecuaciones predicen que la rapidez de la luz es c 5 3.0 3 108 mys, lo que parece
implicar que debe haber algún marco de referencia especial donde c tendría este valor.


Ignore la rotación y curvatura de la Tierra. SECCIÓN 36–1 953
En los capítulos 15 y 16 se estudió que las ondas pueden viajar en agua y a lo lar-
go de sogas o cuerdas, y que las ondas sonoras viajan en el aire y otros materiales. Los
físicos del siglo XIX visualizaron el mundo material en términos de las leyes de la mecá-
nica, de manera que para ellos era natural suponer que la luz también debía viajar en
algún medio. A este medio transparente lo llamaron éter y supusieron que permeaba
todo el espacio.† En consecuencia, se supuso que la velocidad de la luz dada por las
ecuaciones de Maxwell debía darse con respecto al éter.
Al principio parecía que las ecuaciones de Maxwell no satisfacían el principio de re-
latividad. Eran más simples en el marco donde c 5 3.00 3 108 mys; esto es, en un marco
de referencia en reposo en el éter. En cualquier otro marco de referencia tendrían que
agregarse términos para explicar la velocidad relativa. Por ende, aunque la mayoría
de las leyes de la física obedecían el principio de relatividad, las leyes de la electricidad
y el magnetismo aparentemente no lo hacían. El segundo postulado de Einstein (sec-
ción 36-3) resolvió este problema: las ecuaciones de Maxwell sí satisfacen la relatividad.
Los científicos pronto se dieron a la tarea de determinar la rapidez de la Tierra en
relación con este marco absoluto, cualquiera que pudiera ser. Se diseñaron varios expe-
rimentos ingeniosos. El más directo lo realizaron A. A. Michelson y E. W. Morley en la
década de 1880. Ellos midieron la diferencia en la rapidez de la luz en distintas direc-
ciones con el interferómetro de Michelson (sección 34-6). Esperaban encontrar una di-
ferencia que dependiera de la orientación de sus aparatos con respecto al éter. Así, tal
como un bote tiene diferentes rapideces relativas con respecto a la Tierra cuando se
mueve corriente arriba, corriente abajo o a través de la corriente, del mismo modo
se esperaría que la luz tuviera diferentes rapideces dependiendo de la velocidad del
éter que pase por la Tierra.
Por extraño que parezca, no detectaron diferencia en absoluto. Éste fue un gran
enigma. Durante varios años se esgrimieron varias explicaciones, pero conducían a con-
tradicciones o, por alguna razón, no tenían aceptación generalizada. Este resultado nu-
lo fue uno de los grandes enigmas a finales del siglo XIX.
Entonces, en 1905, Albert Einstein propuso una nueva teoría radical que reconci-
liaba estos múltiples problemas en una forma sencilla. Pero, al mismo tiempo, como se
verá, cambió por completo las ideas del espacio y el tiempo.

* 36–2 El experimento de Michelson


y Morley
El experimento de Michelson y Morley se diseñó para medir la rapidez del éter —el
medio donde se suponía que viajaba la luz— con respecto a la Tierra. De esta forma,
los experimentadores esperaban encontrar un marco de referencia absoluto, uno que
pudiera considerarse en reposo.
Una de las posibilidades que consideraron los científicos del siglo XIX fue que el
éter estaba fijo en relación con el Sol, ya que incluso Newton consideró al Sol como
el centro del Universo. Si éste fuera el caso (desde luego, no había garantía), la rapidez
de la Tierra de aproximadamente 3 3 104 mys en su órbita alrededor del Sol produciría
un cambio de 1 parte en 104 en la rapidez de la luz (3.0 3 108 mys). La medición directa
de la rapidez de la luz con esta precisión no era posible. Pero más tarde, A. A. Michel-
son, con la ayuda de E. W. Morley, pudo usar su interferómetro (sección 34-6) para medir
la diferencia en la rapidez de la luz en diferentes direcciones con esta precisión.
Este famoso experimento se basó en el principio que se muestra en la figura 36-3.
El inciso a) es un diagrama del interferómetro de Michelson, y se supuso que el “viento
de éter” se desplazaba con rapidez v hacia la derecha. (De manera alternativa, se supu-
so que la Tierra se movía hacia la izquierda con respecto al éter, con rapidez v). La luz
proveniente de la fuente se divide en dos haces mediante un espejo semi-plateado MS.
Un haz viaja hacia el espejo M1 y el otro hacia el espejo M2. Los haces se reflejan en
M1 y M2 y se unen de nuevo después de pasar a través de MS. Los haces ahora super-
puestos interfieren entre sí y el ojo del observador ve la resultante como un patrón de
interferencia (que se estudió en la sección 34-6).
Si ocurre interferencia constructiva o destructiva en el centro del patrón de inter-
ferencia depende de las fases relativas de los dos haces después de que recorren sus
trayectorias separadas. Consideremos una analogía de un bote que viaja de arriba aba-
jo y a través de un río cuya corriente se desplaza con rapidez v, como se ilustra en la fi-

El medio para las ondas de luz no podía ser el aire, pues la luz viaja desde el Sol hasta la Tierra a tra-
vés de espacio casi vacío. Por lo tanto, se postuló otro medio: el éter. El éter no sólo era transparente si-
954 CAPÍTULO 36 no que, debido a la dificultad para detectarlo, se supuso que tenía densidad cero.
M1

B
Viento
v
de éter

Haz 1
l1
Espejo
semi-plateado 1
MS
Fuente Haz 2
M2 v (de corriente)

l2 1
2 Rapidez = 2
Rapidez =
c+v c -v
Rápido Lento
A (corriente abajo) (corriente arriba)

b)
a)
v

FIGURA 36–3 El experimento de Michelson y Morley. a) Interferómetro de


Michelson. b) Analogía del bote: el bote 1 pasa a través de la corriente y de
regreso; el bote 2 viaja corriente abajo y regresa corriente arriba (el bote tiene c v¢= c2 - v 2
rapidez c en relación con el agua). c) Cálculo de la velocidad del bote (o el haz
de luz) que viaja perpendicular a la corriente (o viento de éter).
c)

gura 36-3b. En agua tranquila, el bote puede viajar con rapidez c (no la rapidez de la
luz, en este caso).
Considere primero el haz 2 en la figura 36-3a, que viaja paralelo al “viento de
éter”. En su trayecto de MS a M2, la luz viajaría con rapidez c 1 v, de acuerdo con la fí-
sica clásica; tal como en el caso de un bote que viaja corriente abajo (véase la figura
36-3b), se agrega la rapidez del agua del río a la rapidez del bote (en relación con el
agua) para obtener la rapidez del bote en relación con la orilla. Como el haz recorre
una distancia l2, el tiempo que tarda en ir de MS a M2 sería t = l2兾(c + v). Para ha-
cer el viaje de regreso de M2 a MS, la luz se desplaza contra el viento de éter (como el
bote que va corriente arriba), de manera que se espera que su rapidez relativa sea c 2 v.
El tiempo para el viaje de regreso sería l2兾(c - v). El tiempo total para que el haz 2
vaya de MS a M2 y de regreso a MS es
l2 l2 2l2
t2 = + = .
c + v c - v c A1 - v2兾c2 B
Ahora considere el haz 1, que viaja a través del viento de éter. Aquí la analogía del
bote (figura 36-3b) es especialmente útil. El bote va del embarcadero A al embarcade-
ro B directamente a través de la corriente. Si va directamente, el flujo de la corriente lo
arrastrará corriente abajo. Para llegar al embarcadero B, el bote debe dirigirse en un
ángulo corriente arriba. El ángulo preciso depende de las magnitudes de c y v, pero no
es de interés para este análisis por sí mismo. El inciso c) de la figura 36-3 indica cómo
calcular la velocidad v9 del bote en relación con la Tierra conforme cruza la corriente.
Como c, v y v9 forman un triángulo recto, se tiene que v¿ = 2c2 - v2 . El bote tiene
la misma rapidez cuando regresa. Si ahora se aplican estos principios al haz de luz 1 en la
figura 36-3a, se espera que el haz viaje con rapidez 2c2 - v2 para ir de MS a M1 y de
regreso. La distancia total recorrida es 2l1, así que el tiempo requerido para que el haz 1
haga el viaje redondo sería 2l1兾 2c2 - v2 , o

2l1
t1 = .
c 31 - v2兾c2

Note que el denominador en esta ecuación para t1 implica una raíz cuadrada, mientras
que para t2 no la implica.

*SECCIÓN 36–2 955


Si l1 5 l2 5 l, se ve que el haz 2 se retrasará con respecto al haz 1 por una cantidad
2l 1 1
¢ t = t2 - t1 = ¢ - ≤.
c 1 - v2兾c2
31 - v 兾c
2 2

Si v 5 0, entonces Dt 5 0, y los dos haces regresarán en fase, pues inicialmente estaban


en fase. Pero si v Z 0, entonces Dt Z 0 y los dos haces regresarán fuera de fase. Si se pu-
diera medir este cambio de fase de la condición v 5 0 a la de v Z 0, entonces se podría
determinar v. Pero la Tierra no se puede detener. Más aún, no debemos apresurarnos
para suponer que las longitudes no resultan afectadas por el movimiento y, por consi-
guiente, que l1 5 l2.
Michelson y Morley se dieron cuenta de que podían detectar la diferencia de fase
(suponiendo que v Z 0) si giraban su aparato 90º, porque entonces el patrón de interferen-
cia entre los dos haces debía cambiar. En la posición girada, el haz 1 ahora se movería
paralelo al éter y el haz 2 perpendicular al éter. Por lo tanto, los roles podrían invertir-
se, y en la posición girada los tiempos (designados mediante primas) serían
2l1 2l2
t1œ = y t2œ = .
c A1 - v 兾c B
2 2
c 31 - v2兾c2
El retraso de tiempo entre los dos haces en la posición no girada (sin prima) sería
2l2 2l1 .
¢ t = t2 - t1 = -
c A1 - v 兾c B
2 2
c 31 - v2兾c2
En la posición girada, la diferencia de tiempo sería
2l2 2l1
¢ t ¿ = t2œ - t1œ = - .
c 31 - v 兾c c A1 - v2兾c2 B
2 2

Cuando se realiza la rotación, las franjas del patrón de interferencia (sección 34-6) se
correrían una cantidad determinada por la diferencia:
2 1 1
¢t - ¢t¿ = Al + l2 B ¢ - ≤.
c 1 1 - v2兾c2 31 - v 兾c
2 2

Esta expresión se simplifica considerablemente si se supone que vyc V 1. En este caso


se puede usar el desarrollo binomial (Apéndice A), de manera que
1 v2 1 1 v2 .
L 1 + 2 y L 1 +
1 - v 兾c
2 2
c 31 - v 兾c
2 2 2 c2
Así,
2 v2 1 v2
¢t - ¢t¿ L Al1 + l2 B ¢ 1 + 2 - 1 - ≤
c c 2 c2
v2 .
L Al1 + l2 B
c3
4
Ahora suponga que v 5 3.0 3 10 mys, la rapidez de la Tierra en su órbita alrededor
del Sol. En los experimentos de Michelson y Morley, los brazos l1 y l2 fueron tuvieron
aproximadamente 11 m de largo. La diferencia de tiempo sería entonces alrededor de

(22 m)A3.0 * 104 m兾sB 2


L 7.3 * 10–16 s.
A3.0 * 108 m兾sB 3
Para luz visible con longitud de onda l 5 5.5 3 1027 m, la frecuencia sería
f 5 cyl 5 (3.0 3 108 mys)y5.5 3 1027 m) 5 5.5 3 1014 Hz, lo cual significa que las
crestas de onda pasan por un punto cada 1y(5.5 3 1014 Hz) 5 1.8 3 10215 s. Por lo tan-
to, con una diferencia de tiempo de 7.3 3 10216 s, Michelson y Morley deberían notar
un movimiento en el patrón de interferencia de (7.3 3 10216 s)y(1.8 3 10215 s) 5
0.4 franjas. Ellos podían detectar fácilmente esto, pues su aparato era capaz de obser-
var un corrimiento de franjas tan pequeño como de 0.01 franjas.
Pero, ¡no encontraron corrimiento significativo de franjas! Colocaron su aparato en
varias orientaciones. Realizaron observaciones día y noche, de manera que tuvieran va-
rias orientaciones con respecto al Sol (por la rotación de la Tierra).

956 CAPÍTULO 36
Intentaron en diferentes estaciones del año (la Tierra en diferentes posiciones por
su órbita alrededor del Sol). Nunca observaron un corrimiento significativo de franjas.
Este resultado nulo fue uno de los grandes enigmas de la física a finales del siglo
XIX. Explicarlo era un reto difícil. Una posibilidad para explicar el resultado nulo la
plantearon de manera independiente G. F. Fitzgerald y H. A. Lorentz (en la década de
1890), y propusieron que cualquier longitud (incluido el brazo de un interferómetro) se
contrae en un factor de 21 - v2兾c2 en la dirección de movimiento a través del éter.
De acuerdo con Lorentz, esto se podría deber al éter que afecta las fuerzas entre las
moléculas de una sustancia, que se suponía tenían naturaleza eléctrica. Con el tiempo,
esta teoría fue sustituida por la teoría más extensa propuesta por Albert Einstein en
1905: la teoría especial de la relatividad.

36–3 Postulados de la teoría especial


de la relatividad
Los problemas que existían a comienzos del siglo XX con respecto a la teoría electro-
magnética y la mecánica newtoniana fueron hermosamente resueltos por la introduc-
ción de Einstein de la teoría de la relatividad en 1905. Desconocedor del resultado
nulo de Michelson y Morley, Einstein fue motivado por ciertas preguntas concernientes
a la teoría electromagnética y las ondas luminosas. Por ejemplo, se preguntaba: “¿Qué
vería si viajo en un haz de luz?”. La respuesta fue que, en vez de una onda electromag-
nética viajera, vería campos eléctricos y magnéticos alternos en reposo cuya magnitud
cambiaría en el espacio, pero no cambiaría en el tiempo. Tales campos —reconoció—
nunca fueron detectados y de hecho no eran congruentes con la teoría electromagnéti-
ca de Maxwell. Por lo tanto, argumentó, no era razonable pensar que la rapidez de la
luz relativa a algún observador se podría reducir a cero, o de hecho reducirse en absolu-
to. Esta idea se convirtió en el segundo postulado de su teoría de la relatividad.
En su famoso ensayo de 1905, Einstein propuso deshacerse por completo de la
idea del éter y la suposición acompañante de un marco de referencia en reposo prefe-
rido o absoluto. Esta propuesta tomó cuerpo en dos postulados. El primero fue una ex-
tensión del principio de relatividad galileano-newtoniano para incluir no sólo las leyes
de la mecánica, sino también las del resto de la física, incluidas las de la electricidad y
el magnetismo:
Primer postulado (el principio de relatividad): Las leyes de la física tienen la mis-
ma forma en todos los marcos de referencia inerciales.
El primer postulado también se puede enunciar como: No hay experimento alguno que
se pueda realizar en un marco de referencia inercial para decir si se está en reposo o en
movimiento uniforme con velocidad constante.
El segundo postulado es congruente con el primero:
Segundo postulado (constancia de la rapidez de la luz): La luz se propaga a tra-
vés del espacio vacío con una rapidez definida c independiente de la rapidez de la
fuente o el observador.
Estos dos postulados constituyen los fundamentos de la teoría especial de la relatividad
de Einstein. Se le llama “especial” para distinguirla de la posterior “teoría general de la
relatividad”, que trata con marcos de referencia no inerciales (en aceleración; capítulo
44). La teoría especial, que es la que estudia aquí, trata sólo con marcos inerciales
El segundo postulado parece difícil de aceptar, pues parece violar el sentido común.
Antes que todo, se tiene que pensar en luz que viaja a través del espacio vacío. Sin em-
bargo, abandonar la idea del éter no era tan difícil, pues nunca se le detectó. Pero el
segundo postulado también dice que la rapidez de la luz en el vacío siempre es la mis-
ma, 3.00 3 108 mys, sin importar cuál sea la rapidez del observador o de la fuente. En
consecuencia, una persona que viaje hacia o desde una fuente de luz medirá la misma
rapidez para esa luz que alguien en reposo con respecto a la fuente. Esto entra en con-
flicto con la experiencia cotidiana: uno esperaría tener que sumar la velocidad del obser-
vador. Por otra parte, quizás uno no espera que la experiencia cotidiana sea de utilidad
cuando se lidia con la alta velocidad de la luz. Más aún, el resultado nulo del experimen-
to de Michelson y Morley es completamente congruente con el segundo postulado.†


El experimento de Michelson y Morley también se puede considerar como evidencia del primer pos-
tulado, pues tenía la intención de medir el movimiento relativo de la Tierra en un marco de referencia
absoluto. Su fracaso para hacerlo implica la ausencia de cualquiera de tales marcos preferidos. SECCIÓN 36–3 957
La propuesta de Einstein tiene cierta belleza. Al desechar la idea de un marco de
referencia absoluto, fue posible reconciliar la mecánica clásica con la teoría electro-
magnética de Maxwell. La rapidez de la luz predicha por las ecuaciones de Maxwell es
la rapidez de la luz en el vacío en cualquier marco de referencia.
La teoría de Einstein requiere que uno deseche nociones de sentido común acerca
del espacio y el tiempo, y en las siguientes secciones se examinarán algunas extrañas,
pero interesantes, consecuencias de la relatividad especial. Los argumentos para la ma-
yor parte serán sencillos. Se usará una técnica que el mismo Einstein utilizó: imaginar
situaciones experimentales muy sencillas en las que se necesiten pocas matemáticas.
De esta forma, se pueden ver muchas de las consecuencias de la teoría de la relatividad
sin necesidad de hacer cálculos detallados. Einstein llamó a estos experimentos “men-
tales”.

36–4 Simultaneidad
Una consecuencia importante de la teoría de la relatividad es que uno ya no puede
considerar al tiempo como una cantidad absoluta. Nadie duda que el tiempo fluye ha-
cia delante y nunca regresa. Pero el intervalo de tiempo entre dos eventos, e incluso si
dos eventos son o no simultáneos, depende del marco de referencia del observador. Por
evento, que se usa mucho en el texto, se quiere dar a entender algo que ocurre en un
lugar y en un tiempo particulares.
Se dice que dos eventos ocurren simultáneamente si ocurren exactamente al mis-
mo tiempo. Pero, ¿cómo se sabe si dos eventos ocurren precisamente al mismo tiempo? Si
ocurren en el mismo punto en el espacio, como dos manzanas que caen sobre su cabe-
za al mismo tiempo, es fácil. Pero si los dos eventos ocurren en lugares muy separados,
es más difícil saber si son simultáneos, pues se debe considerar el tiempo que la luz
proveniente de ellos tarda en llegar a donde uno está. Puesto que la luz viaja con rapi-
dez finita, una persona que ve dos eventos debe calcular de nuevo para descubrir cuán-
do ocurrieron en realidad. Por ejemplo, si se observa que dos eventos ocurren al mismo
tiempo, pero en realidad uno tuvo lugar más lejos del observador que el otro, entonces
el más distante pudo ocurrir antes, y los dos eventos no fueron simultáneos.
Ahora imagine un sencillo experimento mental. Suponga que un observador, lla-
mado O, se ubica exactamente a la mitad entre los puntos A y B, donde ocurren dos
eventos, figura 36-4. Suponga que los dos eventos son relámpagos que caen en los pun-
tos A y B, como se muestra. Para eventos breves como los relámpagos, sólo cortos pul-
sos de luz (representados por curvas en la figura 36-4) viajarán hacia fuera desde A y
B, y llegarán a O. El observador O “ve” los eventos cuando los pulsos de luz llegan al
punto O. Si los dos pulsos llegan a O al mismo tiempo, entonces los dos eventos tuvie-
ron que ser simultáneos. Esto se debe a que los dos pulsos de luz viajaron con la mis-
ma rapidez (postulado 2) y, puesto que la distancia OA es igual a OB, el tiempo para
que la luz viaje de A a O y de B a O debe ser el mismo. Entonces el observador O pue-
de afirmar definitivamente que los dos eventos ocurrieron de manera simultánea. Por
otra parte, si O ve la luz de un evento antes que la del otro, entonces el primer evento
ocurrió primero.

FIGURA 36–4 Un momento después de que el O


relámpago llega a los puntos A y B, los pulsos de
luz (que se muestran como ondas) viajan hacia el A B
observador O, pero O “ve” el relámpago sólo
cuando la luz llega a O.
Luz proveniente
de los dos eventos
en A y B

958 CAPÍTULO 36 La teoría especial de la relatividad


La pregunta que realmente se quiere examinar es ésta: si dos eventos son simultá-
O1 vB
neos para un observador en un marco de referencia, ¿también son simultáneos para
otro observador que se mueve con respecto al primero? Llamemos a los observadores
O1 y O2 y supongamos que están fijos en los marcos de referencia 1 y 2 que se mueven O2
con rapidez relativa v uno con respecto al otro. Estos dos marcos de referencia se pue-
a)
den considerar como dos cohetes o dos trenes (figura 36-5). O2 dice que O1 se mueve
hacia la derecha con rapidez v, como en la figura 36-5a; y O1 dice que O2 se mueve ha-
cia la izquierda con rapidez v, como en la figura 36-5b. Ambos puntos de vista son legí- O1
timos de acuerdo con el principio de la relatividad. [No hay un tercer punto de vista
que diga cuál se mueve “en realidad”]. vB O2
Ahora suponga que los observadores O1 y O2 observan y miden dos impactos de
relámpagos. Los relámpagos marcan ambos trenes en los puntos adonde llegan: en A1 y b)
B1 en el tren de O1, y en A2 y B2 en el tren de O2, figura 36-6a. En aras de la sencillez, FIGURA 36–5 Los observadores O1
suponga que O1 está exactamente a medio camino entre A1 y B1, y que O2 está a me- y O2, en dos trenes diferentes (dos
dio camino entre A2 y B2. Póngase primero en el marco de referencia de O2, de manera marcos de referencia diferentes), se
que observe que O1 se mueve a la derecha con rapidez v. Suponga también que los dos mueven con rapidez relativa v. O2 dice
eventos ocurren simultáneamente en el marco de O2, y justo en el instante cuando O1 y que O1 se mueve hacia la derecha
O2 están opuestos uno con respecto al otro, figura 36-6a. Poco tiempo después, figura a); O1 dice que O2 se mueve hacia la
36-6b, la luz proveniente de A2 y de B2 llega a O2 al mismo tiempo (esto se supuso). izquierda b). Ambos puntos de vista
Puesto que O2 sabe (o mide) que las distancias O2A2 y O2B2 son iguales, O2 sabe que son legítimos: todo depende del marco
los eventos son simultáneos en el marco de referencia O2. de referencia.

FIGURA 36–6 Experimento mental acerca de la simultaneidad. Tanto


A1 B1 vB en a) como en b) se está en el marco de referencia del observador O2,
O1
quien ve que el marco de referencia O1 se mueve hacia la derecha.
A2 B2 En a), un relámpago llega a los dos marcos de referencia en A1 y A2, y un
O2 segundo relámpago llega a B1 y B2. b) Un momento después, la luz
(que se muestra en gris) proveniente de los dos eventos llega a O2 al
a) mismo tiempo. De manera que, de acuerdo con el observador O2, los dos
relámpagos llegan simultáneamente. Pero en el marco de referencia de O1,
A1 O1 B1 vB la luz proveniente de B1 ya llegó a O1, mientras que la luz proveniente de
A1 todavía no llega a O1. Por lo tanto, en el marco de referencia de O1, el
A2 B2 evento en B1 debe preceder al evento en A1. La simultaneidad en el
O2 tiempo no es absoluta.
b)
Pero, ¿qué observa y mide el observador O1? Desde nuestro marco de referencia
(O2), se puede predecir lo que observará O1. Se ve que O1 se mueve hacia la derecha
durante el tiempo que la luz viaja hacia O1 desde A1 y B1. Como se ilustra en la figura
36-6b, desde el marco de referencia O2 se puede ver que la luz proveniente de B1 ya
pasó a O1, mientras que la luz proveniente de A1 todavía no llega a O1. Esto es, O1 ob-
serva la luz proveniente de B1 antes de observar la luz que viene de A1. Puesto que 1.
la luz viaja con la misma rapidez c en cualquier dirección y en cualquier marco de refe-
rencia, y 2. la distancia O1A1 es igual a O1B1, entonces el observador O1 sólo puede
concluir que el evento en B1 ocurrió antes que el evento en A1. Los dos eventos no son
simultáneos para O1, aun cuando lo sean para O2.
Por ende, se encuentra que dos eventos que tienen lugar en diferentes ubicaciones
y son simultáneos para un observador, en realidad no son simultáneos para un segundo
observador que se mueve en relación con el primero.
Puede ser tentador preguntar: “¿Cuál observador tiene razón, O1 u O2?”. La res-
puesta, de acuerdo con la relatividad, es que ambos tienen razón. No existe un “mejor”
marco de referencia que se pueda elegir para determinar cuál observador está en lo co-
rrecto. Ambos marcos son igualmente buenos. Sólo se puede concluir que la simultanei-
dad no es un concepto absoluto, sino relativo. Uno no está consciente de esta falta de
concordancia en la simultaneidad en la vida diaria, porque el efecto sólo es apreciable
cuando la rapidez relativa de los dos marcos de referencia es muy grande (cerca de c),
o las distancias implicadas son muy grandes.
EJERCICIO A Examine el experimento de la figura 36-6 desde el marco de referencia de
O1. En este caso, O1 estará en reposo y verá que el evento B1 ocurre antes que A1. ¿O1 re-
conocerá que O2, quien se mueve con rapidez v hacia la izquierda, verá los dos eventos co-
mo simultáneos? (Sugerencia: Dibuje un diagrama equivalente a la figura 36-6).

SECCIÓN 36–4 Simultaneidad 959


Espejo

D
Fuente
de luz
a) Receptor Reloj

D2
l2
2 D
2 l
D

FIGURA 36–7 La dilatación del


tiempo se puede demostrar mediante un l l
experimento mental: el tiempo que
tarda la luz en recorrer una nave
espacial y regresar es mayor para el b) Tierra
observador en la Tierra b) que para
el observador en la nave espacial a).
36–5 Dilatación del tiempo
y la paradoja de los gemelos
El hecho de que dos eventos simultáneos para un observador puedan no ser simultá-
neos para un segundo observador sugiere que el tiempo en sí no es absoluto. ¿Será que
el tiempo transcurre de manera diferente en un marco de referencia que en otro? De
hecho, esto es justo lo que predice la teoría de la relatividad de Einstein, como de-
muestra el siguiente experimento mental.
La figura 36-7 ilustra una nave espacial que pasa por la Tierra a alta rapidez. El pun-
to de vista de un observador en la nave espacial se muestra en el inciso a), y el de un ob-
servador en la Tierra en el inciso b). Ambos observadores tienen relojes precisos. La
persona en la nave espacial (figura 36-7a) emite un destello de luz y mide el tiempo que
la luz tarda en viajar directamente a través de la nave espacial y regresar después de re-
flejarse en un espejo (para mayor claridad, los rayos se dibujan con un ligero ángulo). En
el marco de referencia de la nave espacial, la luz recorre una distancia 2D con rapidez c;
de manera que el tiempo requerido para ir de ida y vuelta, que se denotará como Dt0, es
¢ t0 = 2D兾c.
El observador en la Tierra, figura 36-7b, observa el mismo proceso. Pero, para este
observador, la nave espacial está en movimiento. De esta manera, la luz recorre la tra-
yectoria diagonal a través de la nave espacial, se refleja en el espejo y regresa al emisor.
Aunque la luz viaja con la misma rapidez para este observador (segundo postulado), re-
corre una mayor distancia. Por lo tanto, el tiempo requerido, según mide el observador
en la Tierra, será mayor que el medido por el observador en la nave espacial.
Determinemos el intervalo de tiempo Dt medido por el observador en la Tierra en-
tre la emisión y la recepción de la luz. En el tiempo Dt, la nave espacial recorre una
distancia 2l 5 v Dt, donde v es la rapidez de la nave espacial (figura 36-7b). La luz re-
corre una distancia total sobre su trayectoria diagonal (teorema de Pitágoras) de
2 2D 2 + l 2 , donde l 5 v Dty2. En consecuencia,
2 3D2 + l2 2 3D2 + v2(¢ t)2兾4
c = = .
¢t ¢t
Al elevar al cuadrado ambos lados,
4D2
c2 = + v 2,
(¢ t)2
y al despejar Dt, se encuentra
2D .
¢t =
c 31 - v2兾c2
Al combinar esta ecuación para Dt con la fórmula anterior, Dt0 5 2Dyc, se encuentra que:
¢ t0
DILATACIÓN DEL TIEMPO ¢t = . (36–1a)
31 - v 兾c
2 2

Puesto que 31 - v2兾c2 siempre es menor que 1, se ve que Dt . Dt0. Esto es, el inter-
960 CAPÍTULO 36 valo de tiempo entre los dos eventos (el envío de la luz y su recepción en la nave espa-
cial) es mayor para el observador en la Tierra que para el observador en la nave espacial.
Éste es un resultado general de la teoría de la relatividad, y se conoce como dilatación
del tiempo. Enunciado de manera sencilla, el efecto de dilatación del tiempo dice que
los relojes en movimiento relativo a un observador corren más lentamente (en
comparación con los relojes en reposo relativo a ese observador).
Sin embargo, no se debe pensar que los relojes de alguna forma tienen fallas. En reali-
dad, se mide que el tiempo pasa más lentamente en cualquier marco de referencia en
movimiento, comparado con el propio. Este resultado notable es un resultado inevita-
ble de los dos postulados de la teoría de la relatividad.
El factor 1兾31 - v2兾c2 ocurre con tanta frecuencia en relatividad que general-
mente se le da el símbolo abreviado g (letra griega “gamma”), y la ecuación 36-1a se
escribe como
¢ t = g ¢ t0 (36–1b)
donde
g = 1 . (36–2)
31 - v 兾c
2 2

Advierta que g nunca es menor que 1 y no tiene unidades. A rapideces normales, g 5 1


hasta algunos lugares decimales; en general, g $ 1.
El concepto de dilatación del tiempo puede ser difícil de aceptar, porque contradi-
ce la experiencia. A partir de las ecuaciones 36-1 se ve que el efecto de la dilatación del
tiempo de hecho es despreciable a menos que v esté razonablemente cerca de c. Si v es
mucho menor que c, entonces el término v2yc2 es mucho menor que 1 en el denomina-
dor de la ecuación 36-1a, y entonces Dt L Dt0 (véase el ejemplo 36-2). Las rapideces que
se experimentan en la vida cotidiana son mucho menores que c; son tan pequeñitas
que comúnmente no se nota la dilatación del tiempo. Los experimentos pusieron a
prueba el efecto de la dilatación del tiempo y confirmaron las predicciones de Einstein.
En 1971, por ejemplo, relojes atómicos extremadamente precisos volaron alrededor del
mundo en aviones a propulsión. La rapidez de los aviones (103 kmyh) era mucho menor
que c, de manera que los relojes tenían que ser precisos en el orden de los nanosegun-
dos (1029 s) para que se pudiera detectar cualquier dilatación del tiempo. Tuvieron esta
precisión y confirmaron las ecuaciones 36-1 dentro del margen de error experimental.
Sin embargo, la dilatación del tiempo se confirmó décadas antes, mediante observacio-
nes de “partículas elementales” que tienen masas diminutas (por lo general de 10230 a
10227 kg) y en consecuencia requieren poca energía para acelerarse a rapideces cerca-
nas a la de la luz, c. Muchas de estas partículas elementales no son estables y, con el
tiempo, decaen en partículas más ligeras. Un ejemplo es el muón, cuya vida media es de
2.2 ms cuando está en reposo. Cuidadosos experimentos demostraron que, cuando un
muón viaja con alta rapidez, su vida media es más larga que cuando está en reposo, tal
como predijo la fórmula de dilatación del tiempo.
EJEMPLO 36–1 Vida media de un muón en movimiento. a) ¿Cuál será la vida
media de un muón, medida en el laboratorio, si viaja a v 5 0.60c 5 1.80 3 108 mys
con respecto al laboratorio? Su vida media en reposo es 2.20 ms 5 2.20 3 1026 s.
b) ¿Cuánto recorre un muón en el laboratorio, en promedio, antes de decaer?
PLANTEAMIENTO Si un observador se moviera junto con el muón (el muón estaría
en reposo para este observador), el muón tendría una vida media de 2.2 3 1026 s. Para un
observador en el laboratorio, el muón vive más debido a la dilatación del tiempo. En-
cuentre la vida media con la ecuación 36-1a y la distancia media dada por d 5 v Dt.
SOLUCIÓN a) A partir de la ecuación 36-1a, con v 5 0.60c, se tiene
¢ t0 2.20 * 10–6 s 2.20 * 10–6 s = 2.8 * 10–6 s.
¢t = = =
31 - v 兾c 31 - 0.36c 兾c 20.64
2 2 2 2

b) La relatividad predice que un muón con una rapidez de 1.80 3 108 mys recorrería
una distancia media d = v ¢t = A1.80 * 108 m兾sB A2.8 * 10 –6 sB = 500 m, y ésta es
la distancia que se mide experimentalmente en el laboratorio.
NOTA A una rapidez de 1.8 3 108 mys, la física clásica diría que, con una vida media de
2.2 ms, un muón promedio recorrería d 5 vt 5 (1.8 3 108 mys)(2.2 3 1026 s) 5 400 m.
Esto es menos que la distancia medida.

EJERCICIO B ¿Cuál es la vida media del muón (ejemplo 36-1) si viaja a v 5 0.90c? a) 0.42
ms; b) 2.3 ms; c) 5.0 ms; d) 5.3 ms; e) 12.0 ms.

SECCIÓN 36–5 961


Es necesario clarificar cómo usar las ecuaciones 36-1 y el significado de Dt y Dt0.
La ecuación es cierta sólo cuando Dt0 representa el intervalo de tiempo entre los dos
C U I D A D O
eventos en un marco de referencia donde ambos ocurren en el mismo punto del espa-
El tiempo propio Dt0 es para
cio (como en la figura 36-7a, donde los dos eventos son el destello de luz que se envía
2 eventos en el mismo punto
en el espacio y el destello que se recibe). Este intervalo de tiempo, Dt0, se denomina tiempo propio.
Entonces Dt en las ecuaciones 36-1 representa el intervalo de tiempo entre los dos
eventos, medido en un marco de referencia que se mueve con rapidez v con respecto al
primero. En el ejemplo 36-1 anterior, Dt0 (y no Dt) se igualó a 2.2 3 1026 s porque sólo
es en el marco en reposo del muón donde ocurren los dos eventos (“nacimiento” y “de-
caimiento”) en el mismo punto en el espacio. El tiempo propio Dt0 es el tiempo más
corto entre los eventos que puede medir cualquier observador. En cualquier otro marco
de referencia en movimiento, el tiempo Dt es mayor.

EJEMPLO 36–2 Dilatación del tiempo a 100 km/h. Comprobemos la dilatación


del tiempo para rapideces cotidianas. Un automóvil que viaja a 100 kmyh recorre cier-
ta distancia en 10.00 s, de acuerdo con el reloj del conductor. ¿Cuál sería la medición
de un observador en reposo sobre la Tierra para el intervalo de tiempo?
PLANTEAMIENTO La rapidez del automóvil relativa a la Tierra es 100 kmyh 5 (1.00
3 105 m)y(3600 s) 5 27.8 mys. El conductor está en reposo en el marco de referencia
del automóvil, así que se sustituye Dt0 5 10.00 s en la fórmula de dilatación del tiempo.
SOLUCIÓN Se utiliza la ecuación 36-1a:
¢ t0 10.00 s 10.00 s
¢t = = = .
31 - A8.59 * 10 B
2 2 –15
v 27.8 m兾s
1 - 2 1 - ¢ ≤
C c C 3.00 * 108 m兾s
Si usted alimenta una calculadora con estos números, obtendrá Dt 5 10.00 s, pues el
denominador difiere de 1 por tan pequeña cantidad. De hecho, el tiempo medido por
un observador en la Tierra no mostraría diferencia del que mide el conductor, incluso
con los mejores instrumentos. Una computadora capaz de calcular hasta un gran nú-
R ESOLUCIÓN DE P ROB LEMAS mero de lugares decimales revelaría una diferencia entre Dt y Dt0. La diferencia se
Uso del desarrollo binomial puede estimar con el desarrollo binomial (Apéndice A).
(16x)n L 16nx. [para x V 1]
1
En la fórmula de dilatación del tiempo, se tiene el factor g = A1 - v2兾c2 B – 2 . Por lo tanto,
1
v2 – 2 1 v2
¢ t = g ¢ t0 = ¢ t0 ¢ 1 - ≤ L ¢ t0 ¢ 1 + ≤
c2 2 c2

L 10.00 s c1 + ≤ d L 10.00 s + 4 * 10–14 s.


2
1 27.8 m兾s
¢
2 3.00 * 10 m兾s
8

De manera que la diferencia entre Dt y Dt0 se predice como 4 3 10214 s, una cantidad
extremadamente pequeña.

EJERCICIO C Cierto reloj atómico mantiene tiempo perfecto sobre la Tierra. Si el reloj se
lleva en una nave espacial que viaja con una rapidez v 5 0.60c, ¿este reloj ahora corre más
lentamente de acuerdo con las personas en a) la nave espacial, b) la Tierra?

EJEMPLO 36–3 Lectura de una revista en una nave espacial. Un pasajero en


una nave espacial de gran rapidez, que viaja entre la Tierra y Júpiter con una rapidez
constante de 0.75c, lee una revista en 10.0 min de acuerdo con su reloj. a) ¿Cuánto
tarda en leerla, según se mide en relojes colocados en Tierra? b) ¿Cuánto más lejos
está la nave espacial de la Tierra al final de la lectura de la revista, de lo que estaba al
principio?
PLANTEAMIENTO a) El intervalo de tiempo en un marco de referencia se relaciona
con el intervalo de tiempo en el otro mediante la ecuación 36-1a o b. b) Con rapi-
dez constante, la distancia es rapidez 3 tiempo. Puesto que hay dos tiempos (un Dt y
un Dt0), se obtendrán dos distancias: una para cada marco de referencia. [Este resulta-
do sorprendente se explora en la siguiente sección (36-6)].

962 CAPÍTULO 36 La teoría especial de la relatividad


SOLUCIÓN a) El intervalo de tiempo dado de 10.0 min es el tiempo propio: el inicio
y el fin de la lectura de la revista ocurren en el mismo lugar en la nave espacial. Los
relojes en la Tierra miden
¢ t0 10.00 min
¢t = = = 15.1 min.
31 - (0.75)
2 2
v
1 - 2
C c
b) En el marco de la Tierra, el cohete recorre una distancia D = v ¢ t 5
(0.75c)(15.1 min) = (0.75)A3.0 * 108 m兾sB(15.1 min * 60 s兾min) = 2.04 * 1011 m.
En el marco de la nave espacial, la Tierra se aleja de la nave espacial a 0.75c, pero el
tiempo sólo es de 10.0 min, de manera que la distancia se mide en D0 5 v Dt0 5
(2.25 * 108 m兾s)(600s) = 1.35 * 1011 m.
En la tabla 36-1 se indican valores para g = 1兾31 - v2兾c2 para algunos valores TABLA 36–1 Valores de g
de rapidez v. v g
¿Viaje espacial? 0 1.000
La dilatación del tiempo suscitó interesantes especulaciones acerca de los viajes espa- 0.01c 1.000
ciales. De acuerdo con la física clásica (newtoniana), alcanzar una estrella a 100 años luz de
0.10c 1.005
distancia no sería posible para mortales ordinarios (1 año luz es la distancia que recorre
la luz en 1 año 5 3.0 * 108 m兾s * 3.16 * 107 s = 9.5 * 1015 m ). Incluso si una nave 0.50c 1.15
espacial pudiera viajar con una rapidez muy cercana a la de la luz, tardaría cerca de 100 0.90c 2.3
años en llegar a tal estrella. Pero la dilatación del tiempo dice que el tiempo implicado podría 0.99c 7.1
ser menor. En una nave espacial que viaje a v 5 0.999c, el tiempo para tal viaje sería de só-
lo aproximadamente ¢ t0 = ¢ t 31 - v2兾c2 = (100 años)31 - (0.999)2 = 4.5 años.
Por lo tanto, la dilatación del tiempo permite tal viaje, pero es probable que no se su-
peren los enormes problemas prácticos que implica lograr una rapidez así, al menos no
en el futuro cercano.
En este ejemplo, en la Tierra transcurrirían 100 años, mientras que para el astro-
nauta en el viaje sólo pasarían 4.5 años. ¿Sólo son los relojes lo que funcionaría más
lentamente para el astronauta? No. Todos los procesos, incluidos el envejecimiento y
otros procesos vitales, se realizarían más lentamente para el astronauta, de acuerdo con
el observador de la Tierra. Pero para el astronauta, el tiempo pasaría en una forma nor-
mal. El astronauta experimentaría 4.5 años de dormir, comer, leer, etcétera. Y las per-
sonas sobre la Tierra experimentarían 100 años de actividad ordinaria.
Paradoja de los gemelos
No mucho tiempo después de que Einstein propusiera la teoría especial de la relativi-
dad, se puntualizó una aparente paradoja. De acuerdo con esta paradoja de los geme-
los, suponga que uno de un par de gemelos de 20 años de edad despega en una nave
espacial que viaja con muy alta rapidez hacia una estrella distante y regresa, mientras
que el otro gemelo permanece en la Tierra. De acuerdo con el gemelo en la Tierra, el
gemelo astronauta envejecerá menos. Mientras que pueden pasar 20 años para el ge-
melo en la Tierra, tal vez sólo pasaría un año para el viajero (dependiendo de la rapi-
dez de la nave espacial). Por ende, cuando el viajero regresa, el gemelo que se quedó
en Tierra podría tener 40 años de edad, mientras que el gemelo viajero sólo tendría 21.
Éste es el punto de vista del gemelo en la Tierra. Pero, ¿qué hay del gemelo viajero?
Si todos los marcos de referencia inerciales son igualmente buenos, ¿el gemelo viajero
no haría las mismas afirmaciones que el gemelo en la Tierra, sólo que a la inversa? ¿El
gemelo astronauta no puede afirmar que, puesto que la Tierra se mueve alejándose a
alta rapidez, el tiempo pasa más lentamente en la Tierra y que el gemelo en la Tierra
envejecerá menos? Esto es lo opuesto de lo que predice el gemelo de la Tierra. No es
posible que ambos estén en lo correcto, porque, después de todo, la nave espacial re-
gresa a la Tierra y se puede realizar una comparación directa de edades y relojes.
Sin embargo, aquí no existe contradicción. De hecho, uno de los puntos de vista es
incorrecto. Las consecuencias de la teoría especial de la relatividad (en este caso, la dila-
tación del tiempo) se pueden aplicar sólo mediante observadores en un marco de refe-
rencia inercial. La Tierra es (o casi es) uno de tales marcos, mientras que la nave espacial
no lo es. La nave espacial acelera al comienzo y al final de su viaje, y cuando da vuelta en
el punto más lejano de su viaje. Durante la aceleración, el gemelo en la nave espacial no
está en un marco de referencia inercial. Mientras tanto, el gemelo astronauta puede estar
en un marco inercial (y se justifica al decir que el reloj del gemelo de la Tierra funciona
lentamente), pero no siempre es el mismo marco. Así que no puede usar la relatividad es-
pecial para predecir sus edades relativas cuando regresa a Tierra. El gemelo de la Tierra
permanece en el mismo marco inercial y, por ende, se puede confiar en sus predicciones
con base en la relatividad especial. En consecuencia, no hay paradoja. La predicción del
gemelo de la Tierra de que el gemelo viajero envejece menos es la adecuada. SECCIÓN 36–5 963
* Sistema de posicionamiento global (GPS)
F Í S I C A A P L I C A D A Los pilotos de aviones, automóviles y botes, así como los excursionistas usan receptores
Sistema de posicionamiento de sistema de posicionamiento global (GPS, por las siglas de global positioning system)
global (GPS) que les informan con bastante exactitud dónde están en un momento dado. Los 24 saté-
lites del sistema de posicionamiento global envían señales de tiempo precisas que usan
relojes atómicos. El receptor compara los tiempos recibidos desde al menos cuatro saté-
lites, cuyos tiempos se sincronizan cuidadosamente dentro del orden de 1 parte en 1013.
Al comparar las diferencias de tiempo con las posiciones conocidas de los satélites y la
rapidez fija de la luz, el receptor puede determinar a qué distancia está de cada satélite
y, por consiguiente, dónde está sobre la Tierra. Esto se logra hasta con una exactitud de
15 m, si se realizan correcciones como la siguiente debida a la relatividad especial.
EJEMPLO CONCEPTUAL 36–4 Una corrección relativista al GPS. Los satélites
GPS se desplazan aproximadamente a 4 kmys 5 4000 mys. Demuestre que un buen re-
ceptor GPS necesita corregir considerando la dilatación del tiempo si ha de producir re-
sultados congruentes con relojes atómicos precisos hasta 1 parte en 1013.
RESPUESTA Calcule la magnitud del efecto de dilatación del tiempo al insertar v 5
4000 mys en la ecuación 36-1a:
1
¢t = ¢ t0
v2
1 - 2
C c
1
= ¢ t0
4 * 103 m兾s 2
1 - ¢ ≤
C 3 * 108 m兾s
1
= ¢ t0 .
31 - 1.8 * 10
–10

Use el desarrollo binomial: (1 6 x)n L 1 6 nx para x V 1 (véase el Apéndice A) que


1
aquí es (1 - x) - 2 L 1 + 12 x. Esto es,

¢ t = A1 + 12 A1.8 * 10–10 BB ¢ t0 = A1 + 9 * 10–11 B ¢ t0 .


El “error” de tiempo dividido entre el intervalo de tiempo es
A¢ t - ¢ t0 B
= 1 + 9 * 10–11 - 1 = 9 * 10–11 L 1 * 10–10.
¢ t0
La dilatación del tiempo, si no se toma en cuenta, introduciría un error de aproxima-
damente 1 parte en 1010, que es 1000 veces mayor que la precisión de los relojes ató-
micos. No corregir considerando la dilatación del tiempo significa que un receptor
podría obtener una exactitud mucho menor.
NOTA Los dispositivos GPS también deben hacer otras correcciones, incluidas aqué-
llas referentes a los efectos asociados con la relatividad general.

36–6 Contracción de la longitud


No sólo los intervalos de tiempo son diferentes en distintos marcos de referencia. Los
intervalos de espacio —longitudes y distancias— también son diferentes de acuerdo
con la teoría especial de la relatividad, y esto se ilustra con un experimento mental.
Observadores en la Tierra miran una nave espacial que viaja con rapidez v desde la
Tierra hasta Neptuno, figura 36-8a. La distancia entre los planetas, medida por los obser-
vadores de la Tierra, es l0. El tiempo requerido para el viaje, medido desde la Tierra, es
l0
¢t = v . [observador en la Tierra]
En la figura 36-8b se ve el punto de vista de los observadores en la nave espacial. En
este marco de referencia, la nave espacial está en reposo; la Tierra y Neptuno se mue-
ven† con rapidez v. El tiempo entre la partida de la Tierra y la llegada a Neptuno (ob-
servada desde la nave espacial) es el “tiempo propio”, pues los dos eventos ocurren en
el mismo punto en el espacio (es decir, en la nave espacial). En consecuencia, el inter-

Se supone que v es mucho mayor que la rapidez relativa de Neptuno y la Tierra, así que esta última se
964 CAPÍTULO 36 puede ignorar.
v FIGURA 36–8 a) Una nave espacial
que viaja a muy alta rapidez desde la
Tierra Tierra hacia el planeta Neptuno, como
a) La Tierra en reposo Neptuno se ve desde el marco de referencia de
la Tierra. b) De acuerdo con un
observador en la nave espacial, la
Tierra y Neptuno se mueven a la muy
v v alta rapidez v: la Tierra deja la nave
espacial y un tiempo Dt0 más tarde
Tierra Neptuno llega a la nave espacial.
b) Nave espacial en reposo Neptuno
valo de tiempo es menor para los observadores en la nave especial que para los observa-
dores en la Tierra. Esto es, a causa de la dilatación del tiempo (ecuación 36-1a), el tiem-
po para el viaje, según se ve en la nave espacial, es
¢ t0 = ¢ t 31 - v2兾c2 = ¢t兾g. [observador en la nave espacial]
Puesto que los observadores en la nave espacial miden la misma rapidez pero menos
tiempo entre estos dos eventos, también miden la distancia como menor. Si se con-
sidera que l es la distancia entre los planetas, vista por los observadores de la nave es-
pacial, entonces l5 v Dt0, que se puede rescribir como l 5 v Dt0 5 v Dt 31 - v2兾c2 =
l031 - v2兾c2 . Por lo tanto, se tiene el importante resultado
CONTRACCIÓN DE LA
l = l031 - v2兾c2 (36–3a) LONGITUD

o, empleando g (ecuación 36-2),


l0
l = g. (36–3b)

Éste es un resultado general de la teoría especial de la relatividad y se aplica a longitu-


des de objetos así como a distancia entre objetos. El resultado se puede enunciar de
forma más sencilla con palabras:
La longitud de un objeto que se mueve en relación con el observador, según las
mediciones, es más corta a lo largo de su dirección de movimiento que cuando es-
tá en reposo. C U I D A D O
A esto se le conoce como contracción de la longitud. La longitud l0 en las ecuaciones La longitud propia se mide
36-3 se llama longitud propia. Es la longitud del objeto (o la distancia entre dos puntos en el marco de referencia
cuyas posiciones se miden al mismo tiempo) según determinan observadores en reposo donde las dos posiciones
con respecto al objeto. Las ecuaciones 36-3 dan la longitud l que medirán los observa- están en reposo
dores cuando el objeto pase junto a ellos con rapidez v.
Es importante notar que la contracción de la longitud ocurre sólo a lo largo de la
dirección de movimiento. Por ejemplo, la nave espacial en movimiento de la figura 36-8a
se acorta en longitud, pero su altura es la misma que cuando está en reposo. FIGURA 36–9 Ejemplo 36–5.
La contracción de la longitud, al igual que la dilatación del tiempo, no es aprecia-
ble en la vida cotidiana debido a que el factor 31 - v2兾c2 en la ecuación 36-3a difiere
significativamente de 1.00 sólo cuando v es muy grande.
EJEMPLO 36–5 Contracción de una pintura. Una pintura rectangular, que mi- 1.00 m
de 1.00 m de alto y 1.50 m de ancho, cuelga de la pared lateral de una nave espacial
que pasa junto a la Tierra con una rapidez de 0.90c. Véase la figura 36-9a. a) ¿Cuá-
les son las dimensiones de la pintura, de acuerdo con el capitán de la nave espacial? 1.50 m
b) ¿Cuáles son las dimensiones que ve un observador en la Tierra? a)
PLANTEAMIENTO Se aplica la fórmula de la contracción de la longitud, ecuación 36-3a,
a la dimensión paralela al movimiento; v es la rapidez de la pintura relativa al observador.
SOLUCIÓN a) La pintura está en reposo (v 5 0) en la nave espacial, de manera que
parece perfectamente normal a todos en la nave espacial (al igual que todo lo demás
en la nave espacial). El capitán ve una pintura de 1.00 m por 1.50 m. 1.00 m
b) Sólo se acorta la dimensión en la dirección del movimiento, así que la altura queda
inalterada en 1.00 m, figura 36-9b. Sin embargo, la longitud se contrae a
v2 b)
?
l = l0 1 - 2
C c
= (1.50 m)31 - (0.90)2 = 0.65 m.
De manera que la pintura tiene dimensiones de 1.00 m 3 0.65 m.
SECCIÓN 36–6 965
EJEMPLO 36–6 Un supertrén de fantasía. Un tren muy rápido, con una longi-
tud propia de 500 m, pasa a través de un túnel de 200 m de largo. Imagine que la ra-
pidez del tren es tan elevada que el tren cabe completamente dentro del túnel, según
ve un observador en reposo sobre la Tierra. Esto es, la máquina está a punto de salir
por un extremo del túnel cuando el último vagón desaparece por el otro extremo.
¿Cuál es la rapidez del tren?
PLANTEAMIENTO Puesto que el tren cabe justo en el interior del túnel, su longitud
medida por la persona en el suelo es de 200 m. Entonces se puede usar la fórmula de
contracción de la longitud, ecuación 36-3a o b, para despejar v.
SOLUCIÓN Al sustituir l 5 200 m y l0 5 500 m en la ecuación 36-3a se obtiene

v2
200 m = 500 m 1 - ;
C c2

Al dividir ambos lados entre 500 m y elevar al cuadrado se obtiene

v2
(0.40)2 = 1 -
c2

v
= 31 - (0.40)2
c

v = 0.92c.

NOTA Ningún tren real podría ir tan rápido. Pero es divertido pensar en ello.
NOTA Un observador en el tren no vería los dos extremos del tren dentro del túnel al
mismo tiempo. Recuerde que los observadores que se mueven unos en relación con
otros no concuerdan acerca de la simultaneidad.

EJERCICIO D ¿Cuál es la longitud del túnel, según miden los observadores a bordo del
tren en el ejemplo 36-6?

EJEMPLO CONCEPTUAL 36–7 Determinación de la longitud del tren y el túnel.


Observadores en reposo sobre la Tierra ven un tren muy rápido de 200 m de largo que pa-
sa a través de un túnel de 200 m de largo (como en el ejemplo 36-6), de manera que el tren
momentáneamente desaparece de la vista dentro del túnel. Observadores a bordo del
tren miden la longitud del tren en 500 m y la longitud del túnel sólo en 80 m (ejercicio D,
con la ecuación 36-3a). Como es obvio, un tren de 500 m de largo no puede caber dentro
de un túnel de 80 m de largo. ¿Cómo se explica esta aparente incongruencia?
RESPUESTA Los eventos simultáneos en un marco de referencia tal vez no sean si-
multáneos en otro. Sea el “evento A” la máquina que sale de un extremo del túnel, y
el “evento B” el último vagón que desaparece en el otro extremo. Para los observado-
res en el marco de la Tierra, los eventos A y B son simultáneos. Sin embargo, para los
observadores a bordo del tren, los eventos no son simultáneos. En el marco del tren,
el evento A ocurre antes que el evento B. Conforme la máquina sale del túnel, los ob-
servadores a bordo del tren observan al último vagón todavía a 500 m 2 80 m 5 420
m de la entrada al túnel.

966 CAPÍTULO 36
7 7

FIGURA 36–10 De acuerdo con un


reloj exacto sobre un tren que viaja
rápidamente, una persona a) comienza
a cenar a las 7:00 y b) termina a las
7:15. Al comienzo de la cena, dos
observadores en la Tierra ajustan sus
relojes para corresponder con el reloj
del tren. Estos observadores miden
que el tiempo de comida es de 20
minutos.
a) b)

36–7 Espacio-tiempo tetradimensional


Imagine que una persona se encuentra a bordo de un tren que viaja a una rapidez muy
alta, por ejemplo, 0.65c (figura 36-10). Esta persona comienza a cenar a las 7:00 y ter-
mina a las 7:15, de acuerdo con un reloj en el tren. Los dos eventos, comienzo y fin de
la cena, tienen lugar en el mismo punto a bordo del tren. Así que el tiempo propio en-
tre estos dos eventos es de 15 min. Para observadores en la Tierra, la comida tardará
más: 20 min, de acuerdo con las ecuaciones 36-1. Suponga que la comida se sirvió en un
plato de 20 cm de diámetro. Para observadores en la Tierra, el plato mide sólo 15 cm de
diámetro (contracción de la longitud). Por ende, para observadores en la Tierra, la ra-
ción de comida parece más pequeña, pero dura más tiempo.
En un sentido, los dos efectos (dilatación del tiempo y contracción de la longitud)
se equilibran mutuamente. Cuando se ven desde la Tierra, lo que un objeto parece perder
en tamaño lo gana en longitud de tiempo de duración. Espacio, o longitud, se intercam-
bian por tiempo.
Consideraciones como ésta conducen a la idea de un espacio-tiempo tetradimen-
sional: el espacio ocupa tres dimensiones y el tiempo es una cuarta dimensión. Espacio
y tiempo están íntimamente vinculados. Tal como cuando al apretar un globo, una di-
mensión de éste se hace más larga y otra más corta, de igual forma, cuando se examinan
objetos y eventos desde diferentes marcos de referencia, cierta cantidad de espacio se
intercambia por tiempo, o viceversa.
Aunque la idea de cuatro dimensiones parece extraña, se refiere a la idea de que
cualquier objeto o evento se especifica mediante cuatro cantidades: tres para describir
dónde en el espacio, y una para describir cuándo en el tiempo. El aspecto realmente
inusual del espacio-tiempo tetradimensional es que espacio y tiempo pueden entre-
mezclarse: un poco de uno puede intercambiarse por un poco del otro cuando cambia
el marco de referencia.
Para la mayoría de las personas es difícil entender la idea de espacio-tiempo tetradi-
mensional. De algún modo sienten, como los físicos lo hicieron antes de la llegada de la
relatividad, que espacio y tiempo son entidades completamente separadas. Sin embargo,
en los experimentos mentales se encontró que no están completamente separados. Y pien-
se acerca de Galileo y Newton. Antes de Galileo, la dirección vertical, aquélla en la que
caen los objetos, se consideraba distintivamente diferente de las dos dimensiones horizon-
tales. Galileo demostró que la dimensión vertical sólo difiere en que resulta ser la direc-
ción en la que actúa la gravedad. De otro modo, las tres dimensiones son equivalentes, un
punto de vista que todos aceptan en la actualidad. Ahora se pide aceptar una dimensión
más, el tiempo, que anteriormente se consideraba como algo diferente. Esto no equivale a
decir que no hay distinción entre espacio y tiempo. Lo que la relatividad demostró es que
las determinaciones de espacio y tiempo no son independientes una de otra.
En la relatividad galileana-newtoniana, el intervalo de tiempo entre dos eventos, Dt,
y la distancia entre dos eventos o puntos, Dx, son cantidades invariantes sin importar des-
de cuál marco de referencia se vean. Ninguna de estas cantidades es invariante de acuerdo
con la relatividad de Einstein. Pero hay una cantidad invariante en el espacio- tiempo
tetradimensional, llamada intervalo espacio-tiempo, que es (¢s)2 = (c ¢ t)2 - (¢x)2.
Este problema (indicado con el número 97 al final del capítulo) se deja para demostrar que
tal cantidad, de hecho, es invariante bajo una transformación de Lorentz (sección 36-8).

*SECCIÓN 36–7 967


36–8 Transformaciones galileanas
y de Lorentz
Ahora examinaremos con detalle las matemáticas de cantidades relacionadas en un marco
de referencia inercial, con las cantidades equivalentes en otro. En particular, se verá có-
mo las posiciones y velocidades se transforman (esto es, cambian) de un marco a otro.
Comenzaremos con el punto de vista clásico o galileano. Considere dos marcos de
referencia inerciales S y S9 que se caracterizan, cada uno, por un conjunto de ejes coorde-
nados, figura 36-11. Los ejes x y y se refieren a S, y x9 y y’ a S9 (no se muestra z). Los ejes
x9 y x se traslapan entre sí, y se supone que el marco S9 se mueve hacia la derecha en la
dirección x con rapidez constante v con respecto a S. Para simplicar la situación, suponga
que los orígenes 0 y 09 de los dos marcos de referencia se superponen en el tiempo t 5 0.

y y¢
S S¢
vB
FIGURA 36–11 El marco de
referencia inercial S9 se mueve hacia la
derecha con rapidez constante v con P
respecto al marco S. yt x¢
x

0 x 0¢ x¢

Ahora considere un evento que ocurre en algún punto P (figura 36-11) representa-
do por las coordenadas x9, y9, z9 en el marco de referencia S9 en el tiempo t9. ¿Cuáles
serán las coordenadas de P en S? Como S y S9 inicialmente se traslapan con precisión,
después de un tiempo t9, S9 se moverá una distancia vt9. Por lo tanto, en el tiempo t9,
x 5 x9 1 vt9. Por otra parte, las coordenadas y y z, no se alteran por el movimiento a lo
largo del eje x; por lo tanto, y 5 y9 y z 5 z9. Finalmente, puesto que el tiempo se supo-
ne absoluto en la física galileana-newtoniana, los relojes en los dos marcos concorda-
rán entre sí; de manera que t 5 t9. Esto se resume en las siguientes ecuaciones de
transformación galileana:
x = x¿ + vt ¿
y = y¿
z = z¿ [galileana] (36–4)
t = t ¿.
Estas ecuaciones dan las coordenadas de un evento en el marco S cuando se conocen
las coordenadas en el marco S9. Si se conocen las del marco S, entonces las coordena-
das S9 se obtienen a partir de
x¿ = x - vt, y¿ = y, z¿ = z, t ¿ = t. [galileana]
Estas cuatro ecuaciones son la transformación “inversa” y se obtienen muy fácilmente
de las ecuaciones 36-4. Advierta que el efecto es simplemente intercambiar cantidades
primas y no primas, y sustituir v por –v. Esto tiene sentido pues, desde el marco S9, S se
mueve hacia la izquierda (dirección x negativa) con rapidez v.
Ahora suponga que el punto P en la figura 36-11 representa una partícula que se
mueve. Sean uxœ , uyœ , uzœ las componentes de su vector velocidad en S9. (Se usa u para
distinguirla de la velocidad relativa de los dos marcos, v). Ahora uxœ = dx¿兾dt ¿,
uyœ = dy¿兾d t ¿ y uzœ = dz¿兾d t ¿. La velocidad de P vista desde S tendrá componentes ux,
uy y uz. Se puede demostrar cómo se relacionan éstas con las componentes de veloci-
dad en S9 al diferenciar las ecuaciones 36-4. Para ux se obtiene
dx d(x¿ + vt ¿)
ux = = = uxœ + v
dt dt ¿
pues v se supuso constante. Para las otras componentes, uyœ = uy y uzœ = uz , de mane-
ra que se tiene

968 CAPÍTULO 36 La teoría especial de la relatividad


ux = uxœ + v
uy = uyœ [galileana] (36–5)
uz = uzœ .
Éstas se conocen como ecuaciones de transformación galileana de velocidad. Se ve que
las componentes y y z de velocidad no cambian, pero las componentes x difieren por v:
ux = uxœ + v. Esto es justo lo que se empleó antes (véase el capítulo 3, sección 3-9)
cuando se trató con velocidad relativa.
Las transformaciones galileanas, ecuaciones 36-4 y 36-5, sólo son válidas cuando
las velocidades implicadas son mucho menores que c. Se puede ver, por ejemplo, que la
primera de las ecuaciones 36-5 no funcionará para la rapidez de la luz: la luz que viaja
en S9 con rapidez uxœ = c tendría rapidez c 1 v en S, mientras que la teoría de la rela-
tividad insiste en que debe ser c en S. Así que, sin duda, se necesita un nuevo conjunto
de ecuaciones de transformación para lidiar con velocidades relativistas.
Deducimos la ecuación requerida al observar de nuevo la figura 36-11. Se tratará
la suposición simple de que la transformación es lineal y de la forma
x = g(x¿ + vt ¿), y = y¿, z = z¿. (i)
Esto es, modificamos la primera de las ecuaciones 36-4 multiplicando por una constan-
te g que todavía se tiene que determinar† (g 5 1 no relativista). Pero se supone que las
ecuaciones y y z no cambian, pues no hay contracción de la longitud en estas direccio-
nes. No se supondrá una forma para t, pero se le deducirá. Las ecuaciones inversas de-
ben tener la misma forma sólo que con 2v en lugar de v. (El principio de relatividad lo
demanda, pues S9 que se mueve hacia la derecha con respecto a S es equivalente a S
que se mueve hacia la izquierda con respecto a S9). Por lo tanto,
x¿ = g(x - vt). (ii)
Ahora, si un pulso de luz sale del origen común de S y S9 en el tiempo t 5 t9 5 0, des-
pués de un tiempo t habrá recorrido una distancia x 5 ct o x9 5 ct9 a lo largo del eje x.
En consecuencia, a partir de las ecuaciones (i) y (ii) anteriores,
ct = g(ct ¿ + vt ¿) = g(c + v) t ¿, (iii)
ct ¿ = g(ct - vt) = g(c - v)t. (iv)
Sustituya t9 de la ecuación (iv) en la ecuación (iii) y encuentre ct 5 g(c 1 v) g(c 2
v)(tyc) 5 g2 Ac2 - v2 B t兾c. Cancele la t en cada lado y despeje g para encontrar
g = 1 .
31 - v 兾c
2 2

Aquí la constante g tiene el mismo valor que la g que se usó antes, ecuación 36-2. Aho-
ra que se encontró g, sólo se necesita encontrar la relación entre t y t9. Para ello, com-
bine x¿ = g(x - vt) con x = g(x¿ + vt ¿):
x¿ = g(x - vt) = g(g[x¿ + vt ¿] - vt).
Despeje t para obtener t = g A t ¿ + vx¿兾c2 B. En resumen,
x = g(x¿ + vt ¿)
y = y¿
TRANSFORMACIONES
z = z¿ (36–6)
DE LORENTZ
vx¿
t = g ¢ t¿ + 2 ≤
c
Estas ecuaciones se conocen como la transformación de Lorentz. Lorentz las propuso
por primera vez en 1904, en una forma ligeramente diferente, para explicar el resulta-
do nulo del experimento de Michelson y Morley y para hacer que las ecuaciones de
Maxwell tomaran la misma forma en todos los marcos de referencia inerciales. Un año
después Einstein las dedujo de manera independiente con base en su teoría de la rela-
tividad. Observe que no sólo se modificó la ecuación x en comparación con la transfor-
mación galileana, sino también la ecuación t; de hecho, en esta última ecuación se ve
directamente cómo se mezclan las coordenadas espacio y tiempo.


Aquí no se supuso que g está dada por la ecuación 36-2.

SECCIÓN 36–8 Transformaciones galileanas y de Lorentz 969


Deducción de la contracción de la longitud
Ahora se deducirá la fórmula de contracción de la longitud, ecuación 36-3, a partir de
las ecuaciones de transformación de Lorentz. Considere dos marcos de referencia S y
S9, como en la figura 36-11.
Sea un objeto con longitud l0 en reposo sobre el eje x en S. Las coordenadas de sus
dos puntos extremos son x1 y x2, de manera que x2 – x1 5 l0. En cualquier instante en S9,
los puntos extremos estarán en x 1œ y x 2œ según las ecuaciones de transformación de Lo-
rentz. La longitud medida en S9 es l = x 2œ - x 1œ . Un observador en S9 determina esta
longitud al medir x 2œ y x 1œ al mismo tiempo (en el marco de referencia S9), de manera
que t2œ = t1œ . Así, a partir de la primera de las ecuaciones 36-6,
1
l0 = x2 - x1 = Ax 2œ + vt2œ - x1œ - vt1œ B.
31 - v 兾c
2 2

Puesto que t2œ = t1œ , se tiene


1 l
l0 = Ax 2œ - x 1œ B = ,
31 - v 兾c 31 - v 兾c
2 2 2 2

o
l = l031 - v2兾c2 ,
que es la ecuación 36-3.

Deducción de la dilatación del tiempo


Ahora se deducirá la fórmula de dilatación del tiempo, ecuación 36-1a, a partir de las
ecuaciones de transformación de Lorentz.
El tiempo Dt0 entre dos eventos que ocurren en el mismo lugar Ax 2œ = x 1œ B en S9 se
mide en ¢ t0 = t2œ - t1œ . Puesto que x 2œ = x 1œ , entonces, a partir de la última de las ecua-
ciones 36-6, el tiempo Dt entre los eventos medidos en S es
1 vx2œ vx1œ
¢ t = t2 - t1 = ¢ t2œ + - t œ
1 - ≤
31 - v 兾c
2 2 c2 c2
1 ¢ t0
= A t2œ - t1œ B = ,
3 1 - v 2
兾c 2
3 - v 兾c
1 2 2

que es la ecuación 36-1a. Note que se elige S9 como el marco donde ocurren los dos eventos
en el mismo lugar, de manera que se cancelan x 1œ = x 2œ y los términos que contienen x 1œ y x 2œ .
Suma relativista de velocidades
Las ecuaciones de velocidad relativísticamente correctas se obtienen con facilidad al
derivar las ecuaciones 36-6 con respecto al tiempo. Por ejemplo (con g = 1兾31 - v2兾c2
y la regla de la cadena para derivadas):

C g Ax¿ + vt ¿ B D
dx d
ux = =
dt dt
dt ¿ dt ¿ .
C Ax¿ + vt ¿ B D = gc + vd
d g dx¿
=
dt ¿ dt dt ¿ dt
Pero dx¿兾dt ¿ = ux y dt ¿兾dt = 1兾(dt兾dt ¿) = 1兾C A1 + vux兾c2 B D donde se derivó la
œ g œ

última de las ecuaciones 36-6 con respecto al tiempo. Por lo tanto,


C g Auxœ + vB D uxœ + v
ux = = .
C g A1 + vux兾c B D
œ 2
1 + vuxœ 兾c2
Las otras se obtienen en la misma forma y se les recopila a continuación:
uxœ + v
ux = (36–7a)
1 + vuxœ 兾c2
TRANSFORMACIONES uyœ 31 - v2兾c2
DE VELOCIDAD uy = (36–7b)
DE LORENTZ 1 + vuxœ 兾c2
uzœ 31 - v2兾c2
uz = . (36–7c)
1 + vuxœ 兾c2
B
Observe que, aun cuando la velocidad relativista v esté en la dirección x, si el objeto
tiene componentes de velocidad y o z, éstas también resultan afectadas por v y por la
componente x de velocidad del objeto. Esto no fue cierto para la transformación gali-
970 CAPÍTULO 36 leana, ecuaciones 36-5.
EJEMPLO 36–8 Suma de velocidades. Calcule la rapidez del cohete 2 en la fi-
gura 36-12 con respecto a la Tierra. u = 0.60c con 2
respecto al
PLANTEAMIENTO Considere la Tierra como el marco de referencia S y el cohete 1 cohete 1
como el marco de referencia S9. El cohete 2 viaja con rapidez u9 5 0.60c con respecto 1
al cohete 1. El cohete 1 tiene rapidez v 5 0.60c con respecto a la Tierra. Las velocida-
des son a lo largo de la misma línea recta que se considera como el eje x (y x9). Sólo
es necesario usar la primera de las ecuaciones 36-7. Tierra v = 0.60c con
SOLUCIÓN La rapidez del cohete 2 con respecto a la Tierra es respecto a Tierra

u¿ + v 0.60c + 0.60c 1.20c


u = = = = 0.88c.
vu¿ (0.60c)(0.60c) 1.36
1 + 2 1 +
c c2
FIGURA 36–12 El cohete 1 se aleja
NOTA La transformación galileana daría u 5 1.20c.
con rapidez v 5 0.60c. El cohete 2 se
lanza desde el cohete 1 con rapidez u9
5 0.60c. ¿Cuál es la rapidez del cohete
EJERCICIO E Con base en las ecuaciones 36-7, calcule la rapidez del cohete 2 en la figura 2 con respecto a la Tierra?
36-12 en relación con la Tierra, si se lanza desde el cohete 1 con una rapidez u9 5 3000
kmys 5 0.010c. Suponga que el cohete 1 tiene una rapidez v 5 6000 mys 5 0.020c.

EJERCICIO F Regrese a la pregunta de inicio de capítulo, página 951, y respóndala de nue-


vo. Trate de explicar por qué quizás usted la respondió de manera diferente la primera vez.

Advierta que las ecuaciones 36-7 se reducen a las formas clásicas (galileanas) para
velocidades pequeñas comparadas con la rapidez de la luz, pues 1 + vu¿兾c2 L 1 para v
y u9 V c. En el otro extremo, suponga que el cohete 1 en la figura 36-12 envía un haz
de luz, de manera que u9 5 c. Entonces la ecuación 36-7 dice que la rapidez de la luz en
relación con la Tierra es

0.60c + c
u = = c,
(0.60c)(c)
1 +
c2
que es congruente con el segundo postulado de la relatividad.

36–9 Cantidad de movimiento relativista


Hasta ahora en este capítulo se ha visto que dos cantidades mecánicas básicas (longi-
tud e intervalos de tiempo) necesitan modificación porque son relativas: sus valores de-
penden del marco de referencia donde se miden. Cabe esperar que otras cantidades
físicas también necesiten cierta modificación de acuerdo con la teoría de la relatividad,
como la cantidad de movimiento (o momento lineal), la energía y la masa.
El análisis de las colisiones entre dos partículas demuestra que si se quiere preser-
var la ley de conservación de la cantidad de movimiento en la relatividad, la cantidad
de movimiento se debe redefinir como
mv
p = = gmv. (36–8)
31 - v 兾c
2 2

Aquí g es abreviatura para 1兾31 - v2兾c2, como antes (ecuación 36-2). Para casos en
que la rapidez es mucho menor que la rapidez de la luz, la ecuación 36-8 da la cantidad
de movimiento clásica, p 5 mv.
La cantidad de movimiento relativista se ha puesto a prueba muchas veces en pe-
queñas partículas elementales (como los muones) y se encontró que se comporta en
concordancia con la ecuación 36-8. La ecuación 36-8 se deduce en el subapartado de la
siguiente página.

SECCIÓN 36–9 Cantidad de movimiento relativista 971


EJEMPLO 36–9 Cantidad de movimiento de un electrón en movimiento. Com-
pare la cantidad de movimiento de un electrón cuando tiene una rapidez de a) 4.00 3 107
mys en el TRC de un televisor y b) 0.98c en un acelerador que se usa en la terapia de
cáncer.
PLANTEAMIENTO Utilice la ecuación 36-8 para la cantidad de movimiento de un
electrón en movimiento.
SOLUCIÓN a) v 5 4.00 3 107 mys, la cantidad de movimiento del electrón es
mv mv
p = = = 1.01mv.
v2 A4.00 * 107 m兾sB 2
1 - 2 1 -
C c C A3.00 * 108 m兾sB 2

El factor g = 1兾31 - v2兾c2 L 1.01, de manera que la cantidad de movimiento sólo


es aproximadamente 1% mayor que el valor clásico. (Si se considera la masa del electrón,
m 5 9.11 3 10231 kg, la cantidad de movimiento es p = 1.01mv = 3.68 * 10–23 kg m兾s).
b) Con v 5 0.98c, la cantidad de movimiento es
mv mv mv
p = = = = 5.0mv.
31 - (0.98)
2 2
v2 (0.98c)
1 - 2 1 -
C c C c2
Un electrón que viaja al 98% de la rapidez de la luz tiene g 5 5.0 y una cantidad de
movimiento 5.0 veces su valor clásico.

La segunda ley de Newton, enunciada en la forma más general, es


B B
B dp d d mv
F = = (gmv) =
B
¢ ≤ (36–9)
dt dt dt 31 - v2兾c2
y es válida de manera relativista.

* Deducción de la cantidad de movimiento relativista


De manera clásica, la cantidad de movimiento es una cantidad que se conserva. Se es-
pera encontrar una fórmula para la cantidad de movimiento que también sea válida de
manera relativista. Para hacerlo, suponga que tiene la forma general dada por p 5
fmv, donde f es alguna función de v: f(v). Considere una colisión hipotética entre dos
objetos —un experimento mental— y vea qué forma debe tomar f(v) para que la can-
tidad de movimiento se conserve.
El experimento mental implica la colisión elástica de dos pelotas idénticas, A y B.
Considere dos marcos de referencia inerciales, S y S9, que se mueven a lo largo del eje x
con una rapidez v uno con respecto al otro (figura 36-13). Esto es, el marco S9 se mueve
B
hacia la derecha con velocidad v con respecto a los observadores en el marco S; y el
B
marco S se mueve hacia la izquierda con –v con respecto a los observadores en S9. En
el marco de referencia S, la pelota A se lanza con rapidez u en la dirección 1y. En el
marco de referencia S9, la pelota B se lanza con rapidez u en el sentido negativo de la
dirección y9. Las dos pelotas se lanzan justo en el tiempo correcto, de manera que cho-
can. Suponga que rebotan elásticamente y que, a partir de simetría, cada una se mueve
con la misma rapidez u de regreso en sentido opuesto en el marco de referencia de su
lanzador. La figura 36-13a ilustra la colisión como la ve un observador en el marco de
referencia S, y la figura 36-13b representa la colisión vista desde el marco de referencia
S9. En el marco de referencia S, la pelota A tiene vx 5 0 tanto antes como después de la
colisión; tiene vy 5 1u antes de la colisión y –u después de la colisión. En el marco S9,
la pelota A tiene componente de velocidad x uxœ = –v tanto antes como después de la
colisión, y una componente y9 (ecuación 36-7b con uxœ = –v ) de magnitud
uyœ = u 31 - v2兾c2 .
Lo mismo es cierto para la pelota B, excepto que a la inversa. Las componentes de ve-
locidad se indican en la figura 36-13.
Ahora aplicamos la ley de conservación de la cantidad de movimiento, que se es-
pera siga siendo válida en la relatividad, incluso si la cantidad de movimiento debe re-
definirse. Esto es, suponga que la cantidad de movimiento total antes de la colisión es
igual a la cantidad de movimiento después de la colisión. Aplicamos la conservación de
la cantidad de movimiento a la componente y de la cantidad de movimiento en el marco

972 CAPÍTULO 36 La teoría especial de la relatividad


B v S¢
vB Punto
-u 1 - v2/c 2
y¢ de colisión

S
u
y A
a) x


B
-u

S
FIGURA 36–13 Deducción de la
Punto
de colisión fórmula de cantidad de movimiento.
- vB
u 1 - v 2/c 2 Colisión según la ven observadores
y
-v A a) en el marco de referencia S, b) en el
b) x marco de referencia S9.

de referencia S (figura 36-13a). Para hacer la labor más sencilla, suponga que u V v, de
manera que la rapidez de la pelota B, vista en el marco de referencia S, en esencia es v.
Entonces la componente y de la cantidad de movimiento de B en S antes de la colisión
es –f(v)mu 21 - v2兾c2 y después de la colisión es ±f(v)mu 21 - v2兾c2 . La pelota
A en S tiene componente y f(u)mu antes y 2f(u)mu después de la colisión. (Se usa
f(u) para A porque su rapidez en S sólo es u). La conservación de la cantidad de mo-
vimiento en S para la componente y es
(pA + pB)antes = (pA + pB)después

f(u)mu - f(v)mu31 - v2兾c2 = –f(u)mu + f(v)mu31 - v2兾c2.


Se despeja f(v) para obtener
f(u).
f(v) =
3 1 - v 2
兾c 2

Para simplificar esta relación, de manera que se pueda despejar f, permita que u
se vuelva muy pequeña, de manera que tienda a cero (esto corresponde a una colisión
oblicua con una de las pelotas esencialmente en reposo y la otra en movimiento con ra-
pidez v). Entonces los términos f(u)mu de la cantidad de movimiento están en el ám-
bito no relativista y toman la forma clásica, simplemente mu, lo cual significa que f(u)
5 1. De esta manera, la ecuación anterior se convierte en
1 .
f(v) =
3 1 - v 2
兾c 2

Vemos que f(v) resulta ser el factor que se empleó antes como g, y aquí se demostró
que es válido para la pelota A. Al usar la figura 36-13b se puede deducir la misma rela-
ción para la pelota B. Por ende, se concluye que es necesario definir la cantidad de mo-
B
vimiento relativista de una partícula que se mueve con velocidad v como
B
mv
p =
B
= gmv. B

3 - v 兾c
2 2
1
Con esta definición, la ley de conservación de la cantidad de movimiento seguirá sien-
do válida incluso en el reino relativista. Esta fórmula de cantidad de movimiento rela-
tivista (ecuación 36-8) se probó incontables veces en pequeñas partículas elementales y
resultó ser válida.

SECCIÓN 36–9 Cantidad de movimiento relativista 973


* Masa relativista
La definición relativista de la cantidad de movimiento, ecuación 36-8, a veces se inter-
preta como un aumento en la masa de un objeto. En esta interpretación, una partícula
puede tener una masa relativista, mrel, que aumenta con la rapidez de acuerdo con
m .
mrel =
31 - v 兾c
2 2

En esta fórmula de “aumento de masa”, m se conoce como la masa en reposo del obje-
to. Con esta interpretación, la masa de un objeto parece aumentar conforme su rapidez
aumenta. Pero debemos tener cuidado al emplear la masa relativista. No se puede tan
sólo ponerla en fórmulas como F 5 ma o K = 12 mv2. Por ejemplo, si la sustituimos en
F 5 ma, se obtiene una fórmula que no concuerda con los experimentos.
B
Sin embargo,
si la segunda ley de Newton se escribe en su forma más general, F = dp兾dt, se obtiene
B

un resultado correcto (ecuación 36-9).


Además, tenga cuidado de no pensar que una masa adquiere más partículas o más
moléculas conforme su rapidez aumenta, porque no es así. De hecho, muchos físicos
creen que un objeto sólo tiene una masa (su masa en reposo) y que sólo es la cantidad
de movimiento la que aumenta con la rapidez.
Siempre que se hable de masa de un objeto, nos referiremos a su masa en reposo
(un valor fijo).

36–10 La rapidez última


Un resultado básico de la teoría especial de la relatividad es que la rapidez de un obje-
to no puede ser igual a la rapidez de la luz, ni tampoco puede superarla. El hecho de
que la rapidez de la luz es un límite natural de la rapidez en el Universo se puede cons-
tatar a partir de cualquiera de las ecuaciones 36-1, 36-2, 36-8 o la fórmula de suma de
velocidades. Tal vez sea más fácil de ver a partir de la ecuación 36-8. Conforme un ob-
jeto acelera hacia una rapidez cada vez mayor, su cantidad de movimiento se vuelve
más y más grande. De hecho, si v fuera igual a c, el denominador en esta ecuación sería
cero y la cantidad de movimiento sería infinita. En consecuencia, acelerar un objeto a
v 5 c requeriría energía infinita, algo que no es posible.

36–11 E 5 mc2; masa y energía


Si la cantidad de movimiento necesita modificación para concordar con la relatividad,
como se acaba de ver en la ecuación 36-8, entonces cabe esperar que la energía tam-
bién necesite repensarse. De hecho, Einstein no sólo desarrolló una nueva fórmula pa-
ra la energía cinética, sino que también encontró una nueva relación entre masa y
energía, y la sorprendente idea de que la masa es una forma de energía.
Comencemos con el principio trabajo-energía (capítulo 7), esperando que todavía
sea válido en la relatividad y que dé resultados verificables. Esto es, suponga que el
trabajo neto realizado sobre una partícula es igual a su cambio en energía cinética (K).
Con base en este principio, Einstein demostró que, a alta rapidez, la fórmula K = 12 mv2
no es correcta. En vez de ello, como se verá en el subapartado de la página 978, la ener-
gía cinética de una partícula de masa m que viaja con rapidez v está dada por
mc2
K = - mc2. (36–10a)
31 - v 兾c
2 2

En términos de g = 1兾31 - v2兾c2 la ecuación 36-10a se rescribe como


K = gmc2 - mc2 = (g - 1)mc2. (36–10b)
Las ecuaciones 36-10 requieren cierta interpretación. El primer término aumenta con la
rapidez v de la partícula. El segundo término, mc2, es constante; se le llama energía en re-
poso de la partícula y representa una forma de energía que una partícula tiene incluso
cuando está en reposo. Advierta que, si una partícula está en reposo (v 5 0), el primer
término en la ecuación 36-10a se convierte en mc2, de manera que K 5 0, como debiera.
La ecuación 36-10b se reordena para obtener
gmc2 = mc2 + K.
A gmc2 se le llama la energía total E de la partícula (suponiendo que no hay energía
potencial), porque es igual a la energía en reposo más la energía cinética:

974 CAPÍTULO 36 La teoría especial de la relatividad


E = K + mc2. (36–11a)
La energía total también se puede expresar, con las ecuaciones 36-10, como
mc2 .
E = gmc2 = (36–11b)
31 - v 兾c
2 2

Para una partícula en reposo en un marco de referencia dado, K es cero en la


ecuación 36-11a, de manera que la energía total es su energía en reposo:

E = mc2. (36–12) MASA RELACIONADA


CON ENERGÍA
He aquí la famosa fórmula de Einstein, E 5 mc2. Esta fórmula relaciona matemática-
mente los conceptos de energía y masa. Pero, si esta idea debe tener algún significado
físico, entonces la masa debería ser convertible a otras formas de energía y viceversa.
Einstein sugirió que esto era posible, y de hecho se han confirmado experimentalmen-
te cambios de masa a otras formas de energía, y viceversa, en incontables ocasiones en
física nuclear y de partículas elementales. Por ejemplo, un electrón y un positrón (5 un
electrón positivo, sección 37-5) con frecuencia chocan y desaparecen, lo que produce
radiación electromagnética pura. La cantidad de energía electromagnética producida
es exactamente igual a la que predice la fórmula de Einstein, E 5 mc2. El proceso in-
verso también se observa comúnmente en el laboratorio: en ciertas condiciones, la ra-
diación electromagnética se puede convertir en partículas materiales como los
electrones (véase la sección 37-5 acerca de la producción de pares). A mayor escala, la
energía producida en las plantas nucleares es resultado de la pérdida de masa del ura-
nio (que sirve como combustible) conforme éste experimenta el proceso llamado fi-
sión. Incluso la energía radiante que se recibe del Sol es un ejemplo de E 5 mc2; la
masa del Sol continuamente disminuye conforme irradia energía electromagnética ha-
cia el exterior.
Ahora se cree que la relación E 5 mc2 se aplica a todos los procesos, aunque los
cambios a menudo son demasiado pequeños para medirse. Esto es, cuando la energía
de un sistema cambia por una cantidad DE, la masa del sistema cambia por una canti-
dad Dm dada por
¢E = (¢m)Ac2 B.
En una reacción nuclear donde se requiere o se libera una energía E, las masas de los
reactantes y los productos serán diferentes en Dm 5 DEyc2.

EJEMPLO 36–10 Energía cinética de pión. Un mesón p0 (m 5 2.4 3 10228 kg)


viaja con rapidez v 5 0.80c 5 2.4 3 108 mys. ¿Cuál es su energía cinética? Compare
con un cálculo clásico.
PLANTEAMIENTO Use la ecuación 36-10 y compare con 12 mv2.
SOLUCIÓN Sustituya valores en la ecuación 36-10a o b
K = (g - 1)mc2
donde
g = 1 1
= = 1.67.
31 - v 兾c 31 - (0.80)
2 2 2

Luego

K = (1.67 - 1)A2.4 * 10–28 kgBA3.0 * 108 m兾sB 2 R ESOLUCIÓN DE P ROB LEMAS


Energía cinética relativista
= 1.4 * 10 –11
J.
Note que las unidades de mc son kg?m2ys2, que es el joule.
2

NOTA Si tuviéramos que realizar un cálculo clásico obtendríamos K = 12 mv2 =


2 A2.4 * 10 kgB A2.4 * 108 m兾sB 2 = 6.9 * 10 –12 J, aproximadamente la mitad, pe-
1 –28

ro éste no es un resultado correcto. Advierta que 12 g mv2 tampoco funcionará.

EJERCICIO G Un protón viaja en un acelerador con una rapidez de 1.0 3 108 mys. ¿En
qué factor aumenta la energía cinética del protón si su rapidez se duplica? a) 1.3, b) 2.0
c) 4.0 d) 5.6.

SECCIÓN 36–11 E ⴝ mc 2; masa y energía 975


EJEMPLO 36–11 Energía a partir de decaimiento nuclear. La energía requerida
o liberada en las reacciones y los decaimientos nucleares proviene de un cambio en
masa entre las partículas iniciales y finales. En un tipo de decaimiento radiactivo, un
átomo de uranio (m 5 232.03714 u) decae a un átomo de torio (m 5 228.02873 u)
más un átomo de helio (m 5 4.00260 u), donde las masas dadas están en unidades de ma-
sa atómica (1 u 5 1.6605 3 10227 kg). Calcule la energía liberada en este decaimiento.
PLANTEAMIENTO La masa inicial menos la masa final total da la pérdida de masa
en unidades de masa atómica (u); convierta esto a kg y multiplique por c2 para encon-
trar la energía liberada, DE 5 Dmc2.
SOLUCIÓN La masa inicial es 232.03714 u, y después del decaimiento la masa es
228.02873 u 1 4.00260 u 5 232.03133 u, así que existe una disminución en masa de
0.00581 u. Esta masa, que es igual a (0.00581 u)A1.66 * 10 –27 kgB = 9.64 * 10 –30 kg,
se convierte en energía. Por DE 5 Dmc2 se tiene
¢E = A9.64 * 10–30 kgBA3.0 * 108 m兾sB 2 = 8.68 * 10–13 J.
Puesto que 1 MeV 5 1.60 3 10213 J (sección 23-8), la energía liberada es de 5.4 MeV.

En el diminuto mundo de átomos y núcleos, es común expresar las energías en eV


(electrón volts) o múltiplos como MeV (106 eV). La cantidad de movimiento (véase la
ecuación 36-8) se puede expresar en unidades de eVyc (o MeVyc). Y la masa se puede
citar (a partir de E 5 mc2) en unidades de eVyc2 (o MeVyc2). Advierta el uso de c pa-
ra mantener las unidades correctas. Se demuestra fácilmente que las masas en reposo
del electrón y el protón son 0.511 MeVyc2 y 938 MeVyc2, respectivamente. Véase tam-
bién la tabla en la segunda de forros.

EJEMPLO 36–12 Un protón de 1 TeV. El acelerador Tevatrón en el Fermilab en


Illinois es capaz de acelerar protones a una energía cinética de 1.0 TeV (1012 eV).
¿Cuál es la rapidez de un protón en estas condiciones?
PLANTEAMIENTO Despeje v en la fórmula de energía cinética, ecuación 36-10.
SOLUCIÓN La energía en reposo de un protón es mc2 5 938 MeV o 9.38 3 108 eV.
Comparada con la energía cinética de 1012 eV, la energía en reposo se puede despre-
ciar, así que la ecuación 36-10a se simplifica a
mc2 .
K L
31 - v 兾c
2 2

Despeje v en los siguientes pasos:


v2 mc2
1 - = ;
C c2 K
v2 mc2 2
1 - 2 = ¢ ≤ ;
c K
v2 mc2 2 9.38 * 108 eV 2
= 1 - ¢ ≤ = 1 - ¢ ≤ ;
c2 K 1.0 * 1012 eV
v = 31 - A9.38 * 10–4 B 2 c = 0.99999956 c.
Así que el protón viaja con una rapidez muy cercana a c.

A baja rapidez, v V c, la fórmula relativista para la energía cinética se reduce a la


clásica, como se demuestra ahora empleando el desarrollo binomial, (16x)n 5
16nx + n(n - 1)x2兾2! + p . Con n = – 12 , se desarrolla la raíz cuadrada en la
ecuación 36-10a
1
K = mc2 ¢ - 1≤
31 - v 兾c
2 2

de manera que
1 v2
K L mc2 ¢ 1 + + p - 1≤
2 c2
L 1 2
2 mv .

Los puntos en la primera expresión representan términos muy pequeños en la expan-


sión que se desprecian, pues se supuso que v V c. Por lo tanto, a baja rapidez, la forma
976 CAPÍTULO 36
relativista para la energía cinética se reduce a la forma clásica, K = 12 mv2. Esto hace
de la relatividad una teoría viable en tanto que permite predecir resultados exactos
tanto a baja como a alta rapidez. De hecho, las otras ecuaciones de la relatividad espe-
cial también se reducen a sus equivalentes clásicas a rapidez ordinaria: contracción de
la longitud, dilatación del tiempo y modificaciones a la cantidad de movimiento, así como
a la energía cinética, todas desaparecen para v V c pues 31 - v2兾c2 L 1.
También se deduce una relación útil entre la energía total E de una partícula y su
cantidad de movimiento p. La cantidad de movimiento de una partícula con masa m y
rapidez v está dada por la ecuación 36-8:
mv .
p = gmv =
31 - v 兾c
2 2

La energía total es
E = K + mc2
o
mc2 .
E = gmc2 =
31 - v 兾c
2 2

Al elevar al cuadrado esta ecuación (e insertar “v2 2 v2” que es cero, pero que ayudará):
m2c2 Av2 - v2 + c2 B
E2 =
1 - v2兾c2
m2c4 A1 - v2兾c2 B
= p 2c 2 +
1 - v2兾c2
o
E 2 = p2c2 + m2c4. (36–13)
Por ende, la energía total se puede escribir en términos de la cantidad de movimiento
p, o en términos de la energía cinética (ecuación 36-11), donde se supuso que no hay
energía potencial.
La ecuación 36-13 se rescribe como E2 2 p2c2 5 m2c4. Puesto que la masa m de
una partícula dada es la misma en cualquier marco de referencia, se ve que la cantidad
E2 2 p2c2 también debe ser la misma en cualquier marco de referencia. Por lo tanto, en
cualquier momento dado, la energía total E y la cantidad de movimiento p de una par-
tícula serán diferentes en distintos marcos de referencia, pero la cantidad E2 2 p2c2
tendrá el mismo valor en todos los marcos de referencia inerciales. Se dice que la can-
tidad E2 2 p2c2 es invariante ante transformaciones de Lorentz.

* ¿Cuándo se usan fórmula relativistas?


Desde un punto de vista práctico, en la vida diaria no se tiene mucha oportunidad para
usar las matemáticas de la relatividad. Por ejemplo, el factor g, g = 1兾31 - v2兾c2 ,
que aparece en muchas fórmulas relativistas, tiene un valor de 1.005 cuando v 5 0.10c.
Por lo tanto, para una rapidez incluso tan alta como 0.10c 5 3.0 3 107 mys, el factor
31 - v 兾c en las fórmulas relativistas brinda una corrección numérica de menos de
2 2

1%. Para una rapidez menor que 0.10c, o a menos que masa y energía se intercambien,
generalmente no se necesita usar las fórmulas relativistas más complicadas, y se pue-
den usar las fórmulas clásicas más sencillas.
Si se conoce la masa m de una partícula y su energía cinética K, se hace un cálculo
rápido para determinar si es necesario usar fórmulas relativistas o si las clásicas son su-
ficientemente buenas. Simplemente calcule la razón Kymc2 porque (ecuación 36-10b)
K 1
= g - 1 = - 1.
mc2 31 - v 兾c
2 2

Si esta razón resulta ser menor que 0.01, por ejemplo, entonces g # 1.01 y las ecuacio-
nes relativistas corregirán a las clásicas por más o menos 1%. Si la precisión esperada
no es mejor que el 1%, las fórmulas clásicas son suficientemente buenas. Pero si se re-
quiere una precisión de 1 parte en 1000 (0.1%), entonces es conveniente usar las
fórmulas relativistas. Si la precisión esperada sólo es del 10%, necesita relatividad si
AK兾mc2 B g 0.1.

EJERCICIO H Para 1% de exactitud, ¿un electrón con K 5 100 eV necesita tratarse de ma-
nera relativista? [Sugerencia: Considere que la masa de un electrón es de 0.511 MeV].
SECCIÓN 36–11 977
* Deducción de la energía relativista
Para encontrar la relación matemática entre masa y energía, suponga que el teorema
trabajo-energía todavía es válido en la relatividad para una partícula, y considere que
el movimiento de la partícula es a lo largo del eje x. El trabajo realizado para aumen-
tar la rapidez de una partícula desde cero hasta v es
f f f f

冮 冮 冮 冮 v dp
dp dp
W = F dx = dx = v dt =
i i dt i dt i

donde i y f se refieren a los estados inicial (v 5 0) y final (v 5 v). Puesto que d(pv) 5
p dv 1 v dp, se puede escribir
v dp = d(pv) - p dv
de manera que
f f
W = 冮 d(pv) - 冮 p dv.
i i

El primer término a la derecha del signo igual es


f
f
mv2
冮i
d(pv) = pv 2
i
= (gmv)v =
31 - v 兾c
2 2
.

El segundo término en la ecuación anterior para W se integra fácilmente, pues


d
A 1 - v2兾c2 B = –Av兾c2 B兾31 - v2兾c2 ,
dv 3
y por lo tanto se convierte en
f v v

冮 冮 31 - v 兾c
mv
– p dv = – dv = mc2 31 - v2兾c2 2
2 2
i 0 0

= mc2 31 - v2兾c2 - mc2.


Finalmente, para W se tiene:
mv2
W = + mc2 31 - v2兾c2 - mc2.
3 1 - v 2
兾c 2

Al multiplicar el segundo término a la derecha por 31 - v2兾c2兾31 - v2兾c2 = 1, se


obtiene
mc2
W = - mc2.
31 - v 兾c
2 2

Por el teorema trabajo-energía, el trabajo realizado sobre la partícula debe ser igual a
su energía cinética final K, pues la partícula partió desde el reposo. En consecuencia,
mc2
K = - mc2
31 - v 兾c
2 2

= gmc2 - mc2 = (g - 1)mc2,


que son las ecuaciones 36-10.

* 36–12 Corrimiento Doppler para la luz


En la sección 16-7 se estudió cómo la frecuencia y la longitud de onda del sonido se al-
teran si la fuente del sonido y el observador se acercan o se alejan entre sí. Cuando una
fuente se mueve hacia uno, la frecuencia es mayor que cuando la fuente está en repo-
so. Si la fuente se aleja de uno, la frecuencia es menor. Se obtuvieron cuatro diferentes
ecuaciones para el corrimiento Doppler (ecuaciones 16-9a y b, ecuaciones 16-10a y b),
dependiendo de la dirección del movimiento relativo y de si la fuente o el observador
están en movimiento. El efecto Doppler también ocurre para la luz; pero la frecuencia
o longitud de onda recorrida se obtiene con ecuaciones ligeramente diferentes, y sólo
hay dos de éstas, porque en el caso de la luz, de acuerdo con la relatividad especial, no
se puede hacer distinción entre movimiento de la fuente y movimiento del observador.

978 CAPÍTULO 36 La teoría especial de la relatividad


(Recuerde que el sonido viaja en un medio como el aire, mientras que la luz no lo ha- Fuente Observador
ce: no hay evidencia de un éter). (en reposo)
Para determinar el corrimiento Doppler para la luz, considere una fuente de luz y l0
un observador que se mueven uno hacia el otro, y sea v su velocidad relativa según se S
mide en el marco de referencia ya sea de la fuente o del observador. La figura 36-14a
ilustra una fuente en reposo que emite ondas de luz de frecuencia f0 y longitud de on- a)
da l0 5 cyf0. Se aprecian dos crestas de onda, separadas una distancia l0, la segunda
cresta apenas se emitió. En la figura 36-14b, la fuente se muestra en movimiento con
Cresta emitida cuando la
rapidez v hacia un observador estacionario que verá la longitud de onda l un poco me- fuente estaba en el punto 2
nor que l0. (Esto es muy parecido a la figura 16-19 para el sonido). Sea Dt el tiempo
entre crestas como las detecta el observador, cuyo marco de referencia se representa en Cresta emitida
la figura 36-14b. A partir de la figura 36-14b se ve que cuando la fuente
1 2 estaba en el
l
l = c ¢ t - v ¢ t, S v S punto 1
donde c Dt es la distancia que la cresta 1 se movió en el tiempo Dt después de ser emi- vDt
tida, y v Dt es la distancia que la fuente se movió en el tiempo Dt. Hasta el momento, cDt
nuestra deducción no difiere de la del sonido (sección 16-7). Ahora se invocará la teoría b)
de la relatividad. El tiempo entre emisión de crestas de onda experimentó dilatación FIGURA 36–14 Corrimiento
del tiempo: Doppler para la luz. a) Fuente y
¢ t = ¢ t0兾31 - v2兾c2 observador en reposo. b) Fuente que
se mueve hacia un observador
donde Dt0 es el tiempo entre emisiones de crestas de onda en el marco de referencia don- estacionario.
de la fuente está en reposo (el tiempo “propio”). En el marco de referencia de la fuente
(figura 36-14a), se tiene
1 l0
¢ t0 = =
f0 c
(ecuaciones 5-2 y 31-14). Por consiguiente,
¢ t0 (c - v)
l = (c - v) ¢ t = (c - v) = l0
31 - v 兾c 3c - v
2 2 2 2

o
c - v. fuente y observador
l = l0 £ se acercan § (36–14a)
Bc + v entre sí
La frecuencia f es (recuerde que l0 5 cyf0)

c + v. fuente y observador
£ § (36–14b)
c
f = = f0 se elejan
l Bc - v entre sí
Aquí, f0 es la frecuencia de la luz vista en el marco de referencia de la fuente, y f es la
frecuencia medida por un observador que se mueve hacia la fuente o hacia quien
la fuente se mueve. Las ecuaciones 36-14 sólo dependen de la velocidad relativa v. Para
movimiento relativo en el que hay alejamiento entre fuente y observador se establece
que v , 0 en las ecuaciones 36-14 y se obtiene
c + v
l = l0 (36–15a)
B - v
c
fuente y observador
c - v. £ se alejan §
f = f0 entre sí (36–15b)
Bc + v
A partir de las ecuaciones 36-14 y 36-15 se ve que la luz proveniente de una fuen-
te que se mueve hacia uno tendrá una frecuencia mayor y una longitud de onda más
corta, mientras que si una fuente de luz se aleja de uno, se verá una frecuencia menor
y una longitud de onda mayor. En el último caso, la luz visible tendrá su longitud de
onda prolongada hacia el extremo rojo del espectro visible (figura 32-26), un efecto
que se llama corrimiento hacia el rojo. Como se verá en el siguiente capítulo, todos los
átomos tienen su propia firma distintiva en términos de las frecuencias de la luz que
emiten. En 1929 el astrónomo estadounidense Edwin Hubble (1889-1953) descubrió
que la radiación de los átomos en muchas galaxias se recorre hacia el rojo. Esto es, las
frecuencias de la luz que emiten son menores que las que emiten átomos estacionarios
en la Tierra, lo que sugiere que las galaxias se alejan de la Tierra. Éste es el origen de
la idea de que el Universo se expande.
*SECCIÓN 36–12 979
EJEMPLO 36–13 Pasarse un alto a gran velocidad. Un conductor afirma que
él en realidad no se pasó el alto porque la luz tenía corrimiento Doppler y parecía
verde. Calcule la rapidez que debe llevar un conductor para que la luz roja parezca verde.
PLANTEAMIENTO Aplique la ecuación de corrimiento Doppler para luz roja (l0 L
650 nm) y luz verde (l L 500 nm).
SOLUCIÓN La ecuación 36-14a se sostiene para la fuente y el objeto que se acercan
entre sí:

c - v.
l = l0
Bc + v

Eleve al cuadrado esta ecuación:

c - v l 2
= ¢ ≤
c + v l0

donde (l兾l0)2 = (500 nm兾650 nm)2 = 0.59. Despeje v:

v = cc d = 0.26c.
1 - (l兾l0)2
1 + (l兾l0)2

Con esta defensa, el conductor no sería culpable de pasarse un alto, pero sin duda se-
ría culpable de rebasar el límite de rapidez.

36–13 El impacto de la relatividad


especial
Para poner a prueba las predicciones de la teoría especial de la relatividad se han rea-
lizado muchos experimentos. Dentro del error experimental, no se han encontrado
contradicciones. Por lo tanto, los científicos aceptan la relatividad como una descrip-
ción exacta de la naturaleza.
A rapidez mucho menor que la rapidez de la luz, las fórmulas relativistas se redu-
cen a las clásicas, como se estudió. Desde luego, se esperaría (más bien, se insistiría)
que esto sea cierto, pues la mecánica newtoniana funciona tan bien para los objetos que
se mueven con rapidez v V c. Esta insistencia de que una teoría más general (como la
relatividad) proporcione los mismos resultados que una teoría más restringida (como
la mecánica clásica que funciona para v V c) se llama principio de correspondencia. Las
dos teorías deben corresponder donde sus ámbitos de validez se traslapen. De esta for-
ma, la relatividad no contradice a la mecánica clásica. Más bien, es una teoría más ge-
neral, de la cual ahora la mecánica clásica se considera un caso límite.
La importancia de la relatividad no reside simplemente en que ofrece resultados
más exactos, en especial en los casos de rapidez muy alta. Mucho más que eso, cambió
la forma como se ve el mundo. Los conceptos de espacio y tiempo ahora se ven como
relativos, y mutuamente entremezclados, mientras que antes se consideraban absolutos
y separados. Incluso los conceptos de materia y energía cambiaron: una se puede con-
vertir en la otra. El impacto de la relatividad se extiende mucho más allá de la física.
Influyó en otras ciencias, e incluso en el mundo del arte y la literatura; de hecho, entró
a la cultura general.
Desde un punto de vista práctico, en la vida diaria no se tiene mucha oportunidad
para usar las matemáticas de la relatividad. Por ejemplo, el factor g, 1兾31 - v2兾c2 ,
que aparece en las fórmulas relativistas, tiene un valor de sólo 1.005 incluso para una
rapidez tan alta como 0.10c 5 3.0 3 107 mys, que da una corrección de menos del 1%.
Para valores de rapidez menores que 0.10c, a menos que masa y energía se intercam-
bien, generalmente no se necesita usar fórmulas relativistas más complicadas y se pue-
den usar las fórmulas clásicas más sencillas.

980 CAPÍTULO 36 La teoría especial de la relatividad


Resumen
Un marco de referencia inercial es aquél donde se cumple la ley de Las transformaciones de Lorentz relacionan las posiciones y los
Newton de la inercia. Los marcos de referencia inerciales se pueden tiempos de los eventos en un marco de referencia inercial con sus
mover a velocidad constante unos en relación con otros; los marcos posiciones y tiempos en un segundo marco de referencia inercial.
de referencia en aceleración son no inerciales. x = g(x¿ + vt ¿)
La teoría especial de la relatividad se basa en dos principios: el y = y¿
principio de relatividad, que afirma que las leyes de la física son las z = z¿ (36–6)
mismas en todos los marcos de referencia inerciales, y el principio vx¿
de constancia de la rapidez de la luz, que afirma que la rapidez de la t = g ¢ t¿ + 2 ≤
luz en el espacio vacío tiene el mismo valor en todos los marcos de c
referencia inerciales. donde g = 1兾31 - v2兾c2 .
Una consecuencia de la teoría de la relatividad es que dos even- La suma de velocidad también se puede realizar en una forma
tos que son simultáneos en un marco de referencia pueden no ser si- especial. Todos estos efectos relativistas son significativos sólo a alta
multáneos en otro. Otros efectos son la dilatación del tiempo: de rapidez, cercana a la rapidez de la luz, que en sí misma es la rapidez
acuerdo con las mediciones, los relojes en movimiento funcionan más última en el Universo.
lentamente; y la contracción de la longitud: según las mediciones, la La cantidad de movimiento de un objeto está dada por
longitud de un objeto en movimiento es más corta (en su dirección de mv .
p = gmv = (36–8)
movimiento) que cuando está en reposo. Cuantitativamente,
31 - v 兾c
2 2

¢ t0 Masa y energía son interconvertibles. La ecuación


¢t = = g ¢ t0 (36–1) E = mc2 (36–12)
31 - v 兾c
2 2
nos indica cuánta energía E se necesita para crear una masa m, o vi-
l0 ceversa. Dicho de otra forma, E 5 mc2 es la cantidad de energía que
l = l031 - v2兾c2 = g (36–3) tiene un objeto en función de su masa m. La ley de conservación de
la energía debe incluir la masa como forma de energía.
donde l y Dt son la longitud y el intervalo de tiempo de los objetos La energía cinética K de un objeto en movimiento con rapidez
(o eventos) observados conforme se mueven con rapidez v; l0 y Dt0 v está dada por
son la longitud propia y el tiempo propio, esto es, las mismas canti- mc2
dades medidas en el marco en reposo de los objetos o eventos. La K = (g - 1)mc2 = - mc2 (36–10)
31 - v 兾c
2 2
cantidad g es una abreviatura de
donde m es la masa del objeto. La energía total E, si no hay energía
1 . potencial, es
g = (36–2)
31 - v 兾c
2 2
E = K + mc2
(36–11)
La teoría de la relatividad cambió las nociones de espacio y = gmc2.
tiempo, así como de cantidad de movimiento, energía y masa. Espa- La cantidad de movimiento p de un objeto se relaciona con su
cio y tiempo se ven como íntimamente conectados; el tiempo es una energía total E (suponiendo que no hay energía potencial) mediante
cuarta dimensión, además de las tres dimensiones del espacio. E 2 = p2c2 + m2c4. (36–13)

Preguntas
1. Usted está en un vagón sin ventanas en un tren excepcional- 7. ¿La dilatación del tiempo significa que el tiempo en realidad
mente suave que viaja con velocidad constante. ¿Existe algún transcurre más lentamente en los marcos de referencia en mo-
experimento físico que pueda realizar en el vagón para deter- vimiento o que sólo parece transcurrir más lentamente?
minar si está en movimiento? Explique. 8. Una joven mujer astronauta acaba de llegar a casa de un largo
2. Es posible que usted haya tenido la experiencia de estar detenido viaje. Corre hacia un hombre de cabellos grises y, mientras en-
ante un semáforo en alto cuando, por el rabillo del ojo, ve que el tablan conversación, se refiere a él como su hijo. ¿Cómo es po-
auto en el carril de junto se arrastra hacia delante. De manera ins- sible esto?
tintiva, pisa el pedal del freno y piensa que usted es quien rueda ha-
cia atrás. ¿Qué dice esto acerca del movimiento absoluto y relativo? 9. Si usted viaja alejándose de la Tierra con rapidez 0.5c, ¿notaría
3. Un trabajador está de pie en lo alto de un vagón de ferrocarril un cambio en su ritmo cardiaco? ¿Su masa, peso o cintura se
en movimiento, y lanza una pesada bola en línea recta hacia modificarían? ¿Qué dirían de usted los observadores en la Tie-
arriba (desde su punto de vista). Ignorando la resistencia del ai- rra que usen telescopios?
re, ¿la bola regresará directo a su mano o caerá detrás de él? 10. ¿La dilatación del tiempo y la contracción de la longitud ocu-
4. ¿La Tierra realmente gira alrededor del Sol? ¿O también es vá- rren a rapidez ordinaria, por ejemplo, a 90 kmyh?
lido decir que el Sol gira alrededor de la Tierra? Discuta, en tér- 11. Suponga que la rapidez de la luz fuera infinita. ¿Qué ocurriría a
minos del principio de relatividad (que no hay un mejor marco las predicciones relativistas de contracción de la longitud y dila-
de referencia). Explique. tación del tiempo?
5. Si usted va en una nave espacial que viaja a 0.5c alejándose de
una estrella, ¿a qué rapidez lo pasaría la luz de la estrella? 12. Discuta cómo la vida cotidiana sería diferente si la rapidez de la
6. El efecto de dilatación del tiempo a veces se expresa como “los luz fuera de sólo 25 mys.
relojes en movimiento corren más lentamente”. En realidad, es- 13. Explique cómo se pueden usar las fórmulas de contracción de
te efecto no tiene nada que ver con el movimiento que afecte el la longitud y de dilatación del tiempo para indicar que c es la
funcionamiento de los relojes. Entonces, ¿a qué se refiere? rapidez límite del Universo.

Preguntas 981
14. El dibujo al principio del capítulo presenta la calle como la ve 15. Un electrón está limitado a viajar a rapideces menores que c.
el señor Tompkins, donde la rapidez de la luz es c 5 20 miyh. ¿Esto pone un límite superior a la cantidad de movimiento de un
¿Cómo ven al señor Tomp- electrón? Si es así, ¿cuál es este límite superior? Si no, explique.
kins las personas que es-
16. ¿Una partícula con masa distinta de cero puede lograr la rapi-
tán de pie en la calle (fi-
dez de la luz?
gura 36-15)? Explique.
17. ¿La ecuación E 5 mc2 está en conflicto con el principio de con-
servación de la energía? Explique.
18. Si la masa es una forma de energía, ¿significa esto que un resorte
FIGURA 36–15 tiene más masa cuando se comprime que cuando está relajado?
Pregunta 14. El señor 19. No es correcto decir que “la materia no se crea ni se destruye”.
Tompkins visto por la ¿Qué se debe decir en vez de ello?
gente en la acera. Vea
también la figura al 20. ¿La noción intuitiva de que las velocidades simplemente se su-
inicio del capítulo, en la man, como se hizo en la sección 3-9, es completamente errónea?
página 951.

Problemas
36–5 y 36–6 Dilatación del tiempo, contracción 1.00%? (Ésta es una forma razonable de estimar cuándo hacer
de la longitud cálculos relativistas en lugar de clásicos).
12. (II) Cierta estrella se localiza a 18.6 años luz de distancia.
1. (I) Una nave espacial pasa junto a usted con una rapidez de ¿Cuánto le tomaría a una nave espacial viajar a 0.950c para lle-
0.850c. Usted mide su longitud en 38.2 m. ¿Cuál será su longi- gar a esa estrella desde la Tierra, según las mediciones de obser-
tud cuando esté en reposo? vadores: a) en la Tierra, b) en la nave espacial? c) ¿Cuál es la
2. (I) Cierto tipo de partícula elemental viaja con una rapidez de distancia recorrida de acuerdo con observadores en la nave es-
2.70 3 108 mys. A esta rapidez, la vida promedio se mide en 4.76 pacial? d) ¿Qué rapidez calcularán los ocupantes de la nave
3 1026 s. ¿Cuál es el tiempo de vida de la partícula en reposo? espacial a partir de los resultados de b) y c)?
3. (II) De acuerdo con la teoría especial de la relatividad, el factor 13. (II) Suponga que un reporte noticioso afirma que la nave espa-
g que determina la contracción de la longitud y la dilatación del cial Enterprise acaba de regresar de un viaje de 5 años en el que
tiempo está dado por g = 1兾31 - v2兾c2 . Determine los va- registró una rapidez de 0.74c. a) Si el reporte quiere dar a en-
lores numéricos de g para un objeto que se mueve con rapidez tender 5.0 años de tiempo en la Tierra, ¿cuánto tiempo transcu-
v = 0.01c, 0.05c, 0.10c, 0.20c, 0.30c, 0.40c, 0.50c, 0.60c, 0.70c, rrió en la nave? b) Si el reporte significa 5.0 años de tiempo en
0.80c, 0.90c y 0.99c. Elabore una gráfica de g contra v. la nave, ¿cuánto tiempo transcurrió en la Tierra?
14. (II) Una partícula inestable producida en un experimento con ace-
4. (II) Si viajara a una estrella a 135 años luz de la Tierra, con una
8 lerador, viaja con velocidad constante, y recorre 1.00 m en 3.40 ns
rapidez de 2.80 3 10 mys, ¿cuánto mediría esta distancia?
en el marco del laboratorio, antes de “decaer” (convertirse) en
5. (II) ¿Cuál es la rapidez de un pión si su vida media se mide en otras partículas. En el marco en reposo de la partícula, determine
4.40 3 1028 s? En reposo, su vida media es de 2.60 3 1028 s. a) cuánto vivió antes de decaer, b) cuánto recorrió antes de decaer.
6. (II) En un marco de referencia terrestre, una estrella está a 56 15. (II) Cuando cierta partícula subatómica se encuentra estaciona-
años luz de distancia. ¿Con qué rapidez tendría que viajar una ria, su vida media es T0. Esto es, si en cierto momento hay N de
persona de manera que para ella la distancia sólo sea de 35 estas partículas, entonces un tiempo T0 después sólo habrá Ny2
años luz? partículas, suponiendo que las partículas están en reposo. En la
7. (II) Suponga que decide viajar a una estrella a 65 años luz de posición x 5 0 en un laboratorio de física de alta energía se crea
distancia con una rapidez que le indica que la distancia sólo es un haz que porta N de tales partículas por segundo. Este haz via-
de 25 años luz. ¿Cuántos años tardaría en realizar el viaje? ja a lo largo del eje x con rapidez v en el marco de referencia del
laboratorio, y se descubre que sólo Ny2 partículas por segundo
8. (II) ¿A qué rapidez v la longitud de una barra de 1.00 m pare- viajan en el haz en x 5 2cT0, donde c es la rapidez de la luz. En-
cería un 10.0% más corta (es decir, de 90.0 cm)?
cuentre la rapidez v de las partículas dentro del haz.
9. (II) La velocidad de escape de la Tierra es de 11.2 kmys. ¿Cuál 16. (II) En su propio marco de referencia, una caja tiene la forma
sería la disminución porcentual en longitud de una nave espa- de un cubo de 2.0 m de lado. Esta caja se carga sobre el piso
cial de 65.2 m de largo que viaja a esa rapidez, vista desde la plano de una nave espacial y entonces la nave espacial vuela y
Tierra? pasa junto a usted con una rapidez horizontal de 0.80c. ¿Cuál es
10. (II) Un amigo pasa rápidamente junto a usted en su nave espa- el volumen de la caja según la observa usted?
cial, con una rapidez de 0.760c. En el marco de referencia de us- 17. (II) Cuando está en reposo, una nave espacial tiene la forma de
ted, la nave mide 4.80 m de largo por 1.35 m de alto. a) ¿Cuáles un triángulo isósceles cuyos dos lados iguales tienen longitud 2l
serán su longitud y altura en reposo? b) ¿Cuántos segundos y cuya base tiene longitud l. Si esta nave vuela y pasa junto a un
transcurren en el reloj de su amigo cuando para usted pasaron observador con una velocidad relativa de v 5 0.95c dirigida a lo
20.0 s? c) ¿Con qué rapidez parece viajar usted de acuerdo con largo de su base, ¿cuáles son las longitudes de los tres lados de
su amigo? d) ¿Cuántos segundos transcurrieron en el reloj de la nave, de acuerdo con el observador?
usted cuando su amigo vio en su reloj que pasaron 20.0 s? 18. (II) ¿Con qué rapidez debe moverse un pión, en promedio, para
11. (II) ¿A qué rapidez las fórmulas relativistas para a) longitud y recorrer 25 m antes de decaer? La vida media, en reposo, es de
b) intervalos de tiempo difieren de los valores clásicos en 2.6 3 1028 s.

982 CAPÍTULO 36 La teoría especial de la relatividad


36–8 Transformaciones de Lorentz 30. (III) En el Viejo Oeste, un comisario en un tren que viaja a 35.0
mys ve un duelo entre dos hombres de pie en tierra, separados
19. (I) Un observador en Tierra ve que una nave extraterrestre se
55.0 m en forma paralela al tren. Los instrumentos del comisa-
aproxima con una rapidez de 0.60c. El Enterprise viene al resca-
rio indican que, en su marco de referencia, los dos hombres dis-
te (figura 36-16) y rebasa a los extraterrestres mientras se des-
paran simultáneamente. a) ¿Cuál de los dos hombres, el
plaza directamente hacia la Tierra con una rapidez de 0.90c
primero que pasa el tren (A) o el segundo (B) debe ser arresta-
relativa a la Tierra. ¿Cuál es la rapidez relativa de una nave vis-
do por disparar el primer tiro? Esto es, en el marco de referen-
ta por los tripulantes de la otra?
cia de los pistoleros, ¿quién disparó primero? b) ¿Con cuánta
Enterprise
anticipación disparó? c) ¿Quién recibió primero el disparo?
31. (III) Dos bombillas, A y B, se colocan en reposo sobre el eje x
= 0.90c en las posiciones xA 5 0 y xB 5 1l. En este marco de referen-
cia, las bombillas se encienden simultáneamente. Con base en
= 0.60c
las transformaciones de Lorentz, encuentre una expresión para
el intervalo de tiempo entre que las bombillas se encienden, se-
gún las mediciones de un observador que se desplaza con velo-
FIGURA 36–16 Problema 19. cidad v en la dirección 1x. De acuerdo con este observador,
20. (I) Suponga que en la figura 36-11 los orígenes de S y S9 se tras- ¿cuál bombilla se encendió primero?
lapan en t 5 t9 5 0, y que S9 se mueve con rapidez v 5 30 mys 32. (III) Un observador en un marco de referencia S nota que dos
con respecto a S. En S9, una persona descansa en un punto cu- eventos están separados en el espacio por 220 m y en el tiempo
yas coordenadas son x9 5 25 m, y9 5 20 m y z9 5 0. Calcule las por 0.80 ms. ¿Con qué rapidez se debe mover el marco de referen-
coordenadas de esta persona en S (x, y, z) en a) t 5 3.5 s, b) t 5 cia S9, en relación con S, para que un observador en S9 detecte
10.0 s. Use la transformación galileana. que los dos eventos ocurren en la misma ubicación en el espacio?
21. (I) Repita el problema 20 con la transformación de Lorentz y 33. (III) Un granjero que estudia física cree que puede encajar una
una rapidez relativa v 5 1.80 3 108 mys, pero elija el tiempo t pértiga de 12.0 m de largo en un granero de 10.0 m de largo, si co-
que es a) 3.5 ms y b) 10.0 ms. rre suficientemente rápido llevando la pértiga. ¿Lo logrará? Expli-
22. (II) En el problema 21, suponga que la persona se mueve con que en detalle. ¿Cómo coincide esto con la idea de que, cuando el
una velocidad cuyas componentes son u9x 5 u9y 5 1.10 3 108 granjero corre, el granero parece todavía más corto que 10.0 m?
mys, ¿Cuál será su velocidad con respecto a S? (Indique magni-
tud y dirección). 36–9 Cantidad de movimiento relativista
23. (II) Dos naves espaciales abandonan la Tierra en sentidos 34. (I) ¿Cuál es la cantidad de movimiento de un protón que viaja
opuestos, cada una con una rapidez de 0.60c con respecto a la a v 5 0.75c?
Tierra. a) ¿Cuál es la velocidad de la nave espacial 1 en relación 35. (II) a) Una partícula viaja a v 5 0.10c. ¿En qué porcentaje
con la nave espacial 2? b) ¿Cuál es la velocidad de la nave espa- el cálculo de su cantidad de movimiento será erróneo si usa la
cial 2 en relación con la nave espacial 1? fórmula clásica? b) Repita el ejercicio para v 5 0.60c.
24. (II) El marco de referencia S9 se mueve con rapidez v 5 0.92c en 36. (II) Una partícula de masa m viaja con una rapidez v 5 0.26c.
la dirección 1x con respecto al marco de referencia S. Los oríge- ¿A qué rapidez su cantidad de movimiento será el doble?
nes de S y S9 se traslapan en t 5 t9 5 0. Un objeto se encuentra es- 37. (II) Una partícula inestable está en reposo y súbitamente decae
tacionario en S9 en la posición x9 5 100 m. ¿Cuál es la posición del en dos fragmentos. Ninguna fuerza externa actúa sobre la par-
objeto en S cuando el reloj en S registra 1.00 ms de acuerdo con las tícula o sus fragmentos. Uno de los fragmentos tiene una rapi-
ecuaciones a) galileanas y b) de transformación de Lorentz? dez de 0.60c y una masa de 6.68 3 10227 kg, mientras que la
25. (II) Una nave espacial abandona la Tierra y viaja a 0.61c. Una otra tiene una masa de 1.67 3 10227 kg. ¿Cuál es la rapidez del
segunda nave espacial sale de la primera nave con una rapidez fragmento menos masivo?
de 0.87c con respecto a la primera. Calcule la rapidez de la se- 38. (II) ¿Cuál es el cambio porcentual en cantidad de movimiento
gunda nave con respecto a Tierra si se lanza a) en la misma di- de un protón que acelera a) desde 0.45c hasta 0.80c, b) de 0.80c
rección en la que viaja la primera nave, b) directamente hacia a 0.98c?
atrás, hacia la Tierra.
26. (II) Su nave espacial, que viaja a 0.90c, necesita lanzar una son- 36–11 Energía relativista
da por la escotilla frontal, de manera que su rapidez relativa 39. (I) Calcule la energía en reposo de un electrón en joules y en
con el planeta al que se aproxima sea de 0.95c. ¿Con qué rapi- MeV (1 MeV 5 1.60 3 10213 J).
dez debe la sonda abandonar su nave? 40. (I) Cuando un núcleo de uranio en reposo se descompone en el
27. (II) Una nave espacial que viaja a 0.76c alejándose de la Tierra proceso conocido como fisión en un reactor nuclear, los frag-
lanza un módulo con una rapidez de 0.82c en ángulos rectos con mentos resultantes tienen una energía cinética total de aproxi-
su propia dirección de viaje (visto por la nave espacial). ¿Cuál es madamente 200 MeV. ¿Cuánta masa se pierde en el proceso?
la rapidez del módulo y su dirección de viaje (relativa a la direc- 41. (I) El consumo anual total de energía en Estados Unidos es de
ción de la nave espacial), vista por un observador en Tierra? aproximadamente 8 3 1019 J. ¿Cuánta masa tendría que conver-
28. (II) Si una partícula se mueve en el plano xy del sistema S (fi- tirse a energía para satisfacer esta necesidad?
gura 36-11) con rapidez u en una dirección que forma un ángu- 42. (I) Calcule la masa de un protón en MeVyc2.
lo u con el eje x, demuestre que forma un ángulo u9 en S9 dado 43. (II) Suponga que hay un proceso mediante el cual dos fotones,
por tan u¿ = (sen u)31 - v2兾c2兾(cos u - v兾u). cada uno con cantidad de movimiento de 0.50 MeVyc, pudieran
29. (II) Una barra de longitud l0, en reposo en el marco de referen- chocar y formar una sola partícula. ¿Cuál es la masa máxima
cia S, forma un ángulo u con el eje x. En el marco de referencia que podría poseer la partícula?
S9, que se mueve hacia la derecha con velocidad v = vî con res-
B
44. (II) a) ¿Cuánto trabajo se requiere para acelerar un protón des-
pecto a S, determine a) la longitud l de la barra y b) el ángulo de el reposo hasta una rapidez de 0.998c? b) ¿Cuál sería la can-
u9 que forma con el eje x9. tidad de movimiento de este protón?

Problemas 983
45. (II) ¿Cuánta energía se puede obtener de la conversión de 1.0 63. (III) a) En el marco de referencia S, una partícula tiene canti-
dad de movimiento p = pxî a lo largo del eje positivo x. De-
B
gramo de masa? ¿Cuánta masa podría elevar esta energía a una
altura de 1.0 km sobre la superficie de la Tierra? muestre que, en el marco S9, que se mueve con rapidez v, como
46. (II) Para acelerar una partícula de masa m desde el reposo has- en la figura 36-11, la cantidad de movimiento tiene componentes
ta una rapidez de 0.90c se requiere trabajo W1. Para acelerar la px - vE兾c2
partícula desde la rapidez 0.90c hasta 0.99c se requiere trabajo pxœ =
31 - v 兾c
2 2
W2. Determine la razón W2yW1.
47. (II) ¿Cuál es la rapidez de una partícula cuando su energía ci- pyœ = py
nética es igual a su energía en reposo? pzœ = pz
48. (II) ¿Cuál es la cantidad de movimiento de un protón de 950 E - px v
E¿ = .
MeV (esto es, su energía cinética es de 950 MeV)?
31 - v 兾c
2 2
49. (II) Calcule la energía cinética y la cantidad de movimiento de
un protón que viaja a 2.80 3 108 mys. (Estas ecuaciones de transformación se sostienen, en realidad,
B
para cualquier dirección de p, en tanto el movimiento de S9 sea
50. (II) ¿Cuál es la rapidez de un electrón cuya energía cinética es
a lo largo del eje x). b) Demuestre que px, py, pz, Eyc se trans-
de 1.25 MeV?
forman de acuerdo con la transformación de Lorentz al igual
51. (II) ¿Cuál es la rapidez de un protón acelerado por una dife- que x, y, z, ct.
rencia de potencial de 1.25 MeV?
52. (II) Dos partículas idénticas de masa m se aproximan una a otra 36–12 Corrimiento Doppler para la luz
con igual velocidad v, pero en sentido opuesto. La colisión es 64. (II) Cierta galaxia tiene un corrimiento Doppler dado por
completamente inelástica y da por resultado una sola partícula f0 - f = 0.0987f0. Estime con qué rapidez se aleja de la Tierra.
en reposo. ¿Cuál es la masa de la nueva partícula? ¿Cuánta 65. (II) Una nave espacial que se dirige hacia la Tierra a 0.70c
energía se perdió en la colisión? ¿Cuánta energía cinética se transmite señales de radio a 95.0 MHz. ¿A qué frecuencia se
perdió en esta colisión? deben sintonizar los receptores en la Tierra?
53. (II) ¿Cuál es la rapidez de un electrón justo antes de que incida 66. (II) A partir de la ecuación 36-15a, demuestre que el corrimien-
sobre una pantalla de televisión luego de acelerarse desde el re- to Doppler en longitud de onda es
poso mediante los 28,000 V del cinescopio?
54. (II) La energía cinética de una partícula es de 45 MeV. Si la ¢l v
=
cantidad de movimiento es de 121 MeVyc, ¿cuál es la masa de l c
la partícula? si v V c.
55. (II) Calcule la rapidez de un protón (m 5 1.67 3 10227 kg) cu- 67. (III) Una “pistola” de radar emite microondas de frecuencia f0
ya energía cinética es exactamente la mitad a) de su energía to- 5 36.0 GHz. Cuando la pistola se apunta a un objeto que se di-
tal, b) de su energía en reposo. rige hacia ella con rapidez v, el objeto recibe las microondas en
56. (II) Calcule la energía cinética y la cantidad de movimiento de la frecuencia con corrimiento Doppler f. El objeto en movi-
un protón (m 5 1.67 3 10227 kg) que viaja a 8.15 3 107 mys. miento refleja estas microondas con esta misma frecuencia f. El
¿Cuáles serían los errores porcentuales si se emplearan las aparato de radar estacionario detecta estas ondas reflejadas co-
fórmulas clásicas? mo una frecuencia con corrimiento Doppler f9. La pistola com-
57. (II) Suponga que una nave espacial de 17,000 kg de masa se acele- bina su onda emitida a f0 y su onda detectada en f9. Estas
ra a 0.18c. a) ¿Cuánta energía cinética tendría? b) Si usó la fórmu- ondas interfieren, lo que crea un patrón de pulsos cuya frecuen-
la clásica para energía cinética, ¿cuál sería el error porcentual? cia de pulso es fpulso 5 f9 _ f0. a) Demuestre que
* 58. (II) ¿Qué campo magnético B se necesita para mantener a cfpulso
protones de 998 GeV dando vueltas en un círculo de 1.0 km de v L ,
radio (en el sincrotrón del Fermilab, por ejemplo)? Use la ma- 2f0
sa relativista. La masa en reposo del protón es 0.938 GeVyc2. si fpulso V f0. Si fpulso 5 6670 Hz, ¿cuál es v (en kmyh)? b) Si la
(1 GeV 5 109 eV.) [Sugerencia: En relatividad, mrel v2兾r = qvB rapidez del objeto es diferente por Dv, demuestre que la dife-
todavía es válida en un campo magnético, donde mrel 5 gm]. rencia en frecuencia de pulsos Dfpulso está dada por
59. (II) El núcleo de americio, 24195Am, decae a un núcleo de neptu- 2f0 ¢v
nio, 237
93Np, al emitir una partícula alfa de 4.00260 u de masa y ¢fpulso = .
energía cinética de 5.5 MeV. Estime la masa del núcleo de nep-
c
tunio e ignore su retroceso, dado que la masa del americio es de Si la precisión de la pistola es de 1 kmyh, ¿a que precisión se
241.05682 u. debe medir la frecuencia de pulso?
60. (II) Elabore una gráfica de la energía cinética frente a cantidad 68. (III) Cierto átomo emite luz de frecuencia f0 cuando está en re-
de movimiento para a) una partícula de masa distinta de cero y poso. Un gas monoatómico compuesto de estos átomos se en-
b) una partícula con masa cero. cuentra a la temperatura T. Algunos de los átomos del gas se
acercan a un observador y otros se alejan de él en virtud de su
61. (II) Un muón negativo, que viaja al 43% de la rapidez de la luz,
movimiento térmico aleatorio. Con la rapidez rms del movi-
choca de manera frontal con un muón positivo que viaja al 55%
miento térmico, demuestre que la diferencia fraccional entre las
de la rapidez de la luz. Los dos muones (cada uno con masa de
frecuencias con corrimiento Doppler para átomos que se acer-
105.7 MeVyc2) se aniquilan; ¿cuánta energía electromagnética
can directamente hacia el observador y los que se alejan direc-
producen?
tamente del observador es ¢f兾f0 L 223kT兾mc2; suponga
62. (II) Demuestre que la energía cinética K de una partícula de
masa m se relaciona con su cantidad de movimiento p median- que mc2 W 3kT. Evalúe Dfyf0 para un gas de átomos de hi-
te la ecuación drógeno a 550 K. [Este efecto de “ensanchamiento Doppler”
comúnmente se usa para medir la temperatura de los gases, co-
p = 3K2 + 2Kmc2兾c. mo en astronomía].

984 CAPÍTULO 36 La teoría especial de la relatividad


Problemas generales
69. Un reloj atómico se lleva al polo norte, mientras otro permane- 78. Como regla general, cualquier objeto que viaje más rápido que
ce en el ecuador. ¿Cuánto estarán fuera de sincronización des- aproximadamente 0.1c se llama relativista; esto es, la relatividad
pués de transcurridos 2.0 años? [Sugerencia: Use el desarrollo especial es un efecto significativo. Determine la rapidez de un
binomial, Apéndice A]. electrón en un átomo de hidrógeno (radio: 0.53 3 10210 m) e in-
70. Una nave espacial en apuros envía dos cápsulas de escape dique si es o no relativista. (Considere al electrón como si estu-
en sentidos opuestos. Una viaja con una rapidez v1 5 20.60c en un viera en una órbita circular alrededor del protón).
sentido, y la otra viaja con una rapidez v2 5 10.50c en el sentido 79. ¿Qué cantidad mínima de energía electromagnética se necesita
opuesto, según se observa desde la nave espacial. ¿Qué rapidez para producir un electrón y un positrón juntos? Un positrón es
lleva la segunda cápsula de acuerdo con las mediciones de la una partícula con la misma masa que un electrón, pero con car-
primera cápsula de escape? ga opuesta. (Advierta que la carga eléctrica se conserva en este
71. Un avión viaja a 1300 kmyh alrededor de la Tierra en un círcu- proceso. Véase la sección 37-5).
lo de radio esencialmente igual al de la Tierra, y regresa al mis-
mo lugar. Con la relatividad especial, estime la diferencia en 80. ¿Cuántos gramos de materia tendrían que destruirse totalmente
tiempo para hacer el viaje, según observadores en Tierra y en el para mantener encendida una bombilla de 75 W durante 1.0 año?
avión. [Sugerencia: Use el desarrollo binomial, Apéndice A.] 81. Si E es la energía total de una partícula con energía potencial
72. La estrella más cercana a la Tierra es Próxima Centauri, a 4.3 cero, demuestre que dEydp 5 v, donde p y v son la cantidad de
años luz de distancia. a) ¿A qué velocidad constante debe viajar movimiento y la velocidad de la partícula, respectivamente.
una nave espacial desde la Tierra, si debe llegar a la estrella en 82. Un neutrón libre puede decaer en un protón, un electrón y un
4.6 años, según los viajeros en la nave espacial? b) ¿Cuánto tar- neutrino. Suponga que la masa del neutrino es cero; las otras
da el viaje de acuerdo con observadores en Tierra? masas se pueden encontrar en la tabla de la segunda de forros.
73. Un quasar emite líneas de hidrógeno cuyas longitudes de onda Determine la energía cinética total compartida entre las tres
son 7.0% más largas que lo que se mide en el laboratorio. a) Con partículas cuando un neutrón decae en reposo.
la fórmula Doppler para la luz, determine la rapidez de este
quasar. b) ¿Qué resultado obtendría si empleara el corrimiento
83. El Sol irradia energía a una tasa aproximada de 4 3 1026 W.
Doppler “clásico” que estudió en el capítulo 16? a) ¿A qué tasa disminuye la masa del Sol? b) ¿Cuánto tarda el
Sol en perder una masa igual a la de la Tierra? c) Estime cuán-
74. El corazón sano de un astronauta tiene un ritmo cardiaco de 60
to podría durar el Sol si irradiara constantemente a esa tasa.
latidosyminuto. Los médicos del vuelo que permanecen en Tie-
rra pueden monitorear los signos vitales del astronauta a dis- 84. Según las mediciones, una partícula desconocida tiene carga ne-
tancia durante el vuelo. ¿Con qué rapidez tendría que alejarse gativa y una rapidez de 2.24 3 108 mys. Su cantidad de movi-
un astronauta de la Tierra para que el médico registrara que su miento se determina en 3.07 3 10222 kg?mys. Identifique la
ritmo cardiaco es de 30 latidosymin? partícula al encontrar su masa.
75. Una nave espacial (marco de referencia S9) pasa junto a la Tie- 85. ¿Cuánta energía se requeriría para descomponer un núcleo de
B
rra (marco de referencia S) con velocidad v, que apunta a lo helio en sus constituyentes, dos protones y dos neutrones? Las
largo de los ejes x y x9. La nave espacial emite un haz de luz masas en reposo de un protón (incluyendo la masa de un elec-
(rapidez c) a lo largo de su trón), un neutrón y helio neutro son, respectivamente, 1.00783
eje y9, como se muestra en la y ¢ u, 1.00867 u y 4.00260 u. (Esta diferencia de energía se llama
figura 36-17. a) ¿Qué ángulo S¢ energía de enlace total del núcleo de 42He).
u forma este haz de luz con el
eje x en el marco de referen- 86. Demuestre analíticamente que una partícula con cantidad de
cia de la Tierra? b) Use trans- movimiento p y energía E tiene una rapidez dada por
cB = cjˆ
formaciones de velocidad para
demostrar que la luz se des- x¢ vB pc2 pc
v = = .
plaza con rapidez c también E 3m c + p
2 2 2
en el marco de referencia de y
la Tierra. c) Compare estos Tierra
resultados relativistas con el S 87. Dos protones, cada uno con una rapidez de 0.985c en el labora-
que se obtendría de manera torio, se acercan entre sí. Determine a) la cantidad de movi-
clásica (transformaciones ga- cB = ? miento de cada protón en el laboratorio, b) la cantidad de
movimiento total de los dos protones en el laboratorio y c) la
lileanas). FIGURA 36–17 u
x cantidad de movimiento de un protón desde la perspectiva del
Problema 75.
otro protón.
76. El cohete A pasa junto a la Tierra con una rapidez de 0.65c. Al 88. Cuando dos moles de moléculas de hidrógeno (H2) y un mol de
mismo tiempo, el cohete B pasa junto a la Tierra y se desplaza a moléculas de oxígeno (O2) reaccionan para formar dos moles
0.85c en relación con la Tierra en la misma dirección. ¿Con qué de agua (H2O), la energía liberada es 484 kJ. ¿Cuánto disminu-
rapidez viaja B en relación con A cuando rebasa a A? ye la masa en esta reacción? ¿Qué porcentaje de la masa origi-
77. a) ¿Cuál es la rapidez v de un electrón cuya energía cinética es nal total del sistema representa este cambio de masa?
14,000 veces su energía en reposo? Puede enunciar la respuesta 89. La nave espacial ficticia Enterprise obtiene su potencia al com-
como la diferencia c 2 v. Tal rapidez se alcanzó en el Acelera- binar materia y antimateria, y logra la conversión completa de
dor Lineal Stanford, SLAC. b) Si los electrones viajan en el la- masa en energía. Si la masa del Enterprise es aproximadamente
boratorio a través de un tubo de 3.0 km de largo (como en el de 6 3 109 kg, ¿cuánta masa se debe convertir en energía ciné-
SLAC), ¿de qué longitud es este tubo en el marco de referencia tica para acelerarla desde el reposo hasta un décimo de la rapi-
de los electrones? [Sugerencia: Utilice el desarrollo binomial]. dez de la luz?

Problemas generales 985


90. Una nave espacial y sus ocupantes tienen una masa total de 98. Una losa de vidrio, con índice de refracción n, se desplaza ha-
180,000 kg. A los ocupantes les gustaría viajar a una estrella que cia la derecha con rapidez v. Un haz de luz se emite en el pun-
está a 35 años luz de distancia, con una rapidez de 0.70c. Para to A (figura 36-18), pasa a través del vidrio y llega al punto B
acelerar, el motor de la nave espacial convierte masa directa- a una distancia l. El vidrio tiene grosor d en el marco de refe-
mente en energía. ¿Cuánta masa se convertirá en energía para rencia donde está en reposo, y la rapidez de la luz en el vidrio
acelerar la nave espacial a esta rapidez? Suponga que la nave es cyn. ¿Cuánto tarda la luz en ir del punto A al punto B, de
se acelera rápidamente, de manera que la rapidez de todo el acuerdo con un observador en reposo con respecto a los pun-
viaje se puede considerar como 0.70c, e ignore la disminución tos A y B? Com-
en masa total para el cálculo. ¿Cuánto tardará el viaje de acuer- pruebe su respuesta
vidrio
do con los astronautas a bordo? para los casos v 5
91. En una reacción nuclear se crean dos partículas idénticas, que c, v 5 0 y n 5 1. B
v
viajan en sentidos opuestos. Si la rapidez de cada partícula es A B
0.85c, relativa al marco de referencia del laboratorio, ¿cuál es la
rapidez de una partícula en relación con la otra partícula?
92. Una nave espacial de 32,000 kg viajará a la vecindad de una es- FIGURA 36–18
trella a 6.6 años luz de distancia de la Tierra. Los pasajeros en la Problema 98. l
nave quieren que el viaje (de ida) no tarde más de 1.0 año.
¿Cuánto trabajo se debe realizar sobre la nave para llevarla a la
rapidez necesaria para este viaje?
* Problemas numéricos/por computadora
93. Suponga que una nave espacial de 14,500 kg abandona la Tierra
con una rapidez de 0.98c. ¿Cuál es la energía cinética de la na- * 99. (II) Para una masa de 1.0 kg, elabore una gráfica de la energía
ve espacial? Compare con el consumo anual total de energía en cinética como función de la rapidez para valores de 0 a 0.9c,
Estados Unidos (aproximadamente 1020 J). tanto con la fórmula clásica AK = 12 mv2 B como con la fórmula
94. Un mesón pi de masa mp decae en reposo en un muón (masa relativista correcta AK = (g - 1)mc2 B.
mm) y un neutrino de masa despreciable o cero. Demuestre que
* 100. (III) Una partícula de masa m se proyecta horizontalmente a una
la energía cinética del muón es Km = Amp - mm B 2c2兾 A2mp B . rapidez relativista v0 en la dirección 1x. Existe una fuerza descen-
95. Los astrónomos miden la distancia a una estrella particular en dente constante F que actúa sobre la partícula. Con la definición
de cantidad deB movimiento relativista p = gmv y la segunda
B B
6.0 años luz (1 año luz 5 distancia que recorre la luz en un
ley de Newton F = dp兾dt, a) demuestre que las componentes x y
B
año). Una nave espacial viaja de la Tierra a la vecindad de esta
estrella con rapidez constante, y llega en 2.50 años, según los re- y de velocidad de la partícula en el tiempo t están dadas por
lojes en la nave espacial. a) ¿Cuánto tarda el viaje, según los relo-
1
jes en el marco de referencia de la Tierra (que se supone vx(t) = p0 c兾Am2c2 + p20 + F 2t2 B 2
inercial)? b) ¿Qué distancia recorre la nave espacial, según las 1
mediciones en su propio marco de referencia? vy(t) = –Fct兾Am2c2 + p20 + F 2t2 B 2
96. Una masa de 1.88 kg oscila en el extremo de un resorte cuya donde p0 es la cantidad de movimiento inicial de la partícula.
constante de rigidez es k 5 84.2 Nym. Si este sistema se encuen- b) Suponga que la partícula es un electrón (m 5 9.11 3 10231
tra en una nave espacial que pasa junto a la Tierra a 0.900c, kg), con v0 5 0.50c y F 5 1.00 3 10215 N. Calcule los valores
¿cuál es su periodo de oscilación de acuerdo con a) observado- de vx y vy del electrón como función del tiempo t, desde t 5 0
res en la nave y b) observadores en Tierra? hasta t 5 5.00 ms, en intervalos de 0.05 ms. Grafique los valores
97. Demuestre que el intervalo espacio-tiempo, (c ¢ t)2 - (¢x)2, para mostrar cómo cambian con el tiempo las componentes de
es invariante, lo cual significa que todos los observadores en to- velocidad durante este intervalo. c) ¿La trayectoria es parabó-
dos los marcos de referencia inerciales calculan el mismo núme- lica, como lo sería en la mecánica clásica (secciones 3-7 y 3-8)?
ro para esta cantidad en el caso de cualquier par de eventos. Explique.

Respuestas a los ejercicios


A: Sí. E: 0.030c, igual que clásica, a una precisión mejor que 0.1%.
B: c). F: d).
C: a) No; b) sí. G: d).
D: 80 m. H: No.

986 CAPÍTULO 36 La teoría especial de la relatividad


CHAPTER 36: The Special Theory of Relativity

Responses to Questions
1. No. The train is an inertial reference frame, and the laws of physics are the same in all inertial
reference frames, so there is no experiment you can perform inside the train car to determine if you
are moving.

2. The fact that you instinctively think you are moving is consistent with the relativity principle applied
to mechanics. Even though you are at rest relative to the ground, when the car next to you creeps
forward, you are moving backward relative to that car.

3. As long as the railroad car is traveling with a constant velocity, the ball will land back in his hand.

4. The relativity principle refers only to inertial reference frames. Neither the reference frame of the
Earth nor the reference frame of the Sun is inertial. Either reference frame is valid, but the laws of
physics will not be the same in each of the frames.

5. The starlight would pass at c, regardless of your spaceship’s speed. This is consistent with the
second postulate of relativity which states that the speed of light through empty space is independent
of the speed of the source or the observer.

6. It deals with space-time (sometimes called “the fabric of space-time”) and the actual passage of time
in the reference frame, not with the mechanical workings of clocks. Any measurement of time
(heartbeats or decay rates, for instance) would be measured as slower than normal when viewed by
an observer outside the moving reference frame.

7. Time actually passes more slowly in the moving reference frames, according to observers outside
the moving frames.

8. This situation is an example of the “twin paradox” applied to parent-child instead of to twins. This
might be possible if the woman was traveling at high enough speeds during her trip. Time would
have passed more slowly for her and she could have aged less than her son, who stayed on Earth.
(Note that the situations of the woman and son are not symmetric; she must undergo acceleration
during her journey.)

9. No, you would not notice any change in your heartbeat, mass, height, or waistline, because you are
in the inertial frame of the spaceship. Observers on Earth, however, would report that your heartbeat
is slower and your mass greater than if you were at rest with respect to them. Your height and
waistline will depend on your orientation with respect to the motion. If you are “standing up” in the
spaceship such that your height is perpendicular to the direction of travel, then your height would not
change but your waistline would shrink. If you happened to be “lying down” so that your body is
parallel to the direction of motion when the Earth observers peer through the telescope, then you
would appear shorter but your waistline would not change.

10. Yes. However, at a speed of only 90 km/hr, v/c is very small, and therefore γ is very close to one, so
the effects would not be noticeable.

© 2009 Pearson Education, Inc., Upper Saddle River, NJ. All rights reserved. This material is protected under all copyright laws as they
currently exist. No portion of this material may be reproduced, in any form or by any means, without permission in writing from the publisher.
427
Physics for Scientists & Engineers with Modern Physics, 4th Edition Instructor Solutions Manual

11. Length contraction and time dilation would not occur. If the speed of light were infinite, v/c would
be zero for all finite values of v, and therefore γ would always be one, resulting in t  t0 and
l  l0 .

12. The effects of special relativity, such as time dilation and length contraction, would be noticeable in
our everyday activities because everyday speeds would no longer be so small compared to the speed
of light. There would be no “absolute time” on which we would all agree, so it would be more
difficult, for instance, to plan to meet friends for lunch at a certain time! In addition, 25 m/s would
be the limiting speed and nothing in the universe would move faster than that.

13. Both the length contraction and time dilation formulas include the term 1  v 2 c 2 . If c were not
the limiting speed in the universe, then it would be possible to have a situation with v > c. However,
this would result in a negative number under the square root, which gives an imaginary number as a
result, indicating that c must be the limiting speed.

14. Mr. Tompkins appears shrunk in the horizontal direction, since that is the direction of his motion,
and normal size in the vertical direction, perpendicular to his direction of motion. This length
contraction is a result of the fact that, to the people on the sidewalk, Mr. Tompkins is in a moving
frame of reference. If the speed of light were only 20 mi/h, then the amount of contraction, which
depends on γ, would be enough to be noticeable. Therefore, Mr. Tompkins and his bicycle appear
very skinny. (Compare to the chapter-opening figure, which is shown from Mr. Tompkin’s
viewpoint. In this case, Mr. Tompkins sees himself as “normal” but all the objects moving with
respect to him are contracted.)
mv
15. No. The relativistic momentum of the electron is given by p   mv  . At low speeds
1  v2 c2
(compared to c) this reduces to the classical momentum, p = mv. As v approaches c, γ approaches
infinity so there is no upper limit to the electron’s momentum.

16. No. To accelerate a particle with nonzero rest mass up to the speed of light would require an infinite
amount of energy, and so is not possible.

17. No. E = mc² does not conflict with the principle of conservation of energy as long as it is understood
that mass is a form of energy.

18. Yes, mass is a form of energy so technically it is correct to say that a spring has more mass when
compressed. However, the change in mass of the spring is very small and essentially negligible.

19. “Energy can be neither created nor destroyed.” Mass is a form of energy, and mass can be
“destroyed” when it is converted to other forms of energy. The total amount of energy remains
constant.

20. Technically yes, the notion that velocities simply add is wrong. However, at everyday speeds, the
relativistic equations reduce to classical ones, so our ideas about velocity addition are essentially
true for velocities that are low compared to the speed of light.

© 2009 Pearson Education, Inc., Upper Saddle River, NJ. All rights reserved. This material is protected under all copyright laws as they
currently exist. No portion of this material may be reproduced, in any form or by any means, without permission in writing from the publisher.
428
Chapter 36 The Special Theory of Relativity

Solutions to Problems
1. You measure the contracted length. Find the rest length from Eq. 36-3a.
l 38.2 m
l0    72.5m
1 v c 1   0.850 
2 2 2

2. We find the lifetime at rest from Eq. 36-1a.


2
 2.70  108 m s
t0  t 1  v c   4.76  10 s 
2 2 6
1  6
  2.07  10 s
 3.00  10 m
8
s

3. The numerical values and


8
graph were generated in a
spreadsheet. The graph is 7
shown also. The spreadsheet 6
used for this problem can be
5
found on the Media Manager,
with filename 4
“PSE4_ISM_CH36.XLS,” on 3
tab “Problem 36.3.”
2

1
0 0.2 0.4 0.6 0.8 1
v /c

4. The measured distance is the contracted length. Use Eq. 36-3a.


2
 2.80  108 m s 
l  l 0 1  v c  135 ly 
2 2
1    48.5 ly
 3.00  10 m s 
8

5. The speed is determined from the time dilation relationship, Eq. 36-1a.
t0  t 1  v 2 c 2 
2 2
 t   2.60  108 s 
v  c 1  0   c 1  8 
 0.807c  2.42  108 m s
 t   4.40  10 s 

6. The speed is determined from the length contraction relationship, Eq. 36-3a.
2 2
 l   35ly 
l  l0 1  v 2 c2  v  c 1     c 1     0.78c  2.3  10 m s
8

 l0   56ly 

7. The speed is determined from the length contraction relationship, Eq. 36-3a. Then the time is found
from the speed and the contracted distance.
l  l0 1  v 2 c2 

© 2009 Pearson Education, Inc., Upper Saddle River, NJ. All rights reserved. This material is protected under all copyright laws as they
currently exist. No portion of this material may be reproduced, in any form or by any means, without permission in writing from the publisher.
429
Physics for Scientists & Engineers with Modern Physics, 4th Edition Instructor Solutions Manual

 25y  c 
2
 l l l 25ly
v  c 1   ; t    27 y
 l0  v  l 
2
 25ly 
2 c  0.923
c 1   c 1  
 l0   65ly 

8. The speed is determined from the length contraction relationship, Eq. 36-3a.
2
 l
l  l 0 1  v 2 c 2  v  c 1     c 1   0.900   0.436 c
2

l
 0

9. The change in length is determined from the length contraction relationship, Eq. 36-3a. The speed is
very small compared to the speed of light.
l  l0 1  v 2 c2 
1/ 2 2
l  v2  v2  11.2  103 m s 10
 1  v 2 c2  1  2   1  2  1  12 
1
2   1  6.97  10
 3.00  10 m
8
l0  c  c s
So the percent decrease is  6.97  10  % .
8

10. (a) The measured length is the contracted length. We find the rest length from Eq. 36-3a.
l 4.80 m
l0    7.39 m
1 v c 1   0.760 
2 2 2

Distances perpendicular to the motion do not change, so the rest height is 1.35m .
(b) The time in the spacecraft is the rest time, found from Eq. 36-1a.
t0  t 1  v 2 c 2   20.0s  1   0.760   13.0s
2

(c) To your friend, you moved at the same relative speed: 0.760 c .
(d) She would measure the same time dilation: 13.0s .

11. (a) We use Eq. 36-3a for length contraction with the contracted length 99.0% of the rest length.
2
 l 
l  l 0 1  v c  v  c 1     c 1   0.990   0.141 c
2 2 2

 l0 
(b) We use Eq. 36-1a for time dilation with the time as measured from a relative moving frame
1.00% greater than the rest time.
2 2
 t   1 
t0  t 1  v c  v  c 1   0   c 1  
2 2
  0.140 c
 t   1.0100 
We see that a speed of 0.14 c results in about a 1% relativistic effect.

12. (a) To an observer on Earth, 18.6 ly is the rest length, so the time will be the distance divided by
the speed.
l
tEarth  0 
18.6 ly   19.58 yr  19.6 yr
v 0.950 c
(b) The time as observed on the spacecraft is shorter. Use Eq. 36-1a.
t0  t 1  v 2 c 2  19.58 yr  1   0.950   6.114 yr  6.11yr
2

© 2009 Pearson Education, Inc., Upper Saddle River, NJ. All rights reserved. This material is protected under all copyright laws as they
currently exist. No portion of this material may be reproduced, in any form or by any means, without permission in writing from the publisher.
430
Chapter 36 The Special Theory of Relativity

(c) To the spacecraft observer, the distance to the star is contracted. Use Eq. 36-3a.
l  l 0 1  v 2 c 2  18.6 ly  1   0.950   5.808 ly  5.81 ly
2

(d) To the spacecraft observer, the speed of the spacecraft is their observed distance divided by
their observed time.
v
l

 5.808 ly   0.950 c
t0 6.114 yr

13. (a) In the Earth frame, the clock on the Enterprise will run slower. Use Eq. 36-1a.
t0  t 1  v 2 c 2   5.0 yr  1   0.74   3.4 yr
2

(b) Now we assume the 5.0 years is the time as measured on the Enterprise. Again use Eq. 36-1a.
t0  t 1  v 2 c 2  t 
t 0

 5.0 yr   7.4 yr
1  v2 c2 1   0.74 
2

14. We find the speed of the particle in the lab frame, and use that to find the rest frame lifetime and
distance.
x 1.00 m
v  lab   2.941  108 m s  0.9803 c
tlab 3.40  109 s
(a) Find the rest frame lifetime from Eq. 36-1a.
t0  tlab 1  v 2 c 2   3.40  109 s  1   0.9803  6.72  1010 s
2

(b) In its rest frame, the particle will travel the distance given by its speed and the rest lifetime.
x0  vt0   2.941  108 m s  6.72  1010 s   0.198m
xlab
This could also be found from the length contraction relationship: x0  .
1  v 2 c2

15. Since the number of particles passing per second is reduced from N to N / 2, a time T0 must have
elapsed in the particles’ rest frame. The time T elapsed in the lab frame will be greater, according to
Eq. 36-1a. The particles moved a distance of 2cT0 in the lab frame during that time.
T0 x 2cT0
T0  T 1  v 2 c 2  T  ; v   v  45 c  0.894 c
1 v c
2 2 T T0
1  v 2 c2

16. The dimension along the direction of motion is contracted, and the other two dimensions are
unchanged. Use Eq. 36-3a to find the contracted length.
l  l0 1  v 2 c2 ; V  l  l0    l0  1  v 2 c 2   2.0 m  1   0.80   4.8m 3
2 3 3 2

17. The vertical dimensions of the ship will not change, but the horizontal dimensions will be contracted
according to Eq. 36-3a. The base will be contracted as follows.
l base  l 1  v 2 c 2  l 1   0.95  0.31l
2

0.50 l
When at rest, the angle of the sides with respect to the base is given by   cos 1  75.52.
2.0l
The vertical component of l vert  2 l sin   2 l sin 75.52  1.936l is unchanged. The horizontal

© 2009 Pearson Education, Inc., Upper Saddle River, NJ. All rights reserved. This material is protected under all copyright laws as they
currently exist. No portion of this material may be reproduced, in any form or by any means, without permission in writing from the publisher.
431
Physics for Scientists & Engineers with Modern Physics, 4th Edition Instructor Solutions Manual

component, which is 2 l cos   2 l  14   0.50l at rest, will be contracted in the same way as the base.
l horizontal  0.50l 1  v 2 c 2  0.50l 1   0.95  0.156l
2

Use the Pythagorean theorem to find the length of the leg.


 0.156l   1.936l   1.942 l  1.94 l
2 2
l leg  l 2horizontal  l 2vert 

18. In the Earth frame, the average lifetime of the pion will be dilated according to Eq. 36-1a. The speed
of the pion will be the distance moved in the Earth frame times the dilated time.
d d
v  1  v2 c2 
t t0
1 1
vc c  0.95 c
  3.00  10 m s  2.6  108 s  
2 2
 ct0  8
1   1  
 d    25m  
 

19. We take the positive direction in the direction of the Enterprise. Consider the alien vessel as
reference frame S, and the Earth as reference frame S. The velocity of the Earth relative to the alien
vessel is v  0.60 c. The velocity of the Enterprise relative to the Earth is ux  0.90 c. Solve for
the velocity of the Enterprise relative to the alien vessel, ux , using Eq. 36-7a.

ux 
 ux  v    0.90c  0.60c   0.65c
 vux  1    0.60  0.90  
1  2  
 c 
We could also have made the Enterprise as reference frame S, with v  0.90 c, and the velocity of
the alien vessel relative to the Earth as ux  0.60 c. The same answer would result.
Choosing the two spacecraft as the two reference frames would also work. Let the alien vessel be
reference frame S, and the Enterprise be reference frame S. Then we have the velocity of the Earth
relative to the alien vessel as ux  0.60 c, and the velocity of the Earth relative to the Enterprise as
ux  0.90 c. We solve for v, the velocity of the Enterprise relative to the alien vessel.

ux 
 ux  v   v  ux  ux   .60c    0.90c   0.65c
 vux   ux u x    0.90c  .60c  
1  2  1  2  1  
 c   c   c2 

20. The Galilean transformation is given in Eq. 36-4.


(a)  x, y , z    x   vt , y , z   25m   30 m s  3.5s  ,20 m,0   130 m,20 m,0 

(b)  x, y, z    x  vt , y , z   25m   30 m s 10.0s  ,20 m,0    325m,20 m,0 

21. (a) The person’s coordinates in S are found using Eq. 36-6, with x  25 m , y  20 m , z   0 , and
t   3.5 s. We set v  1.80  108 m/s.
x   vt  25m  1.8  108 m/s   3.5 s 
x   820 m
1  v 2 c2 1  1.8  10 m/s   3.0  10 m/s 
8 2 8 2

y  y   20 m ; z  z   0

© 2009 Pearson Education, Inc., Upper Saddle River, NJ. All rights reserved. This material is protected under all copyright laws as they
currently exist. No portion of this material may be reproduced, in any form or by any means, without permission in writing from the publisher.
432
Chapter 36 The Special Theory of Relativity

(b) We repeat part (a) using the time t   10.0 s.


x   vt  25m  1.8  108 m/s  10.0 s 
x   2280 m
1  v 2 c2 1  1.8  108 m/s   3.0  108 m/s 
2 2

y  y   20 m ; z  z   0

22. We determine the components of her velocity in the S frame using Eq. 36-7, where
ux  u y  1.10  108 m/s and v  1.80  108 m/s . Then using trigonometry we combine the
components to determine the magnitude and direction.
ux  v 1.10  108 m/s  1.80  108 m/s
ux    2.38  108 m/s
1  vux / c 2 1  1.80  108 m/s 1.10  108 m/s  /  3.00  108 m/s 2

uy 1  v 2 c 2 1.10  10 m/s  1  1.8  10 m/s   3.0  10 m/s 


8 8 8 2 2

uy    7.21  107 m/s


1  vux / c 1  1.80  10 m/s 1.10  10 m/s  /  3.00  10 m/s 
2 8 8 8 2

 2.38  10 m/s    7.21  10 m/s 


2 2
u  ux2  u 2y  8 7
 2.49  108 m/s
uy 7.21  107 m/s
  tan 1  tan 1  16.9
ux 2.38  108 m/s

23. (a) We take the positive direction to be the direction of motion of spaceship 1. Consider spaceship
2 as reference frame S, and the Earth reference frame S. The velocity of the Earth relative to
spaceship 2 is v  0.60 c. The velocity of spaceship 1 relative to the Earth is ux  0.60 c. Solve
for the velocity of spaceship 1 relative to spaceship 2, u x , using Eq. 36-7a.

ux 
 ux  v    0.60 c  0.60 c   0.88 c
 vux  1   0.60  0.60 
1  2 
 c 
(b) Now consider spaceship 1 as reference frame S. The velocity of the Earth relative to spaceship
1 is v  0.60 c. The velocity of spaceship 2 relative to the Earth is ux  0.60 c. Solve for the
velocity of spaceship 2 relative to spaceship 1, u x , using Eq. 36-7a.

ux 
 ux  v    0.60 c  0.60 c   0.88 c
 vux  1    0.60  0.60 
1  2 
 c 
As expected, the two relative velocities are the opposite of each other.

24. (a) The Galilean transformation is given in Eq. 36-4.


x  x   vt   x  vt  100 m   0.92   3.00  108 m s 1.00  106 s   376 m
(b) The Lorentz transformation is given in Eq. 36-6. Note that we are given t, the clock reading in
frame S.
 vx   t vx
t    t  2   t   2
 c   c
  t vx    v  ct vx  
x    x   vt     x   v   2      x    
   c   c  c 

© 2009 Pearson Education, Inc., Upper Saddle River, NJ. All rights reserved. This material is protected under all copyright laws as they
currently exist. No portion of this material may be reproduced, in any form or by any means, without permission in writing from the publisher.
433
Physics for Scientists & Engineers with Modern Physics, 4th Edition Instructor Solutions Manual


1  100 m    0.92 
1  0.92  2  1  0.922  3.00  108 m s 1.00  106 s   0.92 100 m  
 
 316 m

25. (a) We take the positive direction in the direction of the first spaceship. We choose reference frame
S as the Earth, and reference frame S as the first spaceship. So v  0.61 c. The speed of the
second spaceship relative to the first spaceship is ux  0.87 c. We use Eq. 36-7a to solve for the
speed of the second spaceship relative to the Earth, u.
ux 
 ux  v    0.87c  0.61c   0.97 c
 vux  1   0.61 0.87 
1  2 
 c 
(b) The only difference is now that ux   0.87 c.

ux 
 ux  v    0.87c  0.61c   0.55 c
 vux  1   0.61  0.87 
1  2 
 c 
The problem asks for the speed, which would be 0.55 c

26. We assume that the given speed of 0.90c is relative to the planet that you are approaching. We take
the positive direction in the direction that you are traveling. Consider your spaceship as reference
frame S, and the planet as reference frame S. The velocity of the planet relative to you is
v  0.90 c. The velocity of the probe relative to the planet is ux  0.95 c. Solve for the velocity of
the probe relative to your spaceship, ux , using Eq. 36-7a.

ux 
 ux  v    0.95c  0.90c   0.34c
 vux  1    0.90  0.95
1  2 
 c 

27. We set frame S as the frame at rest with the spaceship. In this frame the module has speed
u  uy  0.82c. Frame S is the frame that is stationary with respect to the Earth. The spaceship, and
therefore frame S moves in the x-direction with speed 0.76c in this frame, or v  0.76c. We use
Eq. 36-7a and 36-7b to determine the components of the module velocity in frame S. Then using
trigonometry we combine the components to determine the speed and direction of travel.
ux  v 0  0.76c uy 1  v 2 c 2 0.82c 1  0.762
ux    0.76 c ; u    0.533c
1  vux / c 2 1  vux / c 2
y
1 0 1 0
u 0.533c
u  ux2  u 2y   0.76c    0.533c   0.93c ;   tan 1 y  tan 1
2 2
 35
ux 0.76c

28. The velocity components of the particle in the S frame are ux  u cos and u y  u sin  . We find the
components of the particle in the S frame from the velocity transformations given in Eqs. 36-7a and
36-7b. Those transformations are for the S frame moving with speed v relative to the S frame. We
can find the transformations from the S frame to the S frame by simply changing v to –v and primed
to unprimed variables.

ux 
 ux  v   ux 
 ux  v  ; uy 
uy 1  v 2 c 2
 uy 
u y 1  v 2 c2
1  vu c 
x
2
1  vu x c2  1  vu x c2  1  vu x c2 
© 2009 Pearson Education, Inc., Upper Saddle River, NJ. All rights reserved. This material is protected under all copyright laws as they
currently exist. No portion of this material may be reproduced, in any form or by any means, without permission in writing from the publisher.
434
Chapter 36 The Special Theory of Relativity

u y 1  v 2 c2

tan   
uy

1  vu x c 2  u y 1  v 2 c 2 u sin  1  v 2 c 2
  
sin  1  v 2 c 2
ux  ux  v   ux  v   u cos  v   cos  v u 
1  vux c  2

29. (a) In frame S the horizontal component of the stick length will be contracted, while the vertical
component remains the same. We use the trigonometric relations to determine the x- and y-
components of the length of the stick. Then using Eq. 36-3a we determine the contracted length
of the x-component. Finally, we use the Pythagorean theorem to determine stick length in
frame S.
l x  l 0 cos  ; l y  l 0 sin   ly ; lx  l x 1  v 2 c 2  l 0 cos  1  v 2 c 2

l  lx 2  ly 2  l 0 2 cos2  1  v 2 c 2   l 0 2 sin 2   l 0 1   v cos c 


2

(b) We calculate the angle from the length components in the moving frame.
l  l 0 sin    tan  
   tan 1 y  tan 1    tan 1    tan 1   tan  
    
 l 0 cos  1  v c 
lx
 1 v c 
2 2 2 2

30. (a) We choose the train as frame S and the Earth as frame S. Since the guns fire simultaneously in
S , we set these times equal to zero, that is tA  tB  0. To simplify the problem we also set the
location of gunman A equal to zero in frame S when the guns were fired, xA  0. This places
gunman B at xB  55.0 m. Use Eq. 36-6 to determine the time that each gunman fired his
weapon in frame S.
 vx   v 0 
tA    tA  2A     0  2   0
 c   c 
 
 vx 
tB    tB  2B  
1  0   35m/s  55.0 m    2.14  1014 s
1   35.0 m/s 3.00  108 m/s 
  3.00  108 m/s  
2
 c  2

Therefore, in Frame S, A fired first.
(b) As found in part (a), the difference in time is 2.14  1014 s .
(c) In the Earth frame of reference, since A fired first, B was struck first. In the train frame, A is
moving away from the bullet fired toward him, and B is moving toward the bullet fired toward
him. Thus B will be struck first in this frame as well.

31. We set frame S as the frame moving with the observer. Frame S is the frame in which the two light
bulbs are at rest. Frame S is moving with velocity v with respect to frame S. We solve Eq. 36-6 for
the time t  in terms of t, x, and v. Using the resulting equation we determine the time in frame S that
each bulb is turned on, given that in frame S the bulbs are turned on simultaneously at tA  tB  0.
Taking the difference in these times gives the time interval as measured by the observing moving
with velocity v.
x
x    x   vt   x    vt 

 vx    v x   v 2  vx t  vx  vx 
t    t   2  =  t   2   vt      t   1  2   2   2  t    t  2 
 c   c    c  c  c  c 
© 2009 Pearson Education, Inc., Upper Saddle River, NJ. All rights reserved. This material is protected under all copyright laws as they
currently exist. No portion of this material may be reproduced, in any form or by any means, without permission in writing from the publisher.
435
Physics for Scientists & Engineers with Modern Physics, 4th Edition Instructor Solutions Manual

 vx   v 0   vx   vl  vl
t A    t A  2A     0  2   0 ; tB    tB  2B     0  c 2    c 2
 c   c   c   
vl
t   tB  t A  
c2
According to the observer, bulb B turned on first.

32. We set up the two frames such that in frame S, the first object is located at the origin and the second
object is located 220 meters from the origin, so xA  0 and xB  220 m. We set the time when event
A occurred equal to zero, so tA  0 and tB  0.80 s. We then set the location of the two events in
frame S equal, and using Eq. 36-6 we solve for the velocity.
x  xB 0  220 m
xA  xB    xA  vt A     xB  vtB  ; v  A   2.5  108 m/s
tA  tB 0  0.88 s

33. From the boy’s frame of reference, the pole remains at rest with respect to him. As such, the pole
will always remain 12.0 m long. As the boy runs toward the barn, relativity requires that the
(relatively moving) barn contract in size, making the barn even shorter than its rest length of 10.0 m.
Thus it is impossible, in the boy’s frame of reference, for the barn to be longer than the pole. So
according to the boy, the pole will never completely fit within the barn.
In the frame of reference at rest with respect to the barn, it is possible for the pole to be shorter than
the barn. We use Eq. 36-3a to calculate the speed that the boy would have to run for the contracted
length of the pole, l, to equal the length of the barn.
l  l 0 1  v 2 c 2  v  c 1  l 2 l 02  c 1  10.0 m  12.0 m 
2 2
 0.5528c
If persons standing at the front and back door of the barn were to close both doors exactly when the
pole was completely inside the barn, we would have two simultaneous events in the barn’s rest frame
S with the pole completely inside the barn. Let us set the time for these two events as tA  tB  0. In
frame S these two events occur at the front and far side of the barn, or at xA  0 and xB  10.0m.
Using Eq. 36-6, we calculate the times at which the barn doors close in the boy’s frame of reference.
 vx   v 0 
t A    t A  2A     0  2   0
 c   c 
 vx  1  0.5528 10.0 m  
tB    tB  2B   0 
8
  2.211  10 s
 c  1  0.5528 
2 3.00  10 8
m/s 
Therefore, in the boy’s frame of reference the far door of the barn closed 22.1 ns before the front
door. If we multiply the speed of the boy by this time difference, we calculate the distance the boy
traveled between the closing of the two doors.
x  vt  0.5528  3.00  108 m/s  2.211  108 s   3.67 m.
We use Eq. 36-3a to determine the length of the barn in the boy’s frame of reference.
l  l 0 1  v 2 c 2  10.0 m  1  0.55282  8.33 m
Subtracting the distance traveled between closing the doors from the length of the pole, we find the
length of the barn in the boy’s frame of reference.
l 0,pole  x  12.0 m  3.67 m  8.33 m  l barn
Therefore, in the boy’s frame of reference, when the front of the pole reached the far door it was
closed. Then 22.1 ns later, when the back of the pole reached the front door, that door was closed.
In the boy’s frame of reference these two events are not simultaneous.

© 2009 Pearson Education, Inc., Upper Saddle River, NJ. All rights reserved. This material is protected under all copyright laws as they
currently exist. No portion of this material may be reproduced, in any form or by any means, without permission in writing from the publisher.
436
Chapter 36 The Special Theory of Relativity

34. The momentum of the proton is given by Eq. 36-8.

p   mv 
mv

1.67  1027 kg   0.75  3.00  108 m s   5.7  1019 kg  m s
1  v2 c2 1   0.75
2

35. (a) We compare the classical momentum to the relativistic momentum.


p classical mv
 1  v 2 c 2  1   0.10   0.995
2

prelativistic mv
1  v2 c2
The classical momentum is about 0.5% in error.
(b) We again compare the two momenta.
p classical mv
 1  v 2 c 2  1   0.60   0.8
2

prelativistic mv
1  v2 c2
The classical momentum is 20% in error.

36. The momentum at the higher speed is to be twice the initial momentum. We designate the initial
state with a subscript “0”, and the final state with a subscript “f”.
mvf vf2
pf 1  vf c
2 2
1  vf2 c 2  vf2    0.26 c 2 
 2   4   2 
 4  2
 0.29c 2 
mv0 v02   1   0.26  
2
p0  1 v f c 
1  v02 c 2 1  v02 c 2
 0.29  2
vf2    c  vf  0.47 c
 1.29 

37. The two momenta, as measured in the frame in which the particle was initially at rest, will be equal
to each other in magnitude. The lighter particle is designated with a subscript “1”, and the heavier
particle with a subscript “2”.
m1v1 m2 v2
p1  p2   
1  v1 c
2 2
1  v22 c 2

v12  m2 
2
v22  6.68  1027 kg 
2
  0.60 c 2 
   2
 9.0c 2 
1  v12 c2   m1  1  v22 c2   1.67  1027 kg 
  
 1   0.60  
v1  0.90 c  0.95 c

38. We find the proton’s momenta using Eq. 36-8.


mp v1 m  0.45 c  mp v 2 m  0.80 c 
p0.45   p  0.5039mp c ; p0.80   p  1.3333mp c
v12    
2 2 2
1  0.45 v 1  0.80
1 2 1  22
c c
mp v2 m  0.98 c 
p0.98   p  4.9247mp c
 
2 2
v2 1  0.98
1 2
c

© 2009 Pearson Education, Inc., Upper Saddle River, NJ. All rights reserved. This material is protected under all copyright laws as they
currently exist. No portion of this material may be reproduced, in any form or by any means, without permission in writing from the publisher.
437
Physics for Scientists & Engineers with Modern Physics, 4th Edition Instructor Solutions Manual

 p  p1   1.3333mp c  0.5039mp c 
(a)  2 100   100  164.6  160%
 p1   0.5039mp c 
 p  p1   4.9247mp c  1.3333mp c 
(b)  2 100   100  269.4  270%
 p1   1.3333mp c 

39. The rest energy of the electron is given by Eq. 36-12.


E  mc 2   9.11  1031 kg  3.00  108 m s   8.20  10 14 J
2


8.20  10 14
J
 0.511MeV
1.60  10 13
J MeV 

40. We find the loss in mass from Eq. 36-12.


E  200 MeV  1.60  10 J MeV 
13

m 2   3.56  1028 kg  4  1028 kg


 3.00  10 m s 
2
c 8

41. We find the mass conversion from Eq. 36-12.


E
m 2 
 8  1019 J 
 900 kg
 3.00  108 m s 
2
c

42. We calculate the mass from Eq. 36-12.


1 1.6726  10 kg  2.9979  10 m s 
27 8 2
E 1
m  2  2  mc   2
2
 938.2 MeV c 2
c c c 1.6022  1013 J MeV 
43. Each photon has momentum 0.50 MeV/c. Thus each photon has mass 0.50 MeV. Assuming the
photons have opposite initial directions, then the total momentum is 0, and so the product mass will
not be moving. Thus all of the photon energy can be converted into the mass of the particle.
Accordingly, the heaviest particle would have a mass of 1.00 MeV c 2 , which is 1.78  1030 kg. .

44. (a) The work is the change in kinetic energy. Use Eq. 36-10b. The initial kinetic energy is 0.
 1 
W  K  K final    1 mc 2    1  938.3MeV   1.39  104 MeV
 1  0.998
2

 13.9GeV
(b) The momentum of the proton is given by Eq. 36-8.
1
p   mv   938.3MeV c2   0.998 c   1.48  104 MeV c  14.8GeV c
1  0.998 2

45. We find the energy equivalent of the mass from Eq. 36-12.
E  mc 2  1.0  103 kg  3.00  108 m s   9.0  1013 J
2

We assume that this energy is used to increase the gravitational potential energy.
E 9.0  1013 J
E  mgh  m    9.2  109 kg
hg 1.0  103 m  9.80 m s2 
© 2009 Pearson Education, Inc., Upper Saddle River, NJ. All rights reserved. This material is protected under all copyright laws as they
currently exist. No portion of this material may be reproduced, in any form or by any means, without permission in writing from the publisher.
438
Chapter 36 The Special Theory of Relativity

46. The work is the change in kinetic energy. Use Eq. 36-10b. The initial kinetic energy is 0.
W1   0.90  1 mc 2 ; W2  K 0.99 c  K 0.90c   0.99  1 mc 2   0.90  1 mc 2
1 1

W2  0.99  1 mc   0.90  1 mc  0.99   0.90
2 2
   1  0.99 1  0.902  3.7
2

W1  0.90  1 mc 2  0.90  1 1
1
1  0.902

47. The kinetic energy is given by Eq. 36-10.


1 3
K    1 mc 2  mc 2    2   v c  0.866 c
1 v c 2 2 4

48. The total energy of the proton is the kinetic energy plus the mass energy. Use Eq. 36-13 to find the
momentum.
E  K  mc 2 ;
 pc   E 2   mc 2    K  mc 2    mc 2   K 2  2 K  mc 2 
2 2 2 2

mc 2 938.3MeV
pc  K 2  2 K  mc 2   K 1  2   950 MeV  1  2  1638 MeV
K 950 MeV
p  1638 MeV c  1.6GeV c

49. We find the speed in terms of c. The kinetic energy is given by Eq. 36-10 and the momentum by Eq.
36-8.

v
 2.80  108 m s 
 0.9333 c
 3.00  108 m s 
 1 
K    1 mc 2    1  938.3MeV   1674.6 MeV  1.67GeV
 1  0.9333
2

1
p   mv   938.3MeV c2   0.9333 c   2439 MeV c  2.44GeV c
1  0.93332

50. We use Eq. 36-10 to find the speed from the kinetic energy.
 1 
K    1 mc 2    1 mc 2 
 1  v2 c2 
 
1 1
v  c 1 2
 c 1 2
 0.957 c
 K   1.25MeV 
 2  1  0.511MeV  1
 mc   

51. Since the proton was accelerated by a potential difference of 125 MV, its potential energy decreased
by 125 MeV, and so its kinetic energy increased from 0 to 125 MeV. Use Eq. 36-10 to find the
speed from the kinetic energy.
 1 
K    1 mc 2    1 mc 2 
 1  v2 c2 
 

© 2009 Pearson Education, Inc., Upper Saddle River, NJ. All rights reserved. This material is protected under all copyright laws as they
currently exist. No portion of this material may be reproduced, in any form or by any means, without permission in writing from the publisher.
439
Physics for Scientists & Engineers with Modern Physics, 4th Edition Instructor Solutions Manual

1 1
v  c 1 2
 c 1 2
 0.470 c
 K   125MeV 
 2  1  938.3MeV  1
 mc   

52. We let M represent the rest mass of the new particle. The initial energy is due to both incoming
particles, and the final energy is the rest energy of the new particle. Use Eq. 36-11 for the initial
energies.
2m
E  2  mc 2   Mc 2  M  2 m 
1  v 2 c2
We assumed that energy is conserved, and so there was no loss of energy in the collision.
The final kinetic energy is 0, so all of the kinetic energy was lost.
 1 
K lost  K initial  2   1 mc 2    1 2mc 2
 1  v 2 c2 
 

53. Since the electron was accelerated by a potential difference of 28 kV, its potential energy decreased
by 28 keV, and so its kinetic energy increased from 0 to 28 MeV. Use Eq. 36-10 to find the speed
from the kinetic energy.
 1 
K    1 mc 2    1 mc 2 
 1  v 2 c2 
 
1 1
v  c 1 2
 c 1 2
 0.32 c
 K   0.028 MeV 
 2  1  0.511MeV  1
 mc   

54. We use Eqs. 36-11 and 36-13 in order to find the mass.
E 2  p 2 c 2  m 2 c 4   K  mc 2   K 2  2 Kmc 2  m 2c 4 
2

p 2 c 2  K 2 121MeV c  c   45MeV 
2 2 2

m   140 MeV c 2  2.5  1028 kg


2 Kc 2 2  45MeV  c 2
The particle is most likely a probably a  0 meson.

55. (a) Since the kinetic energy is half the total energy, and the total energy is the kinetic energy plus
the rest energy, the kinetic energy must be equal to the rest energy. We also use Eq. 36-10.
K  12 E  12  K  mc 2   K  mc 2
1
K    1 mc 2  mc 2    2   v  43 c  0.866 c
1  v2 c2
(b) In this case, the kinetic energy is half the rest energy.
1
K    1 mc 2  12 mc 2    23   v  95 c  0.745 c
1  v 2 c2

© 2009 Pearson Education, Inc., Upper Saddle River, NJ. All rights reserved. This material is protected under all copyright laws as they
currently exist. No portion of this material may be reproduced, in any form or by any means, without permission in writing from the publisher.
440
Chapter 36 The Special Theory of Relativity

56. We use Eq. 36-10 for the kinetic energy and Eq. 36-8 for the momentum.
 
 
 1   1 
K    1 mc 2    1 mc 2    1  938.3MeV 
 1 v c
2 2  
2

   8.15  107 m s 
 1    
 3.00  10 m s 
8
 
 36.7 MeV
 8.15  107 m s 
 938.3MeV   
1 mc  v c  1  3.00  10 m s   265MeV c
2 8
mv
p   mv   
1 v c2 2 c 1  v 2 c2 c  8.15  107 m s 
2

1  
 3.00  10 m s 
8

Evaluate with the classical expressions.


2 2
v  8.15  107 m s 
K c  12 mv 2  12 mc 2    1
2  938.3MeV    34.6 MeV
 3.00  10 m s 
8
c
1 v  8.15  107 m s 
pc  mv  mc 2     938.3MeV     255MeV c
 3.00  10 m s 
8
c c
Calculate the percent error.
K K 34.6  36.7
errorK  c  100   100  5.7%
K 36.7
p p 255  265
errorp  c  100   100  3.8%
p 265

57. (a) The kinetic energy is found from Eq. 36-10.


 1   1 
K    1 mc     1 1.7  104 kg  3.00  108 m s 
2
 1 mc 2  
 1  v2 c2   1  0.18
2

 
 2.541  1019 J  2.5  1019 J
(b) Use the classical expression and compare the two results.
1.7  10 kg   0.18  3.00  10 m s 
2
K  12 mv   1
2
4 8
 2.479  1019 J

% error 
 2.479  10 J    2.541  10 J  100  2.4%
19 19

 2.541  10 J  19

The classical value is 2.4% too low.

58. The kinetic energy of 998 GeV is used to find the speed of the protons. Since the energy is 1000
times the rest mass, we expect the speed to be very close to c. Use Eq. 36-10.
 1 
K    1 mc 2    1 mc 2 
 1  v2 c2 
 
1 1
v  c 1 2
 c 1 2
 c  to 7 sig. fig. 
 K   998GeV 
 2  1  0.938GeV  1
 mc   

© 2009 Pearson Education, Inc., Upper Saddle River, NJ. All rights reserved. This material is protected under all copyright laws as they
currently exist. No portion of this material may be reproduced, in any form or by any means, without permission in writing from the publisher.
441
Physics for Scientists & Engineers with Modern Physics, 4th Edition Instructor Solutions Manual

 998GeV 
 K


 1 mc   1 1.673  1027 kg  3.00  108 m s 
 mv  mv  mc
2 2
 0.938GeV 
B     3.3T
rqv rq rq 1.0  103 m 1.60  1019 C 
59. By conservation of energy, the rest energy of the americium nucleus is equal to the rest energies of
the other particles plus the kinetic energy of the alpha particle.
mAm c 2   mNp  m  c 2  K 
K 5.5MeV  1u 
mNp  mAm  m  2
 241.05682 u  4.00260 u  2  2 
 237.04832 u
c c  931.49 MeV c 

60. (a) For a particle of non-zero mass, we derive the following relationship between kinetic energy
and momentum.
 pc   E 2   mc 2    K  mc 2    mc 2   K 2  2 K  mc 2 
2 2 2 2
E  K  mc 2 ;

2mc 2  4  mc 2   4  pc 
2 2

K  2 K  mc    pc  2
2 2
0  K 
2
For the kinetic energy to be positive, we take the positive root.
2mc 2  4  mc 2   4  pc 
2 2

K
2
 mc 2   mc    pc 
2 2 2

If the momentum is large, we have the following relationship.


K  mc 2   mc    pc 
2 2 2
 pc  mc 2
Thus there should be a linear relationship between kinetic energy and momentum for large
values of momentum.
If the momentum is small, we use the binomial expansion to derive the classical relationship.
2
 pc 
K  mc  2
 mc    pc 
2 2 2
 mc  mc
2 2
1  2 
 mc 
 1  pc  2  p 2
  mc  mc  1  2  2   
2 2
  mc   2m

Thus we expect a quadratic relationship for
m0
small values of momentum. The adjacent
graph verifies these approximations. m0
K

(b) For a particle of zero mass, the relationship is


simply K  pc. See the included graph. The
spreadsheet used for this problem can be
found on the Media Manager, with filename
“PSE4_ISM_CH36.XLS,” on tab “Problem
36.60.” p

© 2009 Pearson Education, Inc., Upper Saddle River, NJ. All rights reserved. This material is protected under all copyright laws as they
currently exist. No portion of this material may be reproduced, in any form or by any means, without permission in writing from the publisher.
442
Chapter 36 The Special Theory of Relativity

61. All of the energy, both rest energy and kinetic energy, becomes electromagnetic energy. We use Eq.
36-11. Both masses are the same.
 1 1 
Etotal  E1  E2   1mc 2   2 mc 2   1   2  mc 2     105.7 MeV 
 1  0.43 1  0.552 
2

 243.6 MeV  240 MeV

62. We use Eqs. 36-11 and 36-13.


 pc   E 2   mc 2    K  mc 2    mc 2   K 2  2 K  mc 2  
2 2 2 2
E  K  mc 2 ;

K 2  2 K  mc 2 
p
c

63. (a) We assume the mass of the particle is m, and we are given that the velocity only has an x-
component, u x . We write the momentum in each frame using Eq. 36-8, and we use the velocity
transformation given in Eq. 36-7. Note that there are three relevant velocities: ux , the velocity
in reference frame S; ux , the velocity in reference frame S; and v, the velocity of one frame
relative to the other frame. There is no velocity in the y or z directions, in either frame. We
1
reserve the symbol  for , and also use Eq. 36-11 for energy.
1  v 2 c2
mu x
px  ; p y  0 ; pz  0
1  ux2 c 2
ux  v ux  v 1  vux c 2 1  vux c 2
ux   ux  ; u   u  0 ; u   u 0
1  vux c 2
y y z z
1  vu x c 2 1  v 2 c2 1  v 2 c2
mux
px  ; py  0  since uy  0  ; pz  0  since uz  0 
1  ux2 c 2
Substitute the expression for ux into the expression for px .

m
 ux  v 
px 
mux

 1  vux c 2 
m
 ux  v  1
1  ux c
2 2
1  ux  v 
2
1  vux c2   ux  v 
2

1 2
c 1  vu c 2 2 1  vu x c  
2 2
c 2

1  vu  1  vu c2 
x
2 2 2
x c x

m
 ux  v  1

m  ux  v 
1  vu c 2
 1 u  v 
2
u  v 
2

1  vu 
2 2
1  vu 
2 2
x
c  x c  x
1  vux c2  x
c2
x
c2
m  ux  v  m  ux  v 
 
2 2
vu  vu  u 2 2u v v 2  vu  u
2
v2
1  2 2x   2x   2x  2x  2 1   2x   2x  2
c  c  c c c  c  c c

© 2009 Pearson Education, Inc., Upper Saddle River, NJ. All rights reserved. This material is protected under all copyright laws as they
currently exist. No portion of this material may be reproduced, in any form or by any means, without permission in writing from the publisher.
443
Physics for Scientists & Engineers with Modern Physics, 4th Edition Instructor Solutions Manual

mu x mv


m  ux  v 

1  u 2
x c2  1  u 2
x c2 

1  v 2
c 2 1  ux2 c 2  1  v 2
c 
2

mux mc 2 v mc 2 v
 px 

1  u 2
x c2  1  u 2
x c2  c2

1  u 2
x c2  c2

p x  vE c 2
1  v c  2 2
1  v 2
c2  1  v 2
c2 

It is obvious from the first few equations of the problem that py  p y   0  and pz  pz   0  .
mc 2 mc 2 mc 2
E   
1  ux2 c 2 1  ux  v   ux  v 
2 2

1
c 2 1  vu c 2 2 1  vu x c  
2 2
c 2

1  vu  1  vu c2 
x
2 2 2
x c x

mc 2 mvux


mc 1  vux c
2 2
 
 mc 2
 mvu x 

1  u 2
x c 2
 1  u 2
x c2 

1  vu c 
2 2 u  v 
 x
2
1  v 2
c 2 1  ux2 c 2  1  v 2
c 
2

x
c2
E  px v

1  v 2
c2 
(b) We summarize these results, and write the Lorentz transformation from Eq. 36-6, but solved in
terms of the primed variables. That can be easily done by interchanged primed and unprimed
quantities, and changing v to  v.
p  vE c 2 E  px v
px  x ; py  p y ; py  p y ; E  
1  v c 
2 2
1  v 2 c2 
x  vt t  vx c 2
x  ; y  y ; z  z ; t 
1  v 2
c2  1  v 2
c2 
These transformations are identical if we exchange px with x, p y with y, pz with z, and E c 2
with t (or E c with ct).

64. The galaxy is moving away from the Earth, and so we use Eq. 36-15b.
f 0  f  0.0987 f 0  f  0.9013 f 0
1   f f 0 2 
cv  c   1  0.9013  c  0.1035 c
2
f  f0  v  2 
cv 1+  f f 0 2   1  0.9013 
 

65. For source and observer moving towards each other, use Eq. 36-14b.
cv 1 v c 1  0.70
f  f0  f0   95.0 MHz   226 MHz  230 MHz
cv 1 v c 1  0.70

© 2009 Pearson Education, Inc., Upper Saddle River, NJ. All rights reserved. This material is protected under all copyright laws as they
currently exist. No portion of this material may be reproduced, in any form or by any means, without permission in writing from the publisher.
444
Chapter 36 The Special Theory of Relativity

66. We use Eq. 36-15a, and assume that v  c.


cv 1 v c 1  v c 1  v c    1  v c 1
  0  0  0  
cv 1 v c 1  v c 1  v c  0 1  v 2 c2 
  0  v
 0 1  v c  1  v 2 c 2 
1/ 2
 0 1  v c   0  0 v c   
0 0 c

67. (a) We apply Eq. 36-14b to determine the received/reflected frequency f. Then we apply this same
equation a second time using the frequency f as the source frequency to determine the Doppler-
shifted frequency f . We subtract the initial frequency from this Doppler-shifted frequency to
obtain the beat frequency. The beat frequency will be much smaller than the emitted frequency
when the speed is much smaller than the speed of light. We then set c  v  c and solve for v.
cv cv cv cv cv
f  f0 f f  f0  f0  
cv cv cv cv cv
cv cv 2v 2v cf beat
f beat  f   f 0  f 0    f0    f0  f0  v
cv cv cv c 2 f0

v
 3.00  10 m/s   6670 Hz   27.8m/s
8

2  36.0  10 Hz 
9

(b) We find the change in velocity and solve for the resulting change in beat frequency. Setting
the change in the velocity equal to 1 km/h we solve for the change in beat frequency.
cf cf beat 2 f v
v  beat  v   f beat  0
2 f0 2 f0 c
2  36.0  109 Hz  1km/h   1m/s 
f beat   70 Hz
 3.00  108 m/s   3.600 km/h 
68. We consider the difference between Doppler-shifted frequencies for atoms moving directly towards
the observer and atoms moving directly away. Use Eqs. 36-14b and 36-15b.
cv cv  cv cv   2v   2v c 
f  f 0  f0  f0     f0  2   f0  
cv cv  cv cv   c v 
2  1  v2 c2 
 
We take the speed to be the rms speed of thermal motion, given by Eq. 18-5. We also assume that
the thermal energy is much less than the rest energy, and so 3kT  mc 2 .
1/ 2
3kT v 3kT  3kT  f 3kT
3kT
v  vrms    
1  2  2
 2
m c mc 2  mc  f0 mc 2
mc 2
We evaluate for a gas of H atoms (not H 2 molecules) at 550 K. Use Appendix F to find the mass.

f 3kT 3 1.38  1023 J K   550 K 


2 2  2.5  105
mc 2 1.008 u  1.66  10 kg u  3.00  10 m s 
2
f0 27 8

© 2009 Pearson Education, Inc., Upper Saddle River, NJ. All rights reserved. This material is protected under all copyright laws as they
currently exist. No portion of this material may be reproduced, in any form or by any means, without permission in writing from the publisher.
445
Physics for Scientists & Engineers with Modern Physics, 4th Edition Instructor Solutions Manual

69. At the North Pole the clock is at rest, while the clock on the equator travels the circumference of the
Earth each day. We divide the circumference of the Earth by the length of the day to determine the
speed of the equatorial clock. We set the dilated time equal to 2.0 years and solve for the change in
rest times for the two clocks.
2 R 2  6.38  10 m 
6

v   464 m/s
T  24 hr  3600s/hr 
t0,eq  v2 
t   t0,eq  t 1  v 2 / c 2  t  1  2 
1  v 2 / c2  2c 
t
t  0,pole  t0,pole  t
1 0
 v2 
t0,eq  t0,pole  t  1  2   t
 2c 
 2.0 yr  464 m/s   3.156  107 s/yr 
2
v2
 t 2   75 s
2  3.00  108 m/s 
2
2c

70. We take the positive direction in the direction of the motion of the second pod. Consider the first
pod as reference frame S, and the spacecraft as reference frame S. The velocity of the spacecraft
relative to the first pod is v  0.60 c. The velocity of the first pod relative to the spacecraft is
ux  0.50 c. Solve for the velocity of the second pod relative to the first pod, ux , using Eq. 36-7a.

ux 
 ux  v    0.50c  0.60c   0.846 c
 vux  1   0.60  0.50 
1  2  
 c 

71. We treat the Earth as the stationary frame, and the airplane as the moving frame. The elapsed time in
the airplane will be dilated to the observers on the Earth. Use Eq. 36-1a.
2 rEarth 2 rEarth
tEarth  ; tplane  tEarth 1  v 2 c 2  1  v 2 c2
v v

t  tEarth  tplane 
2 rEarth
v

1  1  v2 c2  
2 rEarth   1 v 2    rEarth v
1   1  2 2   
v   c  c2
  1m s  
  6.38  106 m  1300 km h  
  3.6 km h  
  8.0  108 s
 3.00  108 m s 
2

72. (a) To travelers on the spacecraft, the distance to the star is contracted, according to Eq. 36-3a.
This contracted distance is to be traveled in 4.6 years. Use that time with the contracted
distance to find the speed of the spacecraft.
xspacecraft xEarth 1  v 2 c 2
v  
tspacecraft tspacecraft
1 1
vc c  0.6829 c  0.68 c
2 2
 ctspacecraft   4.6ly 
1   1  
 xEarth   4.3ly 

© 2009 Pearson Education, Inc., Upper Saddle River, NJ. All rights reserved. This material is protected under all copyright laws as they
currently exist. No portion of this material may be reproduced, in any form or by any means, without permission in writing from the publisher.
446
Chapter 36 The Special Theory of Relativity

(b) Find the elapsed time according to observers on Earth, using Eq. 36-1a.
tspaceship 4.6 y
tEarth    6.3y
1 v c
2 2
1  0.68292
Note that this agrees with the time found from distance and speed.
x 4.3ly
tEarth  Earth   6.3yr
v 0.6829 c

73. (a) We use Eq. 36-15a. To get a longer wavelength than usual means that the object is moving
away from the Earth.

  0
cv
 1.0700 
1.0702  1 c  v  0.067c
cv 1.0702  1
(b) We assume that the quasar is moving and the Earth is stationary. Then we use Eq. 16-9b.
f0 c c  1 
f        0 1  v c   1.0700  v  0.070 c
1 v c  0  1  v c 

74. We assume that some kind of a light signal is being transmitted from the astronaut to Earth, with a
frequency of the heartbeat. That frequency will then be Doppler shifted, according to Eq. 36-15b.
We express the frequencies in beats per minute.

f  f0
cv
 vc 2
 f 02  f 2   c  602  302   0.60 c
cv  f  f 02   602  302 
75. (a) The velocity components of the light in the S frame are ux  0 and uy  c. We transform those
velocities to the S frame according to Eq. 36-7.
ux  v 0v uy 1  v 2 c 2 c 1  v 2 c 2
ux    v ; u    c 1  v2 c2
1  vux c 2 1  0 1  vux c 2
y
1 0
uy c 1  v 2 c2 c2
  tan 1  tan 1  tan 1 2  1
ux v v
(b) u  ux2  u 2y  v 2  c 2 1  v 2 c 2   v 2  c 2  v 2  c
(c) In a Galilean transformation, we would have the following.
c
ux  ux  v  v ; u y  uy  c ; u  v 2  c 2   c  ;   tan 1
v

76. We take the positive direction as the direction of motion of rocket A. Consider rocket A as reference
frame S, and the Earth as reference frame S. The velocity of the Earth relative to rocket A is
v  0.65 c. The velocity of rocket B relative to the Earth is ux  0.85 c. Solve for the velocity of
rocket B relative to rocket A, ux , using Eq. 36-7a.

ux 
 ux  v    0.85c  0.65c   0.45c
 vux  1    0.65 0.85 
1  2  
 c 
Note that a Galilean analysis would have resulted in ux  0.20c.

© 2009 Pearson Education, Inc., Upper Saddle River, NJ. All rights reserved. This material is protected under all copyright laws as they
currently exist. No portion of this material may be reproduced, in any form or by any means, without permission in writing from the publisher.
447
Physics for Scientists & Engineers with Modern Physics, 4th Edition Instructor Solutions Manual

77. (a) We find the speed from Eq. 36-10.


 1 
K    1 mc 2    1 mc 2  14,000mc 2 
 1  v2 c2 
 
2 2
 1  c 1 
v  c 1   c  
 14,001  2  14,001 

cv  
c 1 
2


 3.00  108 m s   1 2  0.77 m s
2  14,001  2  14,001 
 
(b) The tube will be contracted in the rest frame of the electron, according to Eq. 36-3a.
  1 2 
l 0  l 1  v 2 c 2   3.0  103 m  1  1      0.21m
  14,001  

78. The electrostatic force provides the radial acceleration. We solve that relationship for the speed of
the electron.
1 e 2 melectron v 2
Felectrostatic  Fcentripetal   
4 0 r 2 r

8.99  10  1.60  10 C
19 2
1 e2
9
N  m 2 C2
v   2.18  106 m s  0.0073 c
4 0 melectron r 9.11  1031 kg  0.53  1010 m 
Because this is much less than 0.1c, the electron is not relativistic.

79. The minimum energy required would be the energy to produce the pair with no kinetic energy, so the
total energy is their rest energy. They both have the same mass. Use Eq. 36-12.
E  2mc 2  2  0.511MeV   1.022 MeV 1.64  1013 J 

80. The wattage times the time is the energy required. We use Eq. 36-12 to calculate the mass.
Pt  75W   3.16  10 s   1000g 
7
5
E  Pt  mc  m  2 
2
   2.6  10 g
 3.00  10 m s  
2
c 8 1kg 

81. Use Eqs. 36-13, 36-8, and 36-11.


E 2  p 2c 2  m2c 4  E   p 2c 2  m 2c 4 
1/ 2

dE pc 2 pc 2  mvc 2
p c  m2c 4 
1/ 2
 1 2 2
2 pc 2     v
dp
2
E E  mc 2

82. The kinetic energy available comes from the decrease in rest energy.
K  mn c 2   mp c 2  me c 2  mv c 2   939.57 MeV   938.27 MeV  0.511MeV  0   0.79 MeV

83. (a) We find the rate of mass loss from Eq. 36-12.
E  mc 2  E   m  c 2 
m 1  E  4  1026 J s
 2    4.44  109 kg s  4  109 kg s
t c  t   3.00  108 m s 2

© 2009 Pearson Education, Inc., Upper Saddle River, NJ. All rights reserved. This material is protected under all copyright laws as they
currently exist. No portion of this material may be reproduced, in any form or by any means, without permission in writing from the publisher.
448
Chapter 36 The Special Theory of Relativity

(b) Find the time from the mass of the Sun and the rate determined in part (a).
m
t  Earth 
5.98  1024 kg   4.27  107 y  4  107 y
m t  4.44  109 kg s  3.156  107 s y 
(c) We find the time for the Sun to lose all of its mass at this same rate.
m
t  Sun 
1.99  1030 kg   1.42  1013 y  1  1013 y
m t  4.44  109 kg s  3.156  107 s y 

84. Use Eq. 36-8 for the momentum to find the mass.
mv
p   mv  
1  v2 c2
2
 2.24  108 m s 
p 1  v2 c2
 3.07  10 kg  m s 
22
1   3.00  108 m s 
 
m   9.12  1031 kg
v 2.24  10 m s
8

This particle has the mass of an electron, and a negative charge, so it must be an electron.

85. The total binding energy is the energy required to provide the increase in rest energy.
E   2mp+e  2mn   mHe  c 2
 931.5MeV c 2 
  2 1.00783u   2 1.00867 u   4.00260 u  c 2    28.32 MeV
 u 

86. The momentum is given by Eq. 36-8, and the energy is given by Eq. 36-11 and Eq. 36-13.
 mc 2 v Ev pc 2 pc 2 pc
P   mv  2
  v   
c c2 E m2c4  p 2c2 m 2c 2  p 2

87. (a) The magnitudes of the momenta are equal. We use Eq. 36-8.
1 mc  v c  1  938.3MeV  0.985
2
mv
p   mv     5356 MeV c
1  v2 c2 c 1  v2 c2 c 1  0.9852
 1c   1.602  1010 J GeV 
 5.36GeV c   5.36GeV c    
 3.00  10 m s 
8
1GeV 
 2.86  1018 kg m s
(b) Because the protons are moving in opposite directions, the vector sum of the momenta is 0.
(c) In the reference frame of one proton, the laboratory is moving at 0.985c. The other
proton is moving at  0.985c relative to the laboratory. We find the speed of one proton
relative to the other, and then find the momentum of the moving proton in the rest frame of the
other proton by using that relative velocity.

ux 
 v  ux   0.985 c   0.985 c   0.9999 c
 vux  1   0.985 0.985
1  2  
 c 

© 2009 Pearson Education, Inc., Upper Saddle River, NJ. All rights reserved. This material is protected under all copyright laws as they
currently exist. No portion of this material may be reproduced, in any form or by any means, without permission in writing from the publisher.
449
Physics for Scientists & Engineers with Modern Physics, 4th Edition Instructor Solutions Manual

 2  0.985 
 938.3MeV   2 
1 mc  ux c  1  1  0.985 
2
mu x
p   mux     62081MeV c
1 u c2
x
2 c 1  u x2 c 2 c  2  0.985 
2

1  2 
 1  0.985 
 1c   1.602  1010 J GeV 
 62.1GeV c   62.1GeV c    
 3.00  10 m
8
s  1GeV 
 3.31  1017 kg m s

88. We find the loss in mass from Eq. 36-12.


E 484  103 J
m  2   5.38  1012 kg
 3.00  10 m s 
2
c 8

Two moles of water has a mass of 36  103 kg. Find the percentage of mass lost.
5.38  1012 kg
 1.49  1010  1.5  10 8 %
36  103 kg

89. Use Eq. 36-10 for kinetic energy, and Eq. 36-12 for rest energy.
K    1 mEnterprise c 2  mconverted c 2 
 1   1 
mconverted    1 mEnterprise    1  6  109 kg   3  107 kg
 1  v 2 c2   1  0.10
2

 

90. We set the kinetic energy of the spacecraft equal to the rest energy of an unknown mass. Use Eqs.
36-10 and 36-12.
K    1 mship c 2  mc 2 
 1   1 
m    1 mship    1 mship    1 1.8  105 kg   7.2  104 kg
 1  v2 c2   1  0.70
2

 
From the Earth’s point of view, the distance is 35 ly and the speed is 0.70c. That data is used to
calculate the time from the Earth frame, and then Eq. 36-1a is used to calculate the time in the
spaceship frame.
d  35y  c
t    50 y ; t0  t 1  v 2 c 2   50 y  1  0.702  36 y
v 0.70c

91. We assume one particle is moving in the negative direction in the laboratory frame, and the other
particle is moving in the positive direction. We consider the particle moving in the negative
direction as reference frame S, and the laboratory as reference frame S. The velocity of the
laboratory relative to the negative-moving particle is v  0.85 c, and the velocity of the positive-
moving particle relative to the laboratory frame is ux  0.85 c. Solve for the velocity of the positive-
moving particle relative to the negative-moving particle, u x .

ux 
 ux  v    0.85c  0.85c   0.987 c
 vux  1   0.85 0.85
1  2 
 c 

© 2009 Pearson Education, Inc., Upper Saddle River, NJ. All rights reserved. This material is protected under all copyright laws as they
currently exist. No portion of this material may be reproduced, in any form or by any means, without permission in writing from the publisher.
450
Chapter 36 The Special Theory of Relativity

92. We consider the motion from the reference frame of the spaceship. The passengers will see the trip
distance contracted, as given by Eq. 36-3a. They will measure their speed to be that contracted
distance divided by the year of travel time (as measured on the ship). Use that speed to find the work
done (the kinetic energy of the ship).
l l 1  v 2 c2 v 1 1
v  0     0.9887 c
t0 t0 c  2   2 
 1  c  t   1.0ly 
   1   6.6ly  
0
 l
 0      

 1 
W  K    1 mc 2    1 mc 2
 1  v 2 c2 
 
 1 
 1  3.2  104 kg  3.00  108 m s   1.6  1022 J
2

 1  0.9887 2 
 

93. The kinetic energy is given by Eq. 36-10.


   
1 1
K    1 mc 2    1 mc 2    1 14,500 kg   3.00  108 m s 
2

 1  v2 c2   1   0.98 2 
   
 5.3  1021 J
5.3  1021 J
We compare this with annual U.S. energy consumption:  53.
1020 J
The spaceship’s kinetic energy is over 50 times as great.

94. The pi meson decays at rest, and so the momentum of the muon and the neutrino must each have the
same magnitude (and opposite directions). The neutrino has no rest mass, and the total energy must
be conserved. We combine these relationships using Eq. 36-13.
Ev   pv 2 c 2  mv 2 c 4 
1/ 2
 pv c ; p  pv  p

E  E  Ev  m c 2   p 2 c 2  m 2 c 4   pv c   p 2 c 2  m 2 c 4 
1/ 2 1/ 2
 pc 

m c 2  pc   p 2 c 2  m 2 c 4  m c  pc    p 2 c 2  m 2 c 4 
1/ 2 2
 
2

Solve for the momentum.


m 2 c 2  m 2 c 2
m 2 c 4  2m c 2 pc  p 2 c 2  p 2 c 2  m 2 c 4  pc 
2m
Write the kinetic energy of the muon using Eqs. 36-11 and 36-13.
K   E  m c 2 ; E  E  Ev  m c 2  pc 

K    m c 2  pc   m c 2  m c 2  m c 2 
m 
2 2
c  m 2 c 2 
2m
2m  m c 2  m c 2  m  c  m 2 c 2 
2 2

 
2m 2m

 2m  2m m  m 2  m 2  c 2 m  2m m  m 2  c 2 m  m  c 2
2 2 2
  
  
2m 2m 2m

© 2009 Pearson Education, Inc., Upper Saddle River, NJ. All rights reserved. This material is protected under all copyright laws as they
currently exist. No portion of this material may be reproduced, in any form or by any means, without permission in writing from the publisher.
451
Physics for Scientists & Engineers with Modern Physics, 4th Edition Instructor Solutions Manual

95. (a) The relative speed can be calculated in either frame, and will be the same value in both frames.
The time as measured on the Earth will be longer than the time measured on the spaceship, as
given by Eq. 36-1a.
x tspaceship tspaceship
v  Earth ; tEarth   
tEarth 1 v c2 2
 xEarth 
2

1  
 ctEarth 
2 2
 x   x 
 tEarth    Earth    tspaceship    tEarth    Earth    tspaceship  
2 2 2 2

 c   c 
2
 x 
tEarth   Earth    tspaceship    6.0 y    2.50 y   6.5y
2 2 2

 c 
(b) The distance as measured by the spaceship will be contracted.
x x t 2.50 y
v  Earth  spaceship  xspaceship  spaceship xEarth   6.0ly   2.3ly
tEarth tspaceship tEarth 6.5y
This is the same distance as found using the length contraction relationship.

96. (a) To observers on the ship, the period is non-relativistic. Use Eq. 14-7b.

T  2
m
 2
1.88 kg   0.939s
k 84.2 N m 
(b) The oscillating mass is a clock. According to observers on Earth, clocks on the spacecraft run
slow.
TEarth 
T

 0.939s   2.15s
1 v c 1   0.900 
2 2 2

97. We use the Lorentz transformations to derive the result.


 vx   v x  
x    x   vt   x    x   vt  ; t    t   2   t    t   2 
 c   c 
 vx  
2
 vx   
2

 ct    x    c  t   2     x  vt    2   ct    
2 2 2
   x   v  t  
  c    c   
 2 vx   vx 
2
2

  c  t   2ct    x   2 xvt    vt  
2 2
2
  
 c  c  
1  2  v 2  
2
   c  v 2
  t  
2
     1  x   
1  v 2 c2    c   


1
1  v 2 c2

c 2 1  v 2 c 2   t   1  v 2 c 2   x 
2 2

1  v 2
c2 

1  v 2
c 2 ct 2
  x 
2
   ct 2
  x 
2

© 2009 Pearson Education, Inc., Upper Saddle River, NJ. All rights reserved. This material is protected under all copyright laws as they
currently exist. No portion of this material may be reproduced, in any form or by any means, without permission in writing from the publisher.
452
Chapter 36 The Special Theory of Relativity

98. We assume that the left edge of the glass is even with point A when the flash of light is emitted.
There is no loss of generality with that assumption. We do the calculations in the frame of reference
in which points A and B are at rest, and the glass is then moving to the right with speed v.
If the glass is not moving, we would have this “no motion” result.
distance in glass distance in vacuum d ld
tv 0  tglass  tvacuum    
speed in glass speed in vacuum vglass c
d l  d nd l  d nd  l  d l   n  1 d
     
cn c c c c c
If the index of refraction is n  1, then the glass will have no effect on the light, and the time would
simply be the distance divided by the speed of light.
distance in glass distance in vacuum d l  d d  l  d l
tn 1  tglass  tvacuum      
speed in glass speed in vacuum c c c c
Now, let us consider the problem from a relativistic point of view. The speed of light in the glass
will be the relativistic sum of the speed of light in stationary glass, c n , and the speed of the glass, v,
by Eq. 36-7a. We define  to simplify further expressions.
c c  vn    vn  
v v
c 1   c  1   
 
c 
  
c 
vlight  n  n  
cv v n  v  n   v  
in glass
1 2 1 1   1   
nc nc  nc    nc  
The contracted width of the glass, from the Earth frame of reference, is given by Eq. 36-3a.
d
d moving  d 1  v 2 c 2 
glass 
We assume the light enters the block when the left edge of the block is at point A, and write simple
equations for the displacement of the leading edge of the light, and the leading edge of the block. Set
them equal and solve for the time when the light exits the right edge of the block.
c d
xlight  vlight t   t ; xright   vt ;
in glass n edge 
c d d n
xlight  xright   tglass   vtglass  tglass 
edge n     c  nv 
d n
Where is the front edge of the block when the light emerges? Use tglass  with either
   c  nv 
expression – for the leading edge of the light, or the leading edge of the block.
cd n  cd
xlight  vlight tglass   
in glass n    c  nv   c  nv 
 
d d d n d   c  nv   vdn  cd
xright   vtglass  v  
edge      c  nv     c  nv     c  nv 
 cd
The part of the path that is left, l  , will be traveled at speed c by the light. We express
   c  nv 
that time, and then find the total time.
 cd
l
   c  nv 
tvacuum 
c

© 2009 Pearson Education, Inc., Upper Saddle River, NJ. All rights reserved. This material is protected under all copyright laws as they
currently exist. No portion of this material may be reproduced, in any form or by any means, without permission in writing from the publisher.
453
Physics for Scientists & Engineers with Modern Physics, 4th Edition Instructor Solutions Manual

 cd
l
d n    c  nv  l d n
ttotal  tglass  tvacuum  tglass    
   c  nv  c c    c  nv 

l  n  1 d c  v


c c cv
We check this for the appropriate limiting cases.
l  n  1 d c  v l  n  1 d c  c l
Case 1: ttotal     
v c c c cv c c cc c
This result was expected, because the speed of the light would always be c.
l  n  1 d c  v l  n  1 d l   n  1 d
Case 2: ttotal     1 
v 0 c c cv c c c
This result was obtained earlier in the solution.
l  n  1 d c  v l
Case 3: ttotal   
n 1 c c cv c
This result was expected, because then there is no speed change in the glass.

99. The spreadsheet used for this


problem can be found on the 1.2
Media Manager, with filename 1.0
“PSE4_ISM_CH36.XLS,” on Classical
0.8
K (10 J)

tab “Problem 36.99.”


Relativistic
17

0.6

0.4

0.2

0.0
0 0.2 0.4 0.6 0.8
v /c

1
100. (a) We use Eq. 36-98. Since there is motion in two dimensions, we have   .
2
v x2 v y
1 2  2
c c
 
dp dp x dp y
F   Fˆj  ;  0  px   mv x  p0 ;   F  p y   Ft   mv y
dt dt dt
Use the component equations to obtain expressions for v x2 and v 2y .

 mv x  p0  v x 
p0 p2 p2  v2 v 
2
 c2  v 2y 
 v x2  2 0 2  02  1  x2  2y   v x2  p02 2 2
m  m m  c c   m c  p02 
F 2t 2 F 2t 2  v 2 v 
2
 Ft
 mv y   Ft  v y   v 2y  2 2  2  1  x2  2y  
m  m m  c c 

v F t
2 2 2 c 2
 v x2 
y
m c
2 2
 F 2t 2 
Substitute the expression for v 2y into the expression for v x2 .

© 2009 Pearson Education, Inc., Upper Saddle River, NJ. All rights reserved. This material is protected under all copyright laws as they
currently exist. No portion of this material may be reproduced, in any form or by any means, without permission in writing from the publisher.
454
Chapter 36 The Special Theory of Relativity

 2  c2  vx2  
 c  F 2t 2 2 2
v p
2 2 c 2
 v 2y  
2 
 p0
 m c  F 2t 2  
 p0 2 2
2  m 2 c 4  F 2t 2 v x2 

x 0
m c2 2
 p02   m2c2  p02   m c  p02  m2c2  F 2t 2 
v x2  m 2 c 2  p02  m 2 c 2  F 2t 2   p02  m 2 c 4  F 2t 2 v x2  
v x2 m 4 c 4  v x2 m 2 c 2 p02  v x2 F 2t 2 m 2 c 2  v x2 F 2t 2 p02  p02 m 2 c 4  p02 F 2t 2 v x2 
p0 c
v x2 m 2 c 2  v x2 p02  v x2 F 2t 2  p02 c 2  vx 
m c  p02  F 2t 2 
2 2 1/ 2

Use the expression for v x to solve for v y .


 2 p 2c 2 
c  2 2 0 2 
v 2y  F 2t 2
c 2
 v x2  
2 2 
F t
 m c  p0  F 2t 2  
m c  F t 
2 2 2 2
 m 2 c 2  F 2t 2 
F t2 2 c m c  p  F t   p c   F t
2 2 2 2
0
2 2 2 2
0 2 2
c 2  m 2 c 2  F 2t 2 

 m c  F t  m c  p  F t 
2 2 2 2
m c
2 2 2
0
2 2 2 2
 F 2t 2  m 2 c 2  p02  F 2t 2 
 Ftc
vy 
m c  p02  F 2t 2 
2 2 1/ 2

The negative sign comes from taking the negative square root of the previous equation. We
know that the particle is moving down.
(b) See the graph. We are
plotting v x c and  v y c . 1.0

The spreadsheet used for this 0.8


problem can be found on the vx
0.6
Media Manager, with filename
(- vy)
v /c

“PSE4_ISM_CH36.XLS,” on 0.4
tab “Problem 36.100.”
0.2

0.0
0 1 2 t ( s) 3 4 5

(c) The path is not parabolic, because the v x is not constant. Even though there is no force in the x-
direction, as the net speed of the particle increases,  increases. Thus v x must decrease as time
elapses in order for px to stay constant.

© 2009 Pearson Education, Inc., Upper Saddle River, NJ. All rights reserved. This material is protected under all copyright laws as they
currently exist. No portion of this material may be reproduced, in any form or by any means, without permission in writing from the publisher.
455
A science fantasy book called Mr Tompkins in Wonderland (1940), by physicist George Gamow,
imagined a world in which the speed of light was only 10 m兾s (20 mi兾h). Mr Tompkins had
studied relativity and when he began “speeding” on a bicycle, he “expected that he would be
immediately shortened, and was very happy about it as his increasing figure had lately caused him
some anxiety. To his great surprise, however, nothing happened to him or to his cycle. On the
other hand, the picture around him completely changed. The streets grew shorter, the windows
of the shops began to look like narrow slits, and the policeman on the corner became the thinnest
man he had ever seen. ‘By Jove!’
exclaimed Mr Tompkins excitedly,
‘I see the trick now. This is where
the word relativity comes in.’”
Relativity does indeed predict
that objects moving relative to us
at high speed, close to the speed
of light c, are shortened in
length. We don’t notice it as
Mr Tompkins did, because
c = 3 * 108 m兾s is incredibly
fast. We will study length
contraction, time dilation,
simultaneity non-agreement, and
how energy and mass are
equivalent AE = mc2 B.

A P T E
H

26 The Special Theory


R
C

of Relativity
CONTENTS CHAPTER-OPENING QUESTION—Guess now!
26–1 Galilean–Newtonian A rocket is headed away from Earth at a speed of 0.80c. The rocket fires a small
Relativity payload at a speed of 0.70c (relative to the rocket) aimed away from Earth. How
26–2 Postulates of the Special fast is the payload moving relative to Earth?
Theory of Relativity (a) 1.50c;
26–3 Simultaneity (b) a little less than 1.50c;
26–4 Time Dilation and the Twin (c) a little over c;
Paradox
(d) a little under c;
26–5 Length Contraction (e) 0.75c.
26–6 Four-Dimensional
Space–Time

P
26–7 Relativistic Momentum hysics at the end of the nineteenth century looked back on a period of
26–8 The Ultimate Speed great progress. The theories developed over the preceding three centuries
26–9 E = mc2; Mass and Energy
had been very successful in explaining a wide range of natural phenomena.
Newtonian mechanics beautifully explained the motion of objects on Earth and
26–10 Relativistic Addition of
Velocities in the heavens. Furthermore, it formed the basis for successful treatments of
26–11 The Impact of Special fluids, wave motion, and sound. Kinetic theory explained the behavior of gases
Relativity and other materials. Maxwell’s theory of electromagnetism embodied all of electric
and magnetic phenomena, and it predicted the existence of electromagnetic
waves that would behave just like light—so light came to be thought of as an
electromagnetic wave. Indeed, it seemed that the natural world, as seen through
the eyes of physicists, was very well explained. A few puzzles remained, but it
was felt that these would soon be explained using already known principles.
744
It did not turn out so simply. Instead, these puzzles were to be solved only by
the introduction, in the early part of the twentieth century, of two revolutionary
new theories that changed our whole conception of nature: the theory of relativity
and quantum theory.
Physics as it was known at the end of the nineteenth century (what we’ve
covered up to now in this book) is referred to as classical physics. The new
physics that grew out of the great revolution at the turn of the twentieth century
is now called modern physics. In this Chapter, we present the special theory of
relativity, which was first proposed by Albert Einstein (1879–1955; Fig. 26–1) in
1905. In Chapter 27, we introduce the equally momentous quantum theory.

FIGURE 26–1 Albert Einstein


(1879–1955), one of the great minds
of the twentieth century, was the
creator of the special and general
theories of relativity.

26–1 Galilean–Newtonian Relativity


Einstein’s special theory of relativity deals with how we observe events, particu-
larly how objects and events are observed from different frames of reference.†
This subject had already been explored by Galileo and Newton.
The special theory of relativity deals with events that are observed and meas-
ured from so-called inertial reference frames (Section 4–2 and Appendix C),
which are reference frames in which Newton’s first law is valid: if an object
experiences no net force, the object either remains at rest or continues in motion
with constant speed in a straight line. It is usually easiest to analyze events when
they are observed and measured by observers at rest in an inertial frame. The
Earth, though not quite an inertial frame (it rotates), is close enough that for
most purposes we can approximate it as an inertial frame. Rotating or otherwise
accelerating frames of reference are noninertial frames,‡ and won’t concern us in
this Chapter (they are dealt with in Einstein’s general theory of relativity, as we
will see in Chapter 33).
A reference frame that moves with constant velocity with respect to an
inertial frame is itself also an inertial frame, since Newton’s laws hold in it as well.
When we say that we observe or make measurements from a certain reference
frame, it means that we are at rest in that reference frame.


A reference frame is a set of coordinate axes fixed to some object such as the Earth, a train, or the
Moon. See Section 2–1.

On a rotating platform (say a merry-go-round), for example, a ball at rest starts moving outward
even though no object exerts a force on it. This is therefore not an inertial frame. See Appendix C,
Fig. C–1.

SECTION 26–1 Galilean–Newtonian Relativity 745


(a) (b)
Reference frame = car Reference frame = Earth
FIGURE 26–2 A coin is dropped by
a person in a moving car. The upper
Both Galileo and Newton were aware of what we now call the relativity principle
views show the moment of the coin’s applied to mechanics: that the basic laws of physics are the same in all inertial
release, the lower views are a short reference frames. You may have recognized its validity in everyday life. For
time later. (a) In the reference frame example, objects move in the same way in a smoothly moving (constant-velocity)
of the car, the coin falls straight down train or airplane as they do on Earth. (This assumes no vibrations or rocking
(and the tree moves to the left). which would make the reference frame noninertial.) When you walk, drink a
(b) In a reference frame fixed on the cup of soup, play pool, or drop a pencil on the floor while traveling in a train,
Earth, the coin has an initial velocity airplane, or ship moving at constant velocity, the objects move just as they do
(= to car’s) and follows a curved when you are at rest on Earth. Suppose you are in a car traveling rapidly at
(parabolic) path. constant velocity. If you drop a coin from above your head inside the car, how
will it fall? It falls straight downward with respect to the car, and hits the floor
directly below the point of release, Fig. 26–2a. This is just how objects fall on the
Earth—straight down—and thus our experiment in the moving car is in accord
with the relativity principle. (If you drop the coin out the car’s window, this won’t
happen because the moving air drags the coin backward relative to the car.)
CAUTION Note in this example, however, that to an observer on the Earth, the coin fol-
Laws are the same, but lows a curved path, Fig. 26–2b. The actual path followed by the coin is different
paths may be different in different as viewed from different frames of reference. This does not violate the relativity
reference frames principle because this principle states that the laws of physics are the same in
all inertial frames. The same law of gravity, and the same laws of motion, apply in
both reference frames. The acceleration of the coin is the same in both reference
frames. The difference in Figs. 26–2a and b is that in the Earth’s frame of reference,
the coin has an initial velocity (equal to that of the car). The laws of physics there-
fore predict it will follow a parabolic path like any projectile (Chapter 3). In the
car’s reference frame, there is no initial velocity, and the laws of physics predict
that the coin will fall straight down. The laws are the same in both reference frames,
although the specific paths are different.
CAUTION Galilean–Newtonian relativity involves certain unprovable assumptions that
Length and time intervals make sense from everyday experience. It is assumed that the lengths of objects
are absolute (pre-relativity) are the same in one reference frame as in another, and that time passes at the
same rate in different reference frames. In classical mechanics, then, space and
time intervals are considered to be absolute: their measurement does not change
from one reference frame to another. The mass of an object, as well as all forces,
are assumed to be unchanged by a change in inertial reference frame.
CAUTION The position of an object, however, is different when specified in different
Position and velocity are different in reference frames, and so is velocity. For example, a person may walk inside a bus
different reference frames, but length toward the front with a speed of 2 m兾s. But if the bus moves 10 m兾s with respect
is the same (classical) to the Earth, the person is then moving with a speed of 12 m兾s with respect to the
Earth. The acceleration of an object, however, is the same in any inertial
reference frame according to classical mechanics. This is because the change in
velocity, and the time interval, will be the same. For example, the person in the
bus may accelerate from 0 to 2 m兾s in 1.0 seconds, so a = 2 m兾s2 in the reference
frame of the bus. With respect to the Earth, the acceleration is
(12 m兾s - 10 m兾s)兾(1.0 s) = 2 m兾s2,
which is the same.
746 CHAPTER 26
Since neither F, m, nor a changes from one inertial frame to another, Newton’s
second law, F = ma, does not change. Thus Newtons’ second law satisfies the relativity
principle. The other laws of mechanics also satisfy the relativity principle.
That the laws of mechanics are the same in all inertial reference frames
implies that no one inertial frame is special in any sense. We express this
important conclusion by saying that all inertial reference frames are equivalent
for the description of mechanical phenomena. No one inertial reference frame is
any better than another. A reference frame fixed to a car or an aircraft traveling
at constant velocity is as good as one fixed on the Earth. When you travel
smoothly at constant velocity in a car or airplane, it is just as valid to say you are
at rest and the Earth is moving as it is to say the reverse.† There is no experiment
you can do to tell which frame is “really” at rest and which is moving. Thus, there
is no way to single out one particular reference frame as being at absolute rest.
A complication arose, however, in the last half of the nineteenth century.
Maxwell’s comprehensive and successful theory of electromagnetism (Chapter 22)
predicted that light is an electromagnetic wave. Maxwell’s equations gave the
velocity of light c as 3.00 * 108 m兾s; and this is just what is measured. The
question then arose: in what reference frame does light have precisely the value
predicted by Maxwell’s theory? It was assumed that light would have a different
speed in different frames of reference. For example, if observers could travel
on a rocket ship at a speed of 1.0 * 108 m兾s away from a source of light, we
might expect them to measure the speed of the light reaching them to be
A3.0 * 108 m兾sB - A1.0 * 108 m兾sB = 2.0 * 108 m兾s. But Maxwell’s equations
have no provision for relative velocity. They predicted the speed of light to be
c = 3.0 * 108 m兾s, which seemed to imply that there must be some preferred
reference frame where c would have this value.
We discussed in Chapters 11 and 12 that waves can travel on water and along
ropes or strings, and sound waves travel in air and other materials. Nineteenth-
century physicists viewed the material world in terms of the laws of mechanics, so
it was natural for them to assume that light too must travel in some medium.
They called this transparent medium the ether and assumed it permeated all space.‡
It was therefore assumed that the velocity of light given by Maxwell’s equations
must be with respect to the ether.§
Scientists soon set out to determine the speed of the Earth relative to this
absolute frame, whatever it might be. A number of clever experiments were
designed. The most direct were performed by A. A. Michelson and E. W. Morley
in the 1880s. They measured the difference in the speed of light in different
directions using Michelson’s interferometer (Section 24–9). They expected to find
a difference depending on the orientation of their apparatus with respect to the
ether. For just as a boat has different speeds relative to the land when it moves
upstream, downstream, or across the stream, so too light would be expected to
have different speeds depending on the velocity of the ether past the Earth.
Strange as it may seem, they detected no difference at all. This was a great
puzzle. A number of explanations were put forth over a period of years, but they
led to contradictions or were otherwise not generally accepted. This null result
was one of the great puzzles at the end of the nineteenth century.
Then in 1905, Albert Einstein proposed a radical new theory that reconciled
these many problems in a simple way. But at the same time, as we shall see, it
completely changed our ideas of space and time.

We use the reasonable approximation that Earth is an inertial reference frame.

The medium for light waves could not be air, since light travels from the Sun to Earth through nearly
empty space. Therefore, another medium was postulated, the ether. The ether was not only trans-
parent but, because of difficulty in detecting it, was assumed to have zero density.
§
Also, it appeared that Maxwell’s equations did not satisfy the relativity principle: They were simplest
in the frame where c = 3.00 * 108 m兾s, in a reference frame at rest in the ether. In any other
reference frame, extra terms were needed to account for relative velocity. Although other laws of
physics obeyed the relativity principle, the laws of electricity and magnetism apparently did not.
Einstein’s second postulate (next Section) resolved this problem: Maxwell’s equations do satisfy
relativity.

SECTION 26–1 Galilean–Newtonian Relativity 747


26–2 Postulates of the
Special Theory of Relativity
The problems that existed at the start of the twentieth century with regard to
electromagnetic theory and Newtonian mechanics were beautifully resolved by
Einstein’s introduction of the special theory of relativity in 1905. Unaware of the
Michelson–Morley null result, Einstein was motivated by certain questions
regarding electromagnetic theory and light waves. For example, he asked himself:
“What would I see if I rode a light beam?” The answer was that instead of a trav-
eling electromagnetic wave, he would see alternating electric and magnetic fields
at rest whose magnitude changed in space, but did not change in time. Such fields,
he realized, had never been detected and indeed were not consistent with Maxwell’s
electromagnetic theory. He argued, therefore, that it was unreasonable to think
that the speed of light relative to any observer could be reduced to zero, or in fact
reduced at all. This idea became the second postulate of his theory of relativity.
In his famous 1905 paper, Einstein proposed doing away with the idea of the
ether and the accompanying assumption of a preferred or absolute reference frame
at rest. This proposal was embodied in two postulates. The first was an extension
of the Galilean–Newtonian relativity principle to include not only the laws of
mechanics but also those of the rest of physics, including electricity and magnetism:
First postulate (the relati£ity principle): The laws of physics have the same
RELATIVITY PRINCIPLE
form in all inertial reference frames.
The first postulate can also be stated as: there is no experiment you can do in an
inertial reference frame to determine if you are at rest or moving uniformly at
constant velocity.
The second postulate is consistent with the first:
Second postulate (constancy of the speed of light): Light propagates through
SPEED OF LIGHT PRINCIPLE empty space with a definite speed c independent of the speed of the source or
observer.
These two postulates form the foundation of Einstein’s special theory of relativity.
It is called “special” to distinguish it from his later “general theory of relativity,”
which deals with noninertial (accelerating) reference frames (Chapter 33). The
special theory, which is what we discuss here, deals only with inertial frames.
The second postulate may seem hard to accept, for it seems to violate common
sense. First of all, we have to think of light traveling through empty space. Giving
up the ether is not too hard, however, since it had never been detected. But the
second postulate also tells us that the speed of light in vacuum is always the same,
3.00 * 108 m兾s, no matter what the speed of the observer or the source. Thus, a per-
son traveling toward or away from a source of light will measure the same speed
for that light as someone at rest with respect to the source. This conflicts with our
everyday experience: we would expect to have to add in the velocity of the observer.
On the other hand, perhaps we can’t expect our everyday experience to be
helpful when dealing with the high velocity of light. Furthermore, the null result of
the Michelson–Morley experiment is fully consistent with the second postulate.†
Einstein’s proposal has a certain beauty. By doing away with the idea of an
absolute reference frame, it was possible to reconcile classical mechanics with
Maxwell’s electromagnetic theory. The speed of light predicted by Maxwell’s
equations is the speed of light in vacuum in any reference frame.
Einstein’s theory required us to give up common sense notions of space and
time, and in the following Sections we will examine some strange but interesting con-
sequences of special relativity. Our arguments for the most part will be simple ones.

The Michelson–Morley experiment can also be considered as evidence for the first postulate, since it
was intended to measure the motion of the Earth relative to an absolute reference frame. Its failure
to do so implies the absence of any such preferred frame.

748 CHAPTER 26 The Special Theory of Relativity


We will use a technique that Einstein himself did: we will imagine very simple
experimental situations in which little mathematics is needed. In this way, we can
see many of the consequences of relativity theory without getting involved in
detailed calculations. Einstein called these thought experiments.

26–3 Simultaneity
An important consequence of the theory of relativity is that we can no longer
regard time as an absolute quantity. No one doubts that time flows onward and
never turns back. But according to relativity, the time interval between two
events, and even whether or not two events are simultaneous, depends on the
observer’s reference frame. By an event, which we use a lot here, we mean some-
thing that happens at a particular place and at a particular time.
Two events are said to occur simultaneously if they occur at exactly the same
time. But how do we know if two events occur precisely at the same time? If they
occur at the same point in space—such as two apples falling on your head at the
same time—it is easy. But if the two events occur at widely separated places, it is
more difficult to know whether the events are simultaneous since we have to take
into account the time it takes for the light from them to reach us. Because light
travels at finite speed, a person who sees two events must calculate back to find
out when they actually occurred. For example, if two events are observed to
occur at the same time, but one actually took place farther from the observer
than the other, then the more distant one must have occurred earlier, and the two
events were not simultaneous.

FIGURE 26–3 A moment after


O
lightning strikes at points A and B, the
A B pulses of light (shown as blue waves)
are traveling toward the observer O,
but O “sees” the lightning only when
the light reaches O.
Light coming from
the two events
at A and B

We now imagine a simple thought experiment. Assume an observer, called O,


is located exactly halfway between points A and B where two events occur,
Fig. 26–3. Suppose the two events are lightning that strikes the points A and B, as
shown. For brief events like lightning, only short pulses of light (blue in FIGURE 26–4 Observers O1 and O2 ,
on two different trains (two different
Fig. 26–3) will travel outward from A and B and reach O. Observer O “sees” the
reference frames), are moving with
events when the pulses of light reach point O. If the two pulses reach O at the
relative speed v. (a) O2 says that O1
same time, then the two events had to be simultaneous. This is because (i) the two is moving to the right. (b) O1 says
light pulses travel at the same speed (postulate 2), and (ii) the distance OA that O2 is moving to the left. Both
equals OB, so the time for the light to travel from A to O and from B to O must viewpoints are legitimate: it all
be the same. Observer O can then definitely state that the two events occurred depends on your reference frame.
simultaneously. On the other hand, if O sees the light from one event before that
from the other, then the former event occurred first. O1 vB
The question we really want to examine is this: if two events are simultaneous
to an observer in one reference frame, are they also simultaneous to another O2
observer moving with respect to the first? Let us call the observers O1 and O2
(a)
and assume they are fixed in reference frames 1 and 2 that move with speed v
relative to one another. These two reference frames can be thought of as two
O1
rockets or two trains (Fig. 26–4). O2 says that O1 is moving to the right with speed v,
as in Fig. 26–4a; and O1 says O2 is moving to the left with speed v, as in Fig. 26–4b.
vB O2
Both viewpoints are legitimate according to the relativity principle. [There is no
third point of view that will tell us which one is “really” moving.] (b)

SECTION 26–3 Simultaneity 749


Now suppose that observers O1 and O2 observe and measure two lightning
strikes. The lightning bolts mark both trains where they strike: at A 1 and B1 on
O1’s train, and at A 2 and B2 on O2’s train, Fig. 26–5a. For simplicity, we assume that
O1 is exactly halfway between A 1 and B1 , and O2 is halfway between A 2 and B2 .
Let us first put ourselves in O2’s reference frame, so we observe O1 moving to the
right with speed v. Let us also assume that the two events occur simultaneously in O2’s
frame, and just at the instant when O1 and O2 are opposite each other, Fig. 26–5a.
A short time later, Fig. 26–5b, light from A 2 and from B2 reach O2 at the same time
(we assumed this). Since O2 knows (or measures) the distances O2A 2 and O2B2
as equal, O2 knows the two events are simultaneous in the O2 reference frame.

FIGURE 26–5 Thought experiment on simultaneity. In both


(a) and (b) we are in the reference frame of observer O2 ,
who sees the reference frame of O1 moving to the right.
A1 O1 B1 vB
In (a), one lightning bolt strikes the two reference frames at
A 1 and A 2 , and a second lightning bolt strikes at B1 and B2 . B2
A2 O2
(b) A moment later, the light (shown in blue) from the two
events reaches O2 at the same time. So according to
(a)
observer O2 , the two bolts of lightning struck simultaneously.
But in O1’s reference frame, the light from B1 has already
reached O1 , whereas the light from A 1 has not yet reached O1 . A1 O1 B1 vB
So in O1’s reference frame, the event at B1 must have
preceded the event at A 1 . Simultaneity in time is not A2 B2
O2
absolute.
(b)
But what does observer O1 observe and measure? From our AO2 B reference
frame, we can predict what O1 will observe. We see that O1 moves to the right
during the time the light is traveling to O1 from A 1 and B1 . As shown in Fig. 26–5b,
we can see from our O2 reference frame that the light from B1 has already
passed O1 , whereas the light from A 1 has not yet reached O1 . That is, O1 observes
the light coming from B1 before observing the light coming from A 1 . Given
(i) that light travels at the same speed c in any direction and in any reference
frame, and (ii) that the distance O1 A 1 equals O1 B1 , then observer O1 can only
conclude that the event at B1 occurred before the event at A 1 . The two events are
not simultaneous for O1 , even though they are for O2 .
We thus find that two events which take place at different locations and are
simultaneous to one observer, are actually not simultaneous to a second observer
who moves relative to the first.
It may be tempting to ask: “Which observer is right, O1 or O2?” The answer,
according to relativity, is that they are both right. There is no “best” reference
frame we can choose to determine which observer is right. Both frames are
equally good. We can only conclude that simultaneity is not an absolute concept,
but is relative. We are not aware of this lack of agreement on simultaneity in
everyday life because the effect is noticeable only when the relative speed of the
two reference frames is very large (near c), or the distances involved are very large.

26–4 Time Dilation and the Twin Paradox


The fact that two events simultaneous to one observer may not be simultaneous
to a second observer suggests that time itself is not absolute. Could it be that time
passes differently in one reference frame than in another? This is, indeed, just what
Einstein’s theory of relativity predicts, as the following thought experiment shows.
Figure 26–6 shows a spaceship traveling past Earth at high speed. The point
of view of an observer on the spaceship is shown in part (a), and that of an observer
on Earth in part (b). Both observers have accurate clocks. The person on the space-
ship (Fig. 26–6a) flashes a light and measures the time it takes the light to travel
directly across the spaceship and return after reflecting from a mirror (the rays are
drawn at a slight angle for clarity). In the reference frame of the spaceship, the

750 CHAPTER 26 The Special Theory of Relativity


Mirror

D
Light
source
(a) Receiver Clock timer FIGURE 26–6 Time dilation can be
shown by a thought experiment: the
time it takes for light to travel across
D2 a spaceship and back is longer for the
⫹l2 observer on Earth (b) than for the
2 D
2 ⫹l observer on the spaceship (a).
D

l l

(b) Earth

light travels a distance 2D at speed c, Fig. 26–6a; so the time required to go across
and back, ¢ t0 , is
2D .
¢ t0 =
c
The observer on Earth, Fig. 26–6b, observes the same process. But to this
observer, the spaceship is moving. So the light travels the diagonal path shown
going across the spaceship, reflecting off the mirror, and returning to the sender.
Although the light travels at the same speed to this observer (the second postulate),
it travels a greater distance. Hence the time required, as measured by the observer
on Earth, will be greater than that measured by the observer on the spaceship.
Let us determine the time interval ¢ t measured by the observer on Earth
between sending and receiving the light. In time ¢ t, the spaceship travels
a distance 2l = v ¢ t where v is the speed of the spaceship (Fig. 26–6b). The
light travels a total distance on its diagonal path (Pythagorean theorem) of
2 2D2 + l2 = c ¢ t, where l = v ¢ t兾2. Therefore
c ¢ t = 2 3D2 + l2 = 2 3D2 + v2(¢ t)2兾4 .
We square both sides to find c2(¢ t)2 = 4D2 + v2(¢ t)2, and solve for (¢ t)2 :
(¢t)2 = 4D2兾 Ac2 - v2 B
so
2D
¢t = .
c 31 - v2兾c2
We combine this equation for ¢ t with the formula for ¢ t0 above, ¢ t0 = 2D兾c :
¢ t0
¢t = . (26–1a) TIME DILATION
31 - v 兾c
2 2

Since 31 - v2兾c2 is always less than 1, we see that ¢ t 7 ¢ t0 . That is, the time
interval between the two events (the sending of the light, and its reception on the
spaceship) is greater for the observer on Earth than for the observer on the space-
ship. This is a general result of the theory of relativity, and is known as time dilation.
The time dilation effect can be stated as
clocks moving relative to an observer are measured to run more slowly, as
compared to clocks at rest.
However, we should not think that the clocks are somehow at fault.
Time is actually measured to pass more slowly in any moving reference frame
as compared to your own.
This remarkable result is an inevitable outcome of the two postulates of the special
theory of relativity.

SECTION 26–4 Time Dilation and the Twin Paradox 751


The factor 1兾31 - v2兾c2 occurs so often in relativity that we often give it
the shorthand symbol g (the Greek letter “gamma”), and write Eq. 26–1a as
¢ t = g ¢ t0 (26–1b)
where
1
g = . (26–2)
3 1 - v 2
兾c 2

Note that g is never less than one, and has no units. At normal speeds, g = 1 to
many decimal places. In general, g ⱖ 1.
TABLE 26–1 Values of G Values for g = 1兾 21 - v2兾c2 at a few speeds v are given in Table 26–1.
g is never less than 1.00 and exceeds 1.00 significantly only at very high speeds,
£ G
much above let’s say 106 m兾s (for which g = 1.000006 ).
0 1.00000 p The concept of time dilation may be hard to accept, for it contradicts our
0.01c 1.00005 experience. We can see from Eq. 26–1 that the time dilation effect is indeed negli-
0.10c 1.005 gible unless v is reasonably close to c. If v is much less than c, then the term v2兾c2
0.50c 1.15 is much smaller than the 1 in the denominator of Eq. 26–1, and then ¢ t L ¢ t0
0.90c 2.3 (see Example 26–2). The speeds we experience in everyday life are much smaller
0.99c 7.1 than c, so it is little wonder we don’t ordinarily notice time dilation. But experiments
that have tested the time dilation effect have confirmed Einstein’s predictions.
In 1971, for example, extremely precise atomic clocks were flown around the Earth
in jet planes. The speed of the planes A103 km兾hB was much less than c, so the clocks
had to be accurate to nanoseconds A10 –9 sB in order to detect any time dilation.
They were this accurate, and they confirmed Eqs. 26–1 to within experimental error.
Time dilation had been confirmed decades earlier, however, by observations on
“elementary particles” which have very small masses (typically 10–30 to 10 –27 kg) and
so require little energy to be accelerated to speeds close to the speed of light, c.
Many of these elementary particles are not stable and decay after a time into
lighter particles. One example is the muon, whose mean lifetime is 2.2 ms when at
rest. Careful experiments showed that when a muon is traveling at high speeds,
its lifetime is measured to be longer than when it is at rest, just as predicted by the
time dilation formula.

EXAMPLE 26;1 Lifetime of a moving muon. (a) What will be the mean
lifetime of a muon as measured in the laboratory if it is traveling at
v = 0.60c = 1.80 * 108 m兾s with respect to the laboratory? A muon’s mean
lifetime at rest is 2.20 ms = 2.20 * 10 –6 s. (b) How far does a muon travel in the
laboratory, on average, before decaying?
APPROACH If an observer were to move along with the muon (the muon would
be at rest to this observer), the muon would have a mean life of 2.20 * 10 –6 s. To
an observer in the lab, the muon lives longer because of time dilation. We find the
mean lifetime using Eq. 26–1 and the average distance using d = v ¢ t.
SOLUTION (a) From Eq. 26–1 with v = 0.60c, we have
¢ t0
¢t =
31 - v2兾c2
2.20 * 10 –6 s 2.20 * 10 –6 s
= = = 2.8 * 10–6 s.
31 - 0.36c 兾c 2 2 20.64
(b) Relativity predicts that a muon with speed 1.80 * 108 m兾s would travel an
average distance d = v ¢t = A1.80 * 108 m兾sBA2.8 * 10 –6 sB = 500 m, and this
is the distance that is measured experimentally in the laboratory.
NOTE At a speed of 1.8 * 108 m兾s, classical physics would tell us that
with a mean life of 2.2 ms, an average muon would travel d = vt =
A1.8 * 108 m兾sBA2.2 * 10–6 sB = 400 m. This is shorter than the distance measured.

EXERCISE A What is the muon’s mean lifetime (Example 26–1) if it is traveling at


v = 0.90c? (a) 0.42 ms; (b) 2.3 ms; (c) 5.0 ms; (d) 5.3 ms; (e) 12.0 ms.

752 CHAPTER 26 The Special Theory of Relativity


We need to clarify how to use Eq. 26–1, ¢ t = g ¢ t0 , and the meaning of ¢ t
and ¢ t0 . The equation is true only when ¢ t0 represents the time interval between CAUTION
the two events in a reference frame where an observer at rest sees the two events Proper time ¢t0 is for 2 events at
occur at the same point in space (as in Fig. 26–6a where the two events are the the same point in space
light flash being sent and being received). This time interval, ¢ t0 , is called the
proper time. Then ¢ t in Eqs. 26–1 represents the time interval between the two
events as measured in a reference frame moving with speed v with respect to the
first. In Example 26–1 above, ¢ t0 (and not ¢ t) was set equal to 2.2 * 10 –6 s
because it is only in the rest frame of the muon that the two events (“birth” and
“decay”) occur at the same point in space. The proper time ¢ t0 is the shortest time CAUTION
between the events any observer can measure. In any other moving reference Proper time is shortest:
frame, the time ¢ t is greater. ¢ t 7 ¢ t0

EXAMPLE 26;2 Time dilation at 100 km/h. Let us check time dilation for
everyday speeds. A car traveling 100 km兾h covers a certain distance in 10.00 s
according to the driver’s watch. What does an observer at rest on Earth measure
for the time interval?
APPROACH The car’s speed relative to Earth, written in meters per second,
is 100 km兾h = A1.00 * 105 mB兾A3600 sB = 27.8 m兾s. The driver is at rest in the
reference frame of the car, so we set ¢ t0 = 10.00 s in the time dilation
formula.
SOLUTION We use Eq. 26–1a:
¢ t0 10.00 s
¢t = =
v2 27.8 m兾s 2
1 - 1 - ¢ ≤
C c2 C 3.00 * 10 m兾s
8

10.00 s .
=
–15
31 - A8.59 * 10 B
If you put these numbers into a calculator, you will obtain ¢ t = 10.00 s, because
the denominator differs from 1 by such a tiny amount. The time measured by an
observer fixed on Earth would show no difference from that measured by the
driver, even with the best instruments. A computer that could calculate to a large
number of decimal places would reveal a slight difference between ¢ t and ¢ t0 .
NOTE We can estimate the difference using the binomial expansion (Appendix A–5), P R O B L E M S O LV I N G
Use of the binomial expansion
(16x)n L 16nx. [for x V 1]
1
In our time dilation formula, we have the factor g = A1 - v2兾c2 B – 2. Thus†
1
v2 – 2 1 v2
¢ t = g ¢ t0 = ¢ t0 ¢ 1 - 2 ≤ L ¢ t0 ¢ 1 + ≤
c 2 c2
2
1 27.8 m兾s
L 10.00 s c1 + ¢ ≤ d
2 3.00 * 10 m兾s
8

L 10.00 s + 4 * 10 –14 s.

So the difference between ¢ t and ¢ t0 is predicted to be 4 * 10 –14 s, an extremely


small amount.

EXERCISE B A certain atomic clock keeps precise time on Earth. If the clock is taken
on a spaceship traveling at a speed v = 0.60c, does this clock now run slow according to
the people (a) on the spaceship, (b) on Earth?


Recall that 1兾xn is written as x –n, such as 1兾x2 = x –2, Appendix A–2.

SECTION 26–4 Time Dilation and the Twin Paradox 753


EXAMPLE 26;3 Reading a magazine on a spaceship. A passenger on a
fictional high-speed spaceship traveling between Earth and Jupiter at a steady
speed of 0.75c reads a magazine which takes 10.0 min according to her watch.
(a) How long does this take as measured by Earth-based clocks? (b) How much
farther is the spaceship from Earth at the end of reading the article than it
was at the beginning?
APPROACH (a) The time interval in one reference frame is related to the time
interval in the other by Eq. 26–1a or b. (b) At constant speed, distance is
speed * time. Because there are two time intervals ( ¢ t and ¢ t0) we will get
two distances, one for each reference frame. [This surprising result is explored
in the next Section (26–5).]
SOLUTION (a) The given 10.0-min time interval is the proper time ¢ t0—starting
and finishing the magazine happen at the same place on the spaceship. Earth
clocks measure
¢ t0 10.00 min
¢t = = = 15.1 min.
31 - Av 兾c B
2 2
31 - (0.75)2
(b) In the Earth frame, the rocket travels a distance D = v ¢ t =
(0.75c)(15.1 min) = (0.75)A3.0 * 108 m兾sB(15.1 min * 60 s兾min) = 2.04 * 1011 m.
In the spaceship’s frame, the Earth is moving away from the spaceship at 0.75c,
but the time is only 10.0 min, so the distance is measured to be D0 = v ¢t0 =
(2.25 * 108 m兾s)(600 s) = 1.35 * 1011 m.

Space Travel?
Time dilation has aroused interesting speculation about space travel. According
to classical (Newtonian) physics, to reach a star 100 light-years away would not
be possible for ordinary mortals (1 light-year is the distance light can travel
in 1 year = 3.0 * 108 m兾s * 3.16 * 107 s = 9.5 * 1015 m ). Even if a spaceship
could travel at close to the speed of light, it would take over 100 years to reach
such a star. But time dilation tells us that the time involved could be less. In
a spaceship traveling at v = 0.999c, the time for such a trip would be only about
¢ t0 = ¢ t 21 - v2兾c2 = (100 yr) 21 - (0.999)2 = 4.5 yr. Thus time dilation
allows such a trip, but the enormous practical problems of achieving such speeds
may not be possible to overcome, certainly not in the near future.
When we talk in this Chapter and in the Problems about spaceships moving
at speeds close to c, it is for understanding and for fun, but not realistic, although
for tiny elementary particles such high speeds are realistic.
In this example, 100 years would pass on Earth, whereas only 4.5 years would
pass for the astronaut on the trip. Is it just the clocks that would slow down for the
astronaut? No.
All processes, including aging and other life processes, run more slowly for
the astronaut as measured by an Earth observer. But to the astronaut, time
would pass in a normal way.
The astronaut would experience 4.5 years of normal sleeping, eating, reading,
and so on. And people on Earth would experience 100 years of ordinary activity.

Twin Paradox
Not long after Einstein proposed the special theory of relativity, an apparent
paradox was pointed out. According to this twin paradox, suppose one of a pair
of 20-year-old twins takes off in a spaceship traveling at very high speed to a
distant star and back again, while the other twin remains on Earth. According to
the Earth twin, the astronaut twin will age less. Whereas 20 years might pass for
the Earth twin, perhaps only 1 year (depending on the spacecraft’s speed) would
pass for the traveler. Thus, when the traveler returns, the earthbound twin could
expect to be 40 years old whereas the traveling twin would be only 21.

754 CHAPTER 26 The Special Theory of Relativity


This is the viewpoint of the twin on the Earth. But what about the traveling
twin? If all inertial reference frames are equally good, won’t the traveling twin
make all the claims the Earth twin does, only in reverse? Can’t the astronaut twin
claim that since the Earth is moving away at high speed, time passes more slowly
on Earth and the twin on Earth will age less? This is the opposite of what the
Earth twin predicts. They cannot both be right, for after all the spacecraft returns
to Earth and a direct comparison of ages and clocks can be made.
There is, however, no contradiction here. One of the viewpoints is indeed
incorrect. The consequences of the special theory of relativity—in this case, time
dilation—can be applied only by observers in an inertial reference frame. The
Earth is such a frame (or nearly so), whereas the spacecraft is not. The spacecraft
accelerates at the start and end of its trip and when it turns around at the far
point of its journey. Part of the time, the astronaut twin may be in an inertial
frame (and is justified in saying the Earth twin’s clocks run slow). But during the
accelerations, the twin on the spacecraft is not in an inertial frame. So she cannot
use special relativity to predict their relative ages when she returns to Earth. The
Earth twin stays in the same inertial frame, and we can thus trust her predictions
based on special relativity. Thus, there is no paradox. The prediction of the Earth
twin that the traveling twin ages less is the correct one.
* Global Positioning System (GPS) PHYSICS APPLIED
Global positioning system
Airplanes, cars, boats, and hikers use global positioning system (GPS) receivers (GPS)
to tell them quite accurately where they are at a given moment (Fig. 26–7). There are
more than 30 global positioning system satellites that send out precise time signals FIGURE 26–7 A visiting professor
using atomic clocks. Your receiver compares the times received from at least four of physics uses the GPS on her smart
satellites, all of whose times are carefully synchronized to within 1 part in 1013. phone to find a restaurant (red dot).
By comparing the time differences with the known satellite positions and the Her location in the physics
fixed speed of light, the receiver can determine how far it is from each satellite department is the blue dot. Traffic
on some streets is also shown
and thus where it is on the Earth. It can do this to an accuracy of a few meters, if
(green = good, orange = slow,
it has been constructed to make corrections such as the one below due to relativity. red = heavy traffic) which comes in
CONCEPTUAL EXAMPLE 26;4 A relativity correction to GPS. GPS part by tracking cell phone
satellites move at about 4 km兾s = 4000 m兾s. Show that a good GPS receiver movements.
needs to correct for time dilation if it is to produce results consistent with atomic
clocks accurate to 1 part in 1013.
RESPONSE Let us calculate the magnitude of the time dilation effect by
inserting v = 4000 m兾s into Eq. 26–1a:
1 1
¢t = ¢ t0 = ¢ t0
v 2
4 * 103 m兾s 2
1 - 2 1 - ¢ ≤
C c C 3 * 108 m兾s
1
= ¢ t0 .
–10
31 - 1.8 * 10
We use the binomial expansion: (16x)n L 16nx for x V 1 (see Appendix A–5)
- 12
which here is (1 - x) L 1 + 12 x. That is
¢ t = A1 + 1
2 A1.8 * 10–10 BB ¢ t0 = A1 + 9 * 10–11 B ¢ t0 .
The time “error” divided by the time interval is
A¢ t - ¢ t0 B
= 1 + 9 * 10 –11 - 1 = 9 * 10 –11 L 1 * 10 –10.
¢ t0
Time dilation, if not accounted for, would introduce an error of about 1 part
in 1010, which is 1000 times greater than the precision of the atomic clocks. Not
correcting for time dilation means a receiver could give much poorer position
accuracy.
NOTE GPS devices must make other corrections as well, including effects
associated with general relativity.
SECTION 26–4 Time Dilation and the Twin Paradox 755
26–5 Length Contraction
Time intervals are not the only things different in different reference frames.
Space intervals—lengths and distances—are different as well, according to the
special theory of relativity, and we illustrate this with a thought experiment.

FIGURE 26–8 (a) A spaceship traveling v


at very high speed v from Earth to the
Earth
planet Neptune, as seen from Earth’s
(a) Earth at rest Neptune
frame of reference. (b) According to an
observer on the spaceship, Earth and
Neptune are moving at the very high
speed v: Earth leaves the spaceship, and v v
a time ¢ t0 later Neptune arrives at the
spaceship. Earth
(b) Spacecraft at rest Neptune

Observers on Earth watch a spacecraft traveling at speed v from Earth to,


say, Neptune, Fig. 26–8a. The distance between the planets, as measured by the
Earth observers, is l0 . The time required for the trip, measured from Earth, is
l0
¢t = v . [Earth observer]
In Fig. 26–8b we see the point of view of observers on the spacecraft. In this
frame of reference, the spaceship is at rest; Earth and Neptune move† with speed v.
The time between departure of Earth and arrival of Neptune, as observed from the
spacecraft, is the “proper time” ¢ t0 (page 753), because these two events occur
at the same point in space (i.e., at the spacecraft). Therefore the time interval is
less for the spacecraft observers than for the Earth observers. That is, because of
time dilation (Eq. 26–1a), the time for the trip as viewed by the spacecraft is
¢ t0 = ¢ t 31 - v2兾c2
[spacecraft observer]
= ¢ t兾g.
Because the spacecraft observers measure the same speed but less time between
these two events, they also measure the distance as less. If we let l be the distance
between the planets as viewed by the spacecraft observers, then l = v ¢ t0 ,
which we can rewrite as l = v ¢ t0 = v ¢ t 31 - v2兾c2 = l031 - v2兾c2 . Thus
we have the important result that
LENGTH CONTRACTION l = l031 - v2兾c2 (26–3a)

or, using g (Eq. 26–2),


l0
l = g. (26–3b)

This is a general result of the special theory of relativity and applies to lengths of
objects as well as to distance between objects. The result can be stated most
simply in words as:
the length of an object moving relative to an observer is measured to be
shorter along its direction of motion than when it is at rest.
CAUTION This is called length contraction. The length l0 in Eqs. 26–3 is called the
Proper length is measured proper length. It is the length of the object (or distance between two points whose
in reference frame where the positions are measured at the same time) as determined by observers at rest with
two positions are at rest respect to the object. Equations 26–3 give the length l that will be measured by
observers when the object travels past them at speed v.

We assume v is much greater than the relative speed of Neptune and Earth (which we thus ignore).

756 CHAPTER 26 The Special Theory of Relativity


It is important to note that length contraction occurs only along the direction
of motion. For example, the moving spaceship in Fig. 26–8a is shortened in
length, but its height is the same as when it is at rest.
Length contraction, like time dilation, is not noticeable in everyday life
because the factor 21 - v2兾c2 in Eq. 26–3a differs significantly from 1.00 only
when v is very large.
EXAMPLE 26;5 Painting’s contraction. A rectangular painting measures
1.00 m tall and 1.50 m wide, Fig. 26–9a. It is hung on the side wall of a spaceship
which is moving past the Earth at a speed of 0.90c. (a) What are the dimen-
sions of the picture according to the captain of the spaceship? (b) What are the 1.00 m
dimensions as seen by an observer on the Earth?
APPROACH We apply the length contraction formula, Eq. 26–3a, to the dimen- 1.50 m
sion parallel to the motion; v is the speed of the painting relative to the Earth (a)
observer.
SOLUTION (a) The painting is at rest (v = 0) on the spaceship so it (as well
as everything else in the spaceship) looks perfectly normal to everyone on the
spaceship. The captain sees a 1.00-m by 1.50-m painting.
(b) Only the dimension in the direction of motion is shortened, so the height is 1.00 m
unchanged at 1.00 m, Fig. 26–9b. The length, however, is contracted to
v2
l = l0 1 - (b)
?
C c2
= (1.50 m)31 - (0.90)2 = 0.65 m. FIGURE 26–9 Example 26–5.

So the picture has dimensions 1.00 m * 0.65 m to an observer on Earth.

EXAMPLE 26;6 A fantasy supertrain. A very fast train with a “proper


length” of l0 = 500 m (measured by people at rest on the train) is passing through
a tunnel that is 200 m long according to observers on the ground. Let us imagine
the train’s speed to be so great that the train fits completely within the tunnel as
seen by observers on the ground. That is, the engine is just about to emerge from
one end of the tunnel at the time the last car disappears into the other end. What
is the train’s speed?
APPROACH Since the train just fits inside the tunnel, its length measured by
the person on the ground is l = 200 m. The length contraction formula,
Eq. 26–3a or b, can thus be used to solve for v.
SOLUTION Substituting l = 200 m and l0 = 500 m into Eq. 26–3a gives
v2
200 m = 500 m 1 - ;
C c2
dividing both sides by 500 m and squaring, we get
v2
(0.40)2 = 1 -
c2
or
v
= 31 - (0.40)2
c
and
v = 0.92c.
NOTE No real train could go this fast. But it is fun to think about.
NOTE An observer on the train would not see the two ends of the train inside
the tunnel at the same time. Recall that observers moving relative to each other
do not agree about simultaneity. (See Example 26–7, next.)

EXERCISE C What is the length of the tunnel as measured by observers on the train in
Example 26–6?

SECTION 26–5 Length Contraction 757


CONCEPTUAL EXAMPLE 26;7 Resolving the train and tunnel length.
Observers at rest on the Earth see a very fast 200-m-long train pass through a
200-m-long tunnel (as in Example 26–6) so that the train momentarily disappears
from view inside the tunnel. Observers on the train measure the train’s length
to be 500 m and the tunnel’s length to be only 80 m (Exercise C, using Eq. 26–3a).
Clearly a 500-m-long train cannot fit inside an 80-m-long tunnel. How is this
apparent inconsistency explained?
RESPONSE Events simultaneous in one reference frame may not be simultane-
ous in another. Let the engine emerging from one end of the tunnel be “event A,”
and the last car disappearing into the other end of the tunnel “event B.” To
observers in the Earth frame, events A and B are simultaneous. To observers on
the train, however, the events are not simultaneous. In the train’s frame, event A
occurs before event B. As the engine emerges from the tunnel, observers on the
train observe the last car as still 500 m - 80 m = 420 m from the entrance to
the tunnel.

26–6 Four-Dimensional Space–Time


Let us imagine a person is on a train moving at a very high speed, say 0.65c,
Fig. 26–10. This person begins a meal at 7:00 and finishes at 7:15, according to
a clock on the train. The two events, beginning and ending the meal, take place at
the same point on the train, so the “proper time” between these two events is
15 min. To observers on Earth, the plate is moving and the meal will take
longer—20 min according to Eqs. 26–1. Let us assume that the meal was served
on a 20-cm-diameter plate (its “proper length”). To observers on the Earth, the
plate is moving and is only 15 cm wide (length contraction). Thus, to observers on
the Earth, the meal looks smaller but lasts longer.

7 7

FIGURE 26–10 According to an


accurate clock on a fast-moving train,
a person (a) begins dinner at 7:00 and
(b) finishes at 7:15. At the beginning
of the meal, two observers on Earth
set their watches to correspond with
the clock on the train. These observers
measure the eating time as 20 minutes.

(a) (b)

In a sense the two effects, time dilation and length contraction, balance each
other. When viewed from the Earth, what an object seems to lose in size it gains
in length of time it lasts. Space, or length, is exchanged for time.
Considerations like this led to the idea of four-dimensional space–time:
space takes up three dimensions and time is a fourth dimension. Space and time
are intimately connected. Just as when we squeeze a balloon we make one
dimension larger and another smaller, so when we examine objects and events
from different reference frames, a certain amount of space is exchanged for time,
or vice versa.

758 CHAPTER 26 The Special Theory of Relativity


Although the idea of four dimensions may seem strange, it refers to the idea
that any object or event is specified by four quantities—three to describe where
in space, and one to describe when in time. The really unusual aspect of four-
dimensional space–time is that space and time can intermix: a little of one can be
exchanged for a little of the other when the reference frame is changed.
[In Galilean–Newtonian relativity, the time interval between two events, ¢ t,
and the distance between two events or points, ¢x, are invariant quantities no
matter what inertial reference frame they are viewed from. Neither of these
quantities is invariant according to Einstein’s relativity. But there is an invariant
quantity in four-dimensional space–time, called the space–time interval, which is
(¢s)2 = (c ¢ t)2 - (¢x)2.]

26–7 Relativistic Momentum


So far in this Chapter, we have seen that two basic mechanical quantities, length
and time intervals, need modification because they are relative—their value
depends on the reference frame from which they are measured. We might expect
that other physical quantities might need some modification according to the
theory of relativity, such as momentum and energy.
The analysis of collisions between two particles shows that if we want to
preserve the law of conservation of momentum in relativity, we must redefine
momentum as
mv
p = = gmv. (26–4)
31 - v 兾c
2 2

Here g is shorthand for 1兾31 - v2兾c2 as before (Eq. 26–2). For speeds much
less than the speed of light, Eq. 26–4 gives the classical momentum, p = mv.
Relativistic momentum has been tested many times on tiny elementary particles
(such as muons), and it has been found to behave in accord with Eq. 26–4.

EXAMPLE 26;8 Momentum of moving electron. Compare the momentum


of an electron to its classical value when it has a speed of (a) 4.00 * 107 m兾s in
the CRT of an old TV set, and (b) 0.98c in an accelerator used for cancer therapy.
APPROACH We use Eq. 26–4 for the momentum of a moving electron.
SOLUTION (a) At v = 4.00 * 107 m兾s, the electron’s momentum is
mv mv
p = = = 1.01mv.
v2 A4.00 * 107 m兾sB 2
1 - 2 1 -
C c C A3.00 * 108 m兾sB 2

The factor g = 1兾31 - v2兾c2 L 1.01, so the momentum is only about 1%


greater than the classical value. (If we put in the mass of an electron,
m = 9.11 * 10 –31 kg, the momentum is p = 1.01mv = 3.68 * 10 –23 kg⭈m兾s,
compared to 3.64 * 10 –23 kg⭈m兾s classically.)
(b) With v = 0.98c, the momentum is
mv mv mv
p = = = = 5.0mv.
(0.98c) 31 - (0.98)
2 2
v 2
1 - 2 1 -
C c C c2

An electron traveling at 98% the speed of light has g = 5.0 and a momentum
5.0 times its classical value.

SECTION 26–7 Relativistic Momentum 759


* Rest Mass and Relativistic Mass
CAUTION The relativistic definition of momentum, Eq. 26–4, has sometimes been inter-
Most physicists prefer preted as an increase in the mass of an object. In this interpretation, a particle
to consider the mass can have a relativistic mass, mrel , which increases with speed according to
of a particle as fixed
m .
mrel =
31 - v 兾c
2 2

In this “mass-increase” formula, m is referred to as the rest mass of the object.


With this interpretation, the mass of an object appears to increase as its speed
increases. But there are problems with relativistic mass. If we plug it into formu-
las like F = ma or ke = 12 mv2, we obtain formulas that do not agree with
B
experiment. (If we write Newton’s second law in its more general form, F = ¢p兾¢ t,B

that would get a correct result.) Also, be careful not to think a mass acquires
more particles or more molecules as its speed becomes very large. It doesn’t.
Today, most physicists prefer not to use relativistic mass, so an object has only
one mass (its rest mass), and it is only the momentum that increases with speed.
Whenever we talk about the mass of an object, we will always mean its rest
mass (a fixed value). [But see Problem 46.]

26–8 The Ultimate Speed


A basic result of the special theory of relativity is that the speed of an object
cannot equal or exceed the speed of light. That the speed of light is a natural
speed limit in the universe can be seen from any of Eqs. 26–1, 26–3, or 26–4. It is
perhaps easiest to see from Eq. 26–4. As an object is accelerated to greater and
greater speeds, its momentum becomes larger and larger. Indeed, if v were to
equal c, the denominator in this equation would be zero, and the momentum
would be infinite. To accelerate an object up to v = c would thus require infinite
energy, and so is not possible.

26–9 E ⴝ mc 2; Mass and Energy


If momentum needs to be modified to fit with relativity as we just saw in Eq. 26–4,
then we might expect that energy would also need to be rethought. Indeed, Einstein
not only developed a new formula for kinetic energy, but also found a new rela-
tion between mass and energy, and the startling idea that mass is a form of energy.
We start with the work-energy principle (Chapter 6), hoping it is still valid in
relativity and will give verifiable results. That is, we assume the net work done on
a particle is equal to its change in kinetic energy (ke). Using this principle, Einstein
showed that at high speeds the formula ke = 12 mv2 is not correct. Instead,
Einstein showed that the kinetic energy of a particle of mass m traveling at speed v
is given by
mc2
ke = - mc2. (26–5a)
31 - v 兾c
2 2

In terms of g = 1兾31 - v2兾c2 we can rewrite Eq. 26–5a as

ke = gmc2 - mc2 = (g - 1)mc2. (26–5b)

Equation 26–5a requires some interpretation. The first term increases with the
speed v of the particle. The second term, mc2, is constant; it is called the rest energy
of the particle, and represents a form of energy that a particle has even when at rest.
Note that if a particle is at rest (v = 0) the first term in Eq. 26–5a becomes mc2,
so ke = 0 as it should.

760 CHAPTER 26 The Special Theory of Relativity


We can rearrange Eq. 26–5b to get
gmc2 = mc2 + ke.
We call gmc2 the total energy E of the particle (assuming no potential energy),
because it equals the rest energy plus the kinetic energy:
E = ke + mc2. (26–6a)

The total energy can also be written, using Eqs. 26–5, as
mc2
E = gmc2 = . (26–6b)
31 - v 兾c
2 2

For a particle at rest in a given reference frame, ke is zero in Eq. 26–6a, so


the total energy is its rest energy:
MASS RELATED
E = mc2. (26–7) TO ENERGY
Here we have Einstein’s famous formula, E = mc2. This formula mathematically
relates the concepts of energy and mass. But if this idea is to have any physical
meaning, then mass ought to be convertible to other forms of energy and vice
versa. Einstein suggested that this might be possible, and indeed changes of mass
to other forms of energy, and vice versa, have been experimentally confirmed
countless times in nuclear and elementary particle physics. For example, an elec-
tron and a positron (= a positive electron, see Section 32–3) have often been
observed to collide and disappear, producing pure electromagnetic radiation. The
amount of electromagnetic energy produced is found to be exactly equal to that
predicted by Einstein’s formula, E = mc2. The reverse process is also commonly
observed in the laboratory: electromagnetic radiation under certain conditions can
be converted into material particles such as electrons (see Section 27–6 on pair
production). On a larger scale, the energy produced in nuclear power plants is
a result of the loss in mass of the uranium fuel as it undergoes the process called
fission (Chapter 31). Even the radiant energy we receive from the Sun is an
example of E = mc2; the Sun’s mass is continually decreasing as it radiates
electromagnetic energy outward.
The relation E = mc2 is now believed to apply to all processes, although the
changes are often too small to measure. That is, when the energy of a system
changes by an amount ¢E, the mass of the system changes by an amount ¢m
given by
¢E = (¢m)Ac2 B. (26–8)
In a nuclear reaction where an energy E is required or released, the masses of the
reactants and the products will be different by ¢m = ¢E兾c2.

EXAMPLE 26;9 Pion’s kinetic energy. A p0 meson Am = 2.4 * 10 –28 kgB


travels at a speed v = 0.80c = 2.4 * 108 m兾s. What is its kinetic energy?
Compare to a classical calculation.
APPROACH We use Eq. 26–5 and compare to 12 mv2.
SOLUTION We substitute values into Eq. 26–5a
1
ke = mc2 ¢ - 1≤
31 - v 兾c
2 2
P R O B L E M S O LV I N G
1
= A2.4 * 10 –28 kgBA3.0 * 108 m兾sB 2 ¢ 1 - 1≤ Relativistic kinetic energy
(1 - 0.64)2
= 1.4 * 10 –11 J.
Notice that the units of mc2 are kg⭈m2兾s2, which is the joule.
NOTE Classically ke = 12 mv2 = 12 A2.4 * 10 –28 kgB A2.4 * 108 m兾sB 2 = 6.9 * 10 –12 J,
about half as much, but this is not a correct result. Note that 12 gmv2 also does
not work.


This is for a “free particle,” without forces and potential energy. Potential energy terms can be added.

SECTION 26–9 E = mc 2; Mass and Energy 761


EXAMPLE 26;10 Energy from nuclear decay. The energy required or
released in nuclear reactions and decays comes from a change in mass between
the initial and final particles. In one type of radioactive decay (Chapter 30),
an atom of uranium (m = 232.03716 u) decays to an atom of thorium
(m = 228.02874 u) plus an atom of helium (m = 4.00260 u) where the masses
given are in atomic mass units A1 u = 1.6605 * 10 –27 kgB. Calculate the energy
released in this decay.
APPROACH The initial mass minus the total final mass gives the mass loss in
atomic mass units (u); we convert that to kg, and multiply by c2 to find the
energy released, ¢E = ¢m c 2.
SOLUTION The initial mass is 232.03716 u, and after the decay the mass is
228.02874 u + 4.00260 u = 232.03134 u, so there is a loss of mass of 0.00582 u.
This mass, which equals (0.00582 u)A1.66 * 10 –27 kgB = 9.66 * 10 –30 kg, is
changed into energy. By ¢E = ¢m c2, we have

¢E = A9.66 * 10 –30 kgBA3.0 * 108 m兾sB 2


= 8.70 * 10–13 J.

Since 1 MeV = 1.60 * 10 –13 J (Section 17–4), the energy released is 5.4 MeV.

In the tiny world of atoms and nuclei, it is common to quote energies in eV


(electron volts) or multiples such as MeV A106 eVB. Momentum (see Eq. 26–4)
can be quoted in units of eV兾c (or MeV兾c). And mass can be quoted (from
E = mc2 ) in units of eV兾c2 (or MeV兾c2). Note the use of c to keep the units
correct. The masses of the electron and the proton can be shown to be
0.511 MeV兾c2 and 938 MeV兾c2, respectively. For example, for the electron,
mc2 = A9.11 * 10 –31 kgBA2.998 * 108 m兾sB 2兾A1.602 * 10 –13 J兾MeVB = 0.511 MeV.
See also the Table inside the front cover.

EXAMPLE 26;11 A 1-TeV proton. The Tevatron accelerator at Fermilab in


Illinois can accelerate protons to a kinetic energy of 1.0 TeV A1012 eVB. What is the
speed of such a proton?
APPROACH We solve the kinetic energy formula, Eq. 26–5a, for v.
SOLUTION The rest energy of a proton is mc2 = 938 MeV or 9.38 * 108 eV.
Compared to the kinetic energy of 1012 eV, the rest energy can be neglected, so
we simplify Eq. 26–5a to
mc2 .
ke L
31 - v 兾c
2 2

We solve this for v in the following steps:

v2 mc2
1 - = ;
C c2 ke
v2 mc2 2
1 - = ¢ ≤ ;
c2 ke
v2 mc2 2 9.38 * 108 eV 2
= 1 - ¢ ≤ = 1 - ¢ ≤ ;
c 2 ke 1.0 * 1012 eV

v = 31 - A9.38 * 10–4 B 2 c
= 0.99999956 c.

So the proton is traveling at a speed very nearly equal to c.

762 CHAPTER 26 The Special Theory of Relativity


At low speeds, v V c, the relativistic formula for kinetic energy reduces to
the classical one, as we now show by using the binomial expansion (Appendix A):
(16x)n = 16nx + p , keeping only two terms because x = v兾c is very
much less than 1. With n = – 12 we expand the square root in Eq. 26–5a
1
ke = mc2 ¢ - 1≤
31 - v 兾c
2 2

so that
1 v2
ke L mc2 ¢ 1 + + p - 1≤ L 2 mv .
1 2
2 c2
The dots in the first expression represent very small terms in the expansion which
we neglect since we assumed that v V c. Thus at low speeds, the relativistic
form for kinetic energy reduces to the classical form, ke = 12 mv2. This makes
relativity a viable theory in that it can predict accurate results at low speed as well
as at high. Indeed, the other equations of special relativity also reduce to their
classical equivalents at ordinary speeds: length contraction, time dilation, and
modifications to momentum as well as kinetic energy, all disappear for v V c
since 31 - v2兾c2 L 1.
A useful relation between the total energy E of a particle and its momentum p
can also be derived. The momentum of a particle of mass m and speed v is given
by Eq. 26–4
mv
p = gmv = .
31 - v 兾c
2 2

The total energy is


E = ke + mc2
or
mc2
E = gmc2 = .
31 - v 兾c
2 2

We square this equation (and we insert “v2 - v2” which is zero, but will help us):
m2c2c2 m2c2 Ac2 - v2 + v2 B m2c2v2 m2c2 Ac2 - v2 B
E2 = = = +
1 - v2兾c2 1 - v2兾c2 1 - v2兾c2 1 - v2兾c2
m2c4 A1 - v2兾c2 B
= p2c2 +
1 - v2兾c2
or
E 2 = p2c2 + m2c4. (26–9)
Thus, the total energy can be written in terms of the momentum p, or in terms of
the kinetic energy (Eq. 26–6a), where we have assumed there is no potential energy.

* Invariant Energy–Momentum
We can rewrite Eq. 26–9 as E 2 - p2c2 = m2c4. Since the mass m of a given
particle is the same in any reference frame, we see that the quantity E 2 - p2c2
must also be the same in any reference frame. Thus, at any given moment the
total energy E and momentum p of a particle will be different in different
reference frames, but the quantity E 2 - p2c2 will have the same value in all
inertial reference frames. We say that the quantity E 2 - p2c2 is invariant.

When Do We Use Relativistic Formulas?


From a practical point of view, we do not have much opportunity in our daily lives
to use the mathematics of relativity. For example, the g factor, g = 1兾 21 - v2兾c2,
has a value of 1.005 when v = 0.10c. Thus, for speeds even as high as
0.10c = 3.0 * 107 m兾s, the factor 21 - v2兾c2 in relativistic formulas gives
a numerical correction of less than 1%. For speeds less than 0.10c, or unless
mass and energy are interchanged, we don’t usually need the more complicated
relativistic formulas, and can use the simpler classical formulas.

SECTION 26–9 E = mc 2; Mass and Energy 763


If you are given a particle’s mass m and its kinetic energy ke, you can do a
quick calculation to determine if you need to use relativistic formulas or if classical
ones are good enough. You simply compute the ratio ke兾mc2 because (Eq. 26–5b)
ke 1
= g - 1 = - 1.
mc2 31 - v 兾c
2 2

If this ratio comes out to be less than, say, 0.01, then g ⱕ 1.01 and relativistic
equations will correct the classical ones by about 1%. If your expected precision
is no better than 1%, classical formulas are good enough. But if your precision is
1 part in 1000 (0.1%) then you would want to use relativistic formulas. If your
expected precision is only 10%, you need relativity if A ke兾mc2 B g 0.1.
EXERCISE D For 1% accuracy, does an electron with ke = 100 eV need to be treated
relativistically? [Hint: The mass of an electron is 0.511 MeV.]

26–10 Relativistic Addition of Velocities


Consider a rocket ship that travels away from the Earth with speed v, and assume
that this rocket has fired off a second rocket that travels at speed u¿ with
respect to the first (Fig. 26–11). We might expect that the speed u of rocket 2
with respect to Earth is u = v + u¿, which in the case shown in Fig. 26–11 is
CAUTION
u = 0.60c + 0.60c = 1.20c. But, as discussed in Section 26–8, no object can travel
Relative velocities do
faster than the speed of light in any reference frame. Indeed, Einstein showed
not add simply, as that since length and time are different in different reference frames, the classical
in classical mechanics (v V c) addition-of-velocities formula is no longer valid. Instead, the correct formula is
Relativistic addition of velocities v + u¿ B B
u and v along
u = c d (26–10)
1 + vu¿兾c
B B
formula (u and v along same line) 2 the same direction
for motion along a straight line. We derive this formula in Appendix E. If u¿ is in
u' = 0.60c with the opposite direction from v, then u¿ must have a minus sign in the above equation
2
respect to so u = (v - u¿)兾A1 - vu¿兾c2 B.
rocket 1
EXAMPLE 26;12 Relative velocity, relativistically. Calculate the speed of
1
rocket 2 in Fig. 26–11 with respect to Earth.
APPROACH We combine the speed of rocket 2 relative to rocket 1 with the
Earth v = 0.60c with speed of rocket 1 relative to Earth, using the relativistic Eq. 26–10 because the
respect to Earth speeds are high and they are along the same line.
SOLUTION Rocket 2 moves with speed u¿ = 0.60c with respect to rocket 1.
Rocket 1 has speed v = 0.60c with respect to Earth. The speed of rocket 2 with
respect to Earth is (Eq. 26–10)
0.60c + 0.60c 1.20c
u = = = 0.88c.
(0.60c)(0.60c) 1.36
FIGURE 26–11 Rocket 1 leaves Earth 1 +
at v = 0.60c. Rocket 2 is fired from c2
rocket 1 with speed u¿ = 0.60c. What NOTE The speed of rocket 2 relative to Earth is less than c, as it must be.
is the speed of rocket 2 with respect to
the Earth? Example 26–12. We can see that Eq. 26–10 reduces to the classical form for velocities small
compared to the speed of light since 1 + vu¿兾c2 L 1 for v and u¿ V c. Thus,
u L v + u¿, as in classical physics (Chapter 3).
Let us test our formula at the other extreme, that of the speed of light.
Suppose that rocket 1 in Fig. 26–11 sends out a beam of light so that u¿ = c.
Equation 26–10 tells us that the speed of this light relative to Earth is
0.60c + c 1.60c
u = = = c,
(0.60c)(c) 1.60
1 +
c2
which is fully consistent with the second postulate of relativity.
EXERCISE E Use Eq. 26–10 to calculate the speed of rocket 2 in Fig. 26–11 relative to
Earth if it was shot from rocket 1 at a speed u¿ = 3000 km兾s = 0.010c. Assume rocket 1
had a speed v = 6000 km兾s = 0.020c.
EXERCISE F Return to the Chapter-Opening Question, page 744, and answer it again
764 CHAPTER 26 now. Try to explain why you may have answered differently the first time.
26–11 The Impact of Special Relativity
A great many experiments have been performed to test the predictions of the special
theory of relativity. Within experimental error, no contradictions have been found.
Scientists have therefore accepted relativity as an accurate description of nature.
At speeds much less than the speed of light, the relativistic formulas reduce to
the old classical ones, as we have discussed. We would, of course, hope—or rather,
insist—that this be true since Newtonian mechanics works so well for objects
moving with speeds v V c. This insistence that a more general theory (such as
relativity) give the same results as a more restricted theory (such as classical
mechanics which works for v V c) is called the correspondence principle. The
two theories must correspond where their realms of validity overlap. Relativity
thus does not contradict classical mechanics. Rather, it is a more general theory,
of which classical mechanics is now considered to be a limiting case.
The importance of relativity is not simply that it gives more accurate results,
especially at very high speeds. Much more than that, it has changed the way we
view the world. The concepts of space and time are now seen to be relative, and
intertwined with one another, whereas before they were considered absolute and
separate. Even our concepts of matter and energy have changed: either can be
converted to the other. The impact of relativity extends far beyond physics. It has
influenced the other sciences, and even the world of art and literature; it has,
indeed, entered the general culture.
The special theory of relativity we have studied in this Chapter deals with
inertial (nonaccelerating) reference frames. In Chapter 33 we will discuss briefly
the more complicated “general theory of relativity” which can deal with non-
inertial reference frames.

Summary
An inertial reference frame is one in which Newton’s law of are seen to be intimately connected, with time being the fourth
inertia holds. Inertial reference frames move at constant veloc- dimension in addition to the three dimensions of space.
ity relative to one another. Accelerating reference frames are The momentum of an object is given by
noninertial. mv
The special theory of relativity is based on two principles: p = gmv = . (26–4)
31 - v 兾c
2 2
the relativity principle, which states that the laws of physics
are the same in all inertial reference frames, and the principle Mass and energy are interconvertible. The equation
of the constancy of the speed of light, which states that the
E = mc2 (26–7)
speed of light in empty space has the same value in all inertial
reference frames. tells how much energy E is needed to create a mass m, or vice
One consequence of relativity theory is that two events versa. Said another way, E = mc2 is the amount of energy an
that are simultaneous in one reference frame may not be simul- object has because of its mass m. The law of conservation of
taneous in another. Other effects are time dilation: moving energy must include mass as a form of energy.
clocks are measured to run slow; and length contraction: The kinetic energy ke of an object moving at speed v is
the length of a moving object is measured to be shorter (in its given by
direction of motion) than when it is at rest. Quantitatively, mc2
ke = - mc2 = (g - 1)mc2 (26–5)
¢ t0
31 - v 兾c
2 2
¢t = = g ¢ t0 (26–1)
31 - v 兾c
2 2
where m is the mass of the object. The total energy E, if there
l0
l = l0 31 - v2兾c2 = g (26–3) is no potential energy, is
E = ke + mc2
where l and ¢ t are the length and time interval of objects (or (26–6)
events) observed as they move by at the speed v; l0 and ¢ t0 are = gmc2.
the proper length and proper time—that is, the same quantities The momentum p of an object is related to its total
as measured in the rest frame of the objects or events. The energy E (assuming no potential energy) by
quantity g is shorthand for
E 2 = p2c2 + m2c4. (26–9)
1
g = . (26–2) Velocity addition also must be done in a special way. All
31 - v 兾c
2 2
these relativistic effects are significant only at high speeds,
The theory of relativity has changed our notions of space close to the speed of light, which itself is the ultimate speed in
and time, and of momentum, energy, and mass. Space and time the universe.

Summary 765
Questions
1. You are in a windowless car in an exceptionally smooth 12. Explain how the length contraction and time dilation for-
train moving at constant velocity. Is there any physical experi- mulas might be used to indicate that c is the limiting speed
ment you can do in the train car to determine whether in the universe.
you are moving? Explain. 13. Discuss how our everyday lives would be different if the
2. You might have had the experience of being at a red light speed of light were only 25 m兾s.
when, out of the corner of your eye, you see the car beside 14. The drawing at the start of this Chapter shows the street as
you creep forward. Instinctively you stomp on the brake seen by Mr Tompkins, for whom the speed of light is
pedal, thinking that you are rolling backward. What does c = 20 mi兾h. What does Mr Tompkins look like to the
this say about absolute and relative motion? people standing on the street (Fig. 26–12)? Explain.
3. A worker stands on top of a railroad car moving at constant
velocity and throws a heavy ball straight up (from his point
of view). Ignoring air resistance, explain whether the ball
will land back in his hand or behind him.
4. Does the Earth really go around the Sun? Or is it also valid FIGURE 26–12
to say that the Sun goes around the Earth? Discuss in view Question 14.
of the relativity principle (that there is no best reference Mr Tompkins as
frame). Explain. See Section 5–8. seen by people on
5. If you were on a spaceship traveling at 0.6c away from a the sidewalk. See
star, at what speed would the starlight pass you? also Chapter-
6. The time dilation effect is sometimes expressed as “moving Opening Figure on
clocks run slowly.” Actually, this effect has nothing to do page 744.
with motion affecting the functioning of clocks. What then
does it deal with?
7. Does time dilation mean that time actually passes more
slowly in moving reference frames or that it only seems to
pass more slowly?
8. A young-looking woman astronaut has just arrived home 15. An electron is limited to travel at speeds less than c. Does
from a long trip. She rushes up to an old gray-haired man this put an upper limit on the momentum of an electron?
and in the ensuing conversation refers to him as her son. If so, what is this upper limit? If not, explain.
How might this be possible? 16. Can a particle of nonzero mass attain the speed of light?
9. If you were traveling away from Earth at speed 0.6c, would Explain.
you notice a change in your heartbeat? Would your mass, 17. Does the equation E = mc 2 conflict with the conservation
height, or waistline change? What would observers on of energy principle? Explain.
Earth using telescopes say about you? 18. If mass is a form of energy, does this mean that a spring has
10. Do time dilation and length contraction occur at ordinary more mass when compressed than when relaxed? Explain.
speeds, say 90 km兾h? 19. It is not correct to say that “matter can neither be created
11. Suppose the speed of light were infinite. What would nor destroyed.” What must we say instead?
happen to the relativistic predictions of length contraction 20. Is our intuitive notion that velocities simply add, as in
and time dilation? Section 3–8, completely wrong?

MisConceptual Questions
1. The fictional rocket ship Adventure is measured to be 50 m 4. The rocket ship of MisConceptual Question 1 travels to a star
long by the ship’s captain inside the rocket. When the rocket many light-years away, then turns around and returns at the
moves past a space dock at 0.5c, space-dock personnel same speed. When it returns to the space dock, who would
measure the rocket ship to be 43.3 m long. What is its have aged less: the space-dock personnel or ship’s captain?
proper length? (a) The space-dock personnel.
(a) 50 m. (b) 43.3 m. (c) 93.3 m. (d) 13.3 m. (b) The ship’s captain.
(c) Both the same amount, because both sets of people
2. As rocket ship Adventure (MisConceptual Question 1) were moving relative to each other.
passes by the space dock, the ship’s captain flashes a flash- (d) We need to know how far away the star is.
light at 1.00-s intervals as measured by space-dock personnel. 5. An Earth observer notes that clocks on a passing space-
How often does the flashlight flash relative to the captain? craft run slowly. The person on the spacecraft
(a) Every 1.15 s. (b) Every 1.00 s. (c) Every 0.87 s. (a) agrees her clocks move slower than those on Earth.
(d) We need to know the distance between the ship and (b) feels normal, and her heartbeat and eating habits are
the space dock. normal.
(c) observes that Earth clocks are moving slowly.
3. For the flashing of the flashlight in MisConceptual Ques- (d) The real time is in between the times measured by the
tion 2, what time interval is the proper time interval? two observers.
(a) 1.15 s. (b) 1.00 s. (c) 0.87 s. (d) 0.13 s. (e) Both (a) and (b).
(f) Both (b) and (c).
766 CHAPTER 26 The Special Theory of Relativity
6. Spaceships A and B are traveling directly toward each 10. You are in a rocket ship going faster and faster. As your
other at a speed 0.5c relative to the Earth, and each has speed increases and your velocity gets closer to the speed of
a headlight aimed toward the other ship. What value do light, which of the following do you observe in your frame
technicians on ship B get by measuring the speed of the of reference?
light emitted by ship A’s headlight? (a) Your mass increases.
(a) 0.5c. (b) 0.75c. (c) 1.0c. (d) 1.5c. (b) Your length shortens in the direction of motion.
7. Relativistic formulas for time dilation, length contraction, (c) Your wristwatch slows down.
and mass are valid (d) All of the above.
(a) only for speeds less than 0.10c. (e) None of the above.
(b) only for speeds greater than 0.10c. 11. You are in a spaceship with no windows, radios, or other
(c) only for speeds very close to c. means to check outside. How could you determine whether
(d) for all speeds. your spaceship is at rest or moving at constant velocity?
8. Which of the following will two observers in inertial refer- (a) By determining the apparent velocity of light in the
ence frames always agree on? (Choose all that apply.) spaceship.
(a) The time an event occurred. (b) By checking your precision watch. If it’s running slow,
(b) The distance between two events. then the ship is moving.
(c) The time interval between the occurence of two events. (c) By measuring the lengths of objects in the spaceship. If
(d) The speed of light. they are shortened, then the ship is moving.
(e) The validity of the laws of physics. (d) Give up, because you can’t tell.
(f) The simultaneity of two events. 12. The period of a pendulum attached in a spaceship is 2 s
9. Two observers in different inertial reference frames moving while the spaceship is parked on Earth. What is the period
relative to each other at nearly the speed of light see to an observer on Earth when the spaceship moves at 0.6c
the same two events but, using precise equipment, record with respect to the Earth?
different time intervals between the two events. Which of (a) Less than 2 s.
the following is true of their measurements? (b) More than 2 s.
(a) One observer is incorrect, but it is impossible to tell (c) 2 s.
which one.
13. Two spaceships, each moving at a speed 0.75c relative to
(b) One observer is incorrect, and it is possible to tell
the Earth, are headed directly toward each other. What do
which one.
occupants of one ship measure the speed of other ship to be?
(c) Both observers are incorrect.
(a) 0.96c. (b) 1.0c. (c) 1.5c. (d) 1.75c. (e) 0.75c.
(d) Both observers are correct.

For assigned homework and other learning materials, go to the MasteringPhysics website.

Problems
26–4 and 26–5 Time Dilation, Length Contraction 9. (II) A friend speeds by you in her spacecraft at a speed of
0.720c. It is measured in your frame to be 4.80 m long and
1. (I) A spaceship passes you at a speed of 0.850c. You measure
1.35 m high. (a) What will be its length and height at rest?
its length to be 44.2 m. How long would it be when at rest?
(b) How many seconds elapsed on your friend’s watch when
2. (I) A certain type of elementary particle travels at a speed 20.0 s passed on yours? (c) How fast did you appear to be
of 2.70 * 108 m兾s. At this speed, the average lifetime is traveling according to your friend? (d) How many seconds
measured to be 4.76 * 10 –6 s. What is the particle’s lifetime elapsed on your watch when she saw 20.0 s pass on hers?
at rest?
10. (II) A star is 21.6 light-years from Earth. How long would
3. (II) You travel to a star 135 light-years from Earth at a speed it take a spacecraft traveling 0.950c to reach that star as
of 2.90 * 108 m兾s. What do you measure this distance to be? measured by observers: (a) on Earth, (b) on the spacecraft?
4. (II) What is the speed of a pion if its average lifetime is (c) What is the distance traveled according to observers
measured to be 4.40 * 10 –8 s? At rest, its average lifetime on the spacecraft? (d) What will the spacecraft occupants
is 2.60 * 10 –8 s. compute their speed to be from the results of (b) and (c)?
5. (II) In an Earth reference frame, a star is 49 light-years 11. (II) A fictional news report stated that starship Enterprise
away. How fast would you have to travel so that to you the had just returned from a 5-year voyage while traveling at
distance would be only 35 light-years? 0.70c. (a) If the report meant 5.0 years of Earth time, how
6. (II) At what speed v will the length of a 1.00-m stick look much time elapsed on the ship? (b) If the report meant
10.0% shorter (90.0 cm)? 5.0 years of ship time, how much time passed on Earth?
7. (II) At what speed do the relativistic formulas for (a) length 12. (II) A box at rest has the shape of a cube 2.6 m on a side.
and (b) time intervals differ from classical values by 1.00%? This box is loaded onto the flat floor of a spaceship and
(This is a reasonable way to estimate when to use relativistic the spaceship then flies past us with a horizontal speed of
calculations rather than classical.) 0.80c. What is the volume of the box as we observe it?
8. (II) You decide to travel to a star 62 light-years from Earth 13. (III) Escape velocity from the Earth is 11.2 km兾s. What
at a speed that tells you the distance is only 25 light- would be the percent decrease in length of a 68.2-m-long
years. How many years would it take you to make the trip? spacecraft traveling at that speed as seen from Earth?

Problems 767
14. (III) An unstable particle produced in an accelerator 33. (II) How much energy can be obtained from conversion of
experiment travels at constant velocity, covering 1.00 m in 1.0 gram of mass? How much mass could this energy raise
3.40 ns in the lab frame before changing (“decaying”) into to a height of 1.0 km above the Earth’s surface?
other particles. In the rest frame of the particle, determine 34. (II) To accelerate a particle of mass m from rest to speed
(a) how long it lived before decaying, (b) how far it moved 0.90c requires work W1 . To accelerate the particle from
before decaying. speed 0.90c to 0.99c requires work W2 . Determine the
15. (III) How fast must a pion be moving on average to travel ratio W2兾W1 .
32 m before it decays? The average lifetime, at rest, is 35. (II) Suppose there was a process by which two photons,
2.6 * 10–8 s. each with momentum 0.65 MeV兾c, could collide and make
a single particle. What is the maximum mass that the parti-
26–7 Relativistic Momentum cle could possess?
16. (I) What is the momentum of a proton traveling at 36. (II) What is the speed of a proton accelerated by a poten-
v = 0.68c? tial difference of 165 MV?
17. (II) (a) A particle travels at v = 0.15c. By what percentage 37. (II) What is the speed of an electron after being accelerated
will a calculation of its momentum be wrong if you use the from rest by 31,000 V?
classical formula? (b) Repeat for v = 0.75c. 38. (II) The kinetic energy of a particle is 45 MeV. If the
momentum is 121 MeV兾c, what is the particle’s mass?
18. (II) A particle of mass m travels at a speed v = 0.22c. At
what speed will its momentum be doubled? 39. (II) Calculate the speed of a proton Am = 1.67 * 10–27 kgB
whose kinetic energy is exactly half (a) its total energy,
19. (II) An unstable particle is at rest and suddenly decays into (b) its rest energy.
two fragments. No external forces act on the particle or its
40. (II) Calculate the kinetic energy and momentum of a
fragments. One of the fragments has a speed of 0.60c and
proton Am = 1.67 * 10 –27 kgB traveling 8.65 * 107 m兾s.
a mass of 6.68 * 10–27 kg, while the other has a mass of
By what percentages would your calculations have been in
1.67 * 10 –27 kg. What is the speed of the less massive
error if you had used classical formulas?
fragment?
41. (II) Suppose a spacecraft of mass 17,000 kg is accelerated
20. (II) What is the percent change in momentum of a proton to 0.15c. (a) How much kinetic energy would it have?
that accelerates from (a) 0.45c to 0.85c, (b) 0.85c to 0.98c? (b) If you used the classical formula for kinetic energy, by
what percentage would you be in error?
26–9 E=mc 2; Mass and Energy
42. (II) A negative muon traveling at 53% the speed of light
21. (I) Calculate the rest energy of an electron in joules and in collides head on with a positive muon traveling at 65% the
MeV A1 MeV = 1.60 * 10–13 JB. speed of light. The two muons (each of mass 105.7 MeV兾c2)
22. (I) When a uranium nucleus at rest breaks apart in the annihilate, and produce how much electromagnetic energy?
process known as fission in a nuclear reactor, the resulting 43. (II) Two identical particles of mass m approach each other
fragments have a total kinetic energy of about 200 MeV. at equal and opposite speeds, v. The collision is completely
How much mass was lost in the process? inelastic and results in a single particle at rest. What is the
23. (I) The total annual energy consumption in the United mass of the new particle? How much energy was lost in the
States is about 1 * 1020 J. How much mass would have to collision? How much kinetic energy was lost in this collision?
be converted to energy to fuel this need? 44. (III) The americium nucleus, 24195 Am, decays to a neptunium

24. (I) Calculate the mass of a proton (1.67 * 10–27 kg) in nucleus, 23793 Np, by emitting an alpha particle of mass

MeV兾c2. 4.00260 u and kinetic energy 5.5 MeV. Estimate the mass
of the neptunium nucleus, ignoring its recoil, given that the
25. (I) A certain chemical reaction requires 4.82 * 104 J of americium mass is 241.05682 u.
energy input for it to go. What is the increase in mass of
45. (III) Show that the kinetic energy ke of a particle of mass m
the products over the reactants?
is related to its momentum p by the equation
26. (II) Calculate the kinetic energy and momentum of a
p = 3ke2 + 2ke mc2兾c.
proton traveling 2.90 * 108 m兾s.
27. (II) What is the momentum of a 950-MeV proton (that is, *46. (III) What magnetic field B is needed to keep 998-GeV
its kinetic energy is 950 MeV)? protons revolving in a circle of radius 1.0 km? Use the
relativistic mass. The proton’s “rest mass” is 0.938 GeV兾c2.
28. (II) What is the speed of an electron whose kinetic energy A1 GeV = 109 eV.B [Hint: In relativity, mrel v2兾r = qvB is
is 1.12 MeV? still valid in a magnetic field, where mrel = gm. ]
29. (II) (a) How much work is required to accelerate a proton
from rest up to a speed of 0.985c? (b) What would be the 26–10 Relativistic Addition of Velocities
momentum of this proton?
47. (I) A person on a rocket traveling at 0.40c (with respect to
30. (II) At what speed will an object’s kinetic energy be 33% of
the Earth) observes a meteor come from behind and pass
its rest energy?
her at a speed she measures as 0.40c. How fast is the
31. (II) Determine the speed and the momentum of an electron meteor moving with respect to the Earth?
Am = 9.11 * 10–31 kgB whose ke equals its rest energy. 48. (II) Two spaceships leave Earth in opposite directions, each
32. (II) A proton is traveling in an accelerator with a speed of with a speed of 0.60c with respect to Earth. (a) What is the
1.0 * 108 m兾s. By what factor does the proton’s kinetic velocity of spaceship 1 relative to spaceship 2? (b) What
energy increase if its speed is doubled? is the velocity of spaceship 2 relative to spaceship 1?

768 CHAPTER 26 The Special Theory of Relativity


49. (II) A spaceship leaves Earth traveling at 0.65c. A second 51. (II) A spaceship in distress sends out two escape pods in
spaceship leaves the first at a speed of 0.82c with respect to opposite directions. One travels at a speed v1 = ±0.70c
the first. Calculate the speed of the second ship with in one direction, and the other travels at a speed
respect to Earth if it is fired (a) in the same direction the v2 = –0.80c in the other direction, as observed from the
first spaceship is already moving, (b) directly backward spaceship. What speed does the first escape pod measure
toward Earth. for the second escape pod?
50. (II) An observer on Earth sees an alien vessel approach 52. (II) Rocket A passes Earth at a speed of 0.65c. At the
at a speed of 0.60c. The fictional starship Enterprise comes same time, rocket B passes Earth moving 0.95c relative to
to the rescue (Fig. 26–13), overtaking the aliens while Earth in the same direction as A. How fast is B moving
moving directly toward Earth at a speed of 0.90c relative relative to A when it passes A?
to Earth. What is the relative speed of one vessel as seen
by the other? 53. (II) Your spaceship, traveling at 0.90c, needs to launch a
Enterprise probe out the forward hatch so that its speed relative to
the planet that you are approaching is 0.95c. With what
speed must it leave your ship?
= 0.90c
= 0.60c

FIGURE 26–13 Problem 50.

General Problems
54. What is the speed of a particle when its kinetic energy 62. An electron Am = 9.11 * 10–31 kgB is accelerated from
equals its rest energy? Does the mass of the particle affect rest to speed v by a conservative force. In this process, its
the result? potential energy decreases by 6.20 * 10–14 J. Determine
55. The nearest star to Earth is Proxima Centauri, 4.3 light- the electron’s speed, v.
years away. (a) At what constant velocity must a spacecraft 63. The Sun radiates energy at a rate of about 4 * 1026 W.
travel from Earth if it is to reach the star in 4.9 years, as (a) At what rate is the Sun’s mass decreasing? (b) How
measured by travelers on the spacecraft? (b) How long does long does it take for the Sun to lose a mass equal to that
the trip take according to Earth observers? of Earth? (c) Estimate how long the Sun could last if it
56. According to the special theory of relativity, the factor g radiated constantly at this rate.
that determines the length contraction and the time 64. How much energy would be required to break a helium
dilation is given by g = 1兾31 - v2兾c2 . Determine the nucleus into its constituents, two protons and two
numerical values of g for an object moving at speed neutrons? The masses of a proton (including an electron),
v = 0.01c, 0.05c, 0.10c, 0.20c, 0.30c, 0.40c, 0.50c, 0.60c, a neutron, and neutral helium are, respectively, 1.00783 u,
0.70c, 0.80c, 0.90c, 0.95c, and 0.99c. Make a graph of 1.00867 u, and 4.00260 u. (This energy difference is called
g versus v. the total binding energy of the 42He nucleus.)
57. A healthy astronaut’s heart rate is 60 beats兾min. Flight 65. Show analytically that a particle with momentum p and
doctors on Earth can monitor an astronaut’s vital signs energy E has a speed given by
remotely while in flight. How fast would an astronaut be pc2 pc
flying away from Earth if the doctor measured her having v = = .
E 3m c + p
2 2 2
a heart rate of 25 beats兾min?
58. (a) What is the speed v of an electron whose kinetic energy 66. Two protons, each having a speed of 0.990c in the labora-
is 14,000 times its rest energy? You can state the answer as tory, are moving toward each other. Determine (a) the
the difference c - v. Such speeds are reached in the Stan- momentum of each proton in the laboratory, (b) the total
ford Linear Accelerator, SLAC. (b) If the electrons travel momentum of the two protons in the laboratory, and
in the lab through a tube 3.0 km long (as at SLAC), how (c) the momentum of one proton as seen by the other proton.
long is this tube in the electrons’ reference frame? [Hint: 67. When two moles of hydrogen molecules (H 2) and one mole
Use the binomial expansion.] of oxygen molecules (O2) react to form two moles of water
59. What minimum amount of electromagnetic energy is (H 2O), the energy released is 484 kJ. How much does the
needed to produce an electron and a positron together? mass decrease in this reaction? What % of the total original
A positron is a particle with the same mass as an electron, mass is this?
but has the opposite charge. (Note that electric charge is 68. The fictional starship Enterprise obtains its power by
conserved in this process. See Section 27–6.) combining matter and antimatter, achieving complete
60. How many grams of matter would have to be totally conversion of mass into energy. If the mass of the
destroyed to run a 75-W lightbulb for 1.0 year? Enterprise is approximately 6 * 109 kg, how much mass
61. A free neutron can decay into a proton, an electron, and a must be converted into kinetic energy to accelerate it from
neutrino. Assume the neutrino’s mass is zero; the other rest to one-tenth the speed of light?
masses can be found in the Table inside the front cover. 69. Make a graph of the kinetic energy versus momentum for
Determine the total kinetic energy shared among the three (a) a particle of nonzero mass, and (b) a particle with zero
particles when a neutron decays at rest. mass.
General Problems 769
70. A spaceship and its occupants have a total mass of 76. Astronomers measure the distance to a particular star to
160,000 kg. The occupants would like to travel to a star be 6.0 light-years (1 ly = distance light travels in 1 year).
that is 35 light-years away at a speed of 0.70c. To accelerate, A spaceship travels from Earth to the vicinity of this star at
the engine of the spaceship changes mass directly to energy. steady speed, arriving in 3.50 years as measured by clocks
(a) Estimate how much mass will be converted to energy on the spaceship. (a) How long does the trip take as
to accelerate the spaceship to this speed. (b) Assuming the measured by clocks in Earth’s reference frame? (b) What
acceleration is rapid, so the speed for the entire trip can be distance does the spaceship travel as measured in its own
taken to be 0.70c, determine how long the trip will take reference frame?
according to the astronauts on board. 77. An electron is accelerated so that its kinetic energy is
71. In a nuclear reaction two identical particles are created, greater than its rest energy mc2 by a factor of (a) 5.00,
traveling in opposite directions. If the speed of each particle (b) 999. What is the speed of the electron in each case?
is 0.82c, relative to the laboratory frame of reference, what 78. You are traveling in a spaceship at a speed of 0.70c away
is one particle’s speed relative to the other particle? from Earth. You send a laser beam toward the Earth
traveling at velocity c relative to you. What do observers
72. A 36,000-kg spaceship is to travel to the vicinity of a star
6.6 light-years from Earth. Passengers on the ship want the on the Earth measure for the speed of the laser beam?
(one-way) trip to take no more than 1.0 year. How much 79. A farm boy studying physics believes that he can fit a
work must be done on the spaceship to bring it to the 13.0-m-long pole into a 10.0-m-long barn if he runs fast
speed necessary for this trip? enough, carrying the pole. Can he do it? Explain in detail.
How does this fit with the idea that when he is running the
73. Suppose a 14,500-kg spaceship left Earth at a speed of 0.90c. barn looks even shorter than 10.0 m?
What is the spaceship’s kinetic energy? Compare with the
80. An atomic clock is taken to the North Pole, while another
total U.S. annual energy consumption (about 1020 J).
stays at the Equator. How far will they be out of synchroni-
74. A pi meson of mass mp decays at rest into a muon (mass mm) zation after 2.0 years has elapsed? [Hint: Use the binomial
and a neutrino of negligible or zero mass. Show that the expansion, Appendix A.]
kinetic energy of the muon is kem = Amp - mm B 2c2兾 A2mp B . 81. An airplane travels 1300 km兾h around the Earth in a circle
75. An astronaut on a spaceship traveling at 0.75c relative to of radius essentially equal to that of the Earth, returning
Earth measures his ship to be 23 m long. On the ship, he to the same place. Using special relativity, estimate the
eats his lunch in 28 min. (a) What length is the spaceship difference in time to make the trip as seen by Earth and
according to observers on Earth? (b) How long does the by airplane observers. [Hint: Use the binomial expansion,
astronaut’s lunch take to eat according to observers on Earth? Appendix A.]

Search and Learn


1. Determine about how fast Mr Tompkins is traveling in the 4. In Example 26–5, the spaceship is moving at 0.90c in the
Chapter-Opening Photograph. Do you agree with the horizontal direction relative to an observer on the Earth.
picture in terms of the way Mr Tompkins would see the If instead the spaceship moved at 0.90c directed at 30°
world? Explain. [Hint: Assume the bank clock and Stop above the horizontal, what would be the painting’s
sign facing us are round according to the people on the dimensions as seen by the observer on Earth?
sidewalk.] 5. Protons from outer space crash into the Earth’s atmosphere
at a high rate. These protons create particles that eventually
2. Examine the experiment of Fig. 26–5 from O1’s reference
decay into other particles called muons. This cosmic debris
frame. In this case, O1 will be at rest and will see the light-
travels through the atmosphere. Every second, dozens of
ning bolt at B1 and B2 , before the lightning bolt at A 1
muons pass through your body. If a muon is created 30 km
and A 2 . Will O1 recognize that O2 , who is moving with
above the Earth’s surface, what minimum speed and kinetic
speed v to the left, will see the two events as simultaneous?
energy must the muon have in order to hit Earth’s surface?
Explain in detail, drawing diagrams equivalent to Fig. 26–5.
A muon’s mean lifetime (at rest) is 2.20 ms and its mass is
[Hint: Include length contraction.]
105.7 MeV兾c2.
3. Using Example 26–2 as a guide, show that for objects that 6. As a rough rule, anything traveling faster than about 0.1c
move slowly in comparison to c, the length contraction is called relativistic—that is, special relativity is a significant
formula is roughly l L l0 A1 - 12 v2兾c2 B. Use this approxi- effect. Determine the speed of an electron in a hydrogen
mation to find the “length shortening” ¢l = l0 - l of the atom (radius 0.53 * 10–10 m ) and state whether or not it
train in Example 26–6 if the train travels at 100 km兾h is relativistic. (Treat the electron as though it were in a
(rather than 0.92c). circular orbit around the proton. See hint for Problem 46.)

A N S W E R S TO E X E R C I S E S
A: (c). E: 0.030c, same as classical, to an accuracy of better than
B: (a) No; (b) yes. 0.1%.
C: 80 m. F: (d).
D: No: ke兾mc2 L 2 * 10–4.

770 CHAPTER 26 The Special Theory of Relativity

También podría gustarte